CHUYÊN đề TOÁN học bắt đàu từ NHỮNG TRƯỜNG hợp NHỎ

271 1.2K 0
CHUYÊN đề TOÁN học bắt đàu từ NHỮNG TRƯỜNG hợp NHỎ

Đang tải... (xem toàn văn)

Tài liệu hạn chế xem trước, để xem đầy đủ mời bạn chọn Tải xuống

Thông tin tài liệu

HỘI CÁC TRƯỜNG THPT CHUYÊN KHU VỰC DUYÊN HẢI - ĐỒNG BẰNG BẮC BỘ HỘI THẢO KHOA HỌC LẦN THỨ VI LỜI NÓI ĐẦU Trong hành trình phát triển của nền giáo dục Việt Nam, hệ thống các trường THPT chuyên ngày càng khẳng định được vị thế quan trọng của mình trong việc phát hiện, tuyển chọn và bồi dưỡng nhân tài, chắp cánh những ước mơ bay cao, bay xa tới chân trời của tri thức và thành công. Đối với các trường THPT chuyên, công tác học sinh giỏi luôn được đặt lên hàng đầu, là nhiệm vụ trọng tâm của mỗi năm học. Hội thảo khoa học các trường THPT chuyên Khu vực Duyên Hải và Đồng bằng Bắc Bộ là một hoạt động bổ ích diễn ra vào tháng 11 thường niên. Đây là dịp gặp gỡ, giao lưu, học hỏi, trao đổi kinh nghiệm giảng dạy, phát hiện, tuyển chọn và bồi dưỡng đội tuyển học sinh giỏi Quốc gia, Quốc tế giữa các trường THPT chuyên trong khu vực. Năm năm qua, các hội thảo khoa học đều nhận được sự hưởng ứng nhiệt tình của các trường, bước đầu đã đem đến những hiệu ứng tốt, tác động không nhỏ đến công tác bồi dưỡng học sinh giỏi và chất lượng đội tuyển học sinh giỏi quốc gia của các trường Chuyên. Năm 2013 là năm thứ 6, hội thảo khoa học của Hội các trường THPT chuyên Khu vực Duyên hải và Đồng bằng Bắc Bộ được tổ chức tại Thái Bình mảnh đất quê lúa, mang trong mình truyền thống yêu nước và truyền thống hiếu học. Tại hội thảo lần này, chúng tôi chủ trương tập trung vào những vấn đề mới mẻ, thiết thực và có ý nghĩa đối với việc bồi dưỡng học sinh giỏi, để quý thầy cô đã, đang và sẽ đảm nhiệm công tác này tiếp tục trao đổi, học tập, nâng cao hơn nữa năng lực chuyên môn của mình. Tập tài liệu của Hội thảo lần thứ VI bao gồm những chuyên đề khoa học đạt giải của quý thầy cô trong Hội các trường THPT chuyên Khu vực Duyên hải và Đồng bằng Bắc bộ. Các bài viết đều tập trung vào những vấn đề trọng tâm đã được hội đồng khoa học trường THPT chuyên Thái Bình thống nhất trong nội dung hội thảo. Nhiều chuyên đề thực sự là những công trình khoa học tâm huyết, say mê của quý thầy cô, tạo điểm nhấn quan trọng cho diễn đàn, có thể coi là những tư liệu quý cho các trường trong công tác bồi dưỡng học sinh giỏi. Xin chân thành cảm ơn sự cộng tác của quý thầy cô đến từ các trường THPT chuyên Khu vực Duyên hải và Đồng bằng Bắc Bộ cùng các trường THPT chuyên với vai trò quan sát viên. Chúng tôi hy vọng, sẽ tiếp tục nhận được nhiều hơn nữa sự phản hồi, đóng góp, trao đổi của quý thầy cô để các chuyên đề khoa học hoàn thiện hơn. Thái Bình, tháng 11 năm 2013 TR­êng THPT Chuyªn th¸i b×nh Trường THPT Chuyên Thái Bình 1 HỘI CÁC TRƯỜNG THPT CHUYÊN KHU VỰC DUYÊN HẢI - ĐỒNG BẰNG BẮC BỘ HỘI THẢO KHOA HỌC LẦN THỨ VI Chuyên đề xếp loại xuất sắc BẮT ĐẦU TỪ NHỮNG TRƯỜNG HỢP NHỎ Nguyễn Thế Sinh Giáo viên THPT chuyên Nguyễn Trãi, Hải Dương ******************** Toán rời rạc luôn là một bài toán rất khó trong bất kỳ đề thi nào, đặc biệt với học sinh Việt Nam-những người ít được học một cách bài bản về mảng này của môn Toán. Cho dù có một số lượng khá lớn các bài toán không cần nhiều kiến thức đồ sộ mà chỉ cần những kiến thức rất tầm thường, đơn giản như nguyên lý quy nạp, như các quy tắc đếm cơ bản, … nhưng vẫn khiến học sinh phải cảm thấy ái ngại khi đứng trước nó. Chính vì thế, tôi muốn đề cập đến một hướng tiếp cận các bài toán rời rạc mà nếu không giải được bài toán thông qua cách tiếp cận này thì chí ít học sinh còn hiểu nổi đề bài. Hướng tiếp cận này cũng vô cùng đơn giản, đó là bắt đầu với những ví dụ, những trường hợp nhỏ, cụ thể. Tuy vậy, cách làm này đòi hỏi sự kiên nhẫn và tất nhiên cũng đòi hỏi một chút sự nhạy cảm Toán học, để phát hiện ra quy luật cũng như những ý tưởng lóe lên trong các trường hợp ấy. Các bài toán mà tôi đưa ra sau đây là các ví dụ minh họa cho điều này, các trường hợp nhỏ lẻ nhiều khi chứa đựng luôn phương pháp giải, nhưng cũng có khi chỉ chứa đựng một ý tưởng làm tiền đề cho lời giải, hoặc có khi chỉ giúp tháo gỡ một vài điểm nút trong một lời giải được xem từ phương diện tổng quát. Nhưng chắc chắn một điều, các ví dụ này đều giúp ta hiểu rõ đề bài hơn, qua đó giúp ta có dũng khí đương đầu với nó. Ngoài ra, trong bài viết này, khi làm việc với các trường hợp nhỏ, tôi sẽ dùng cỡ chữ nhỏ hơn một chút để các bạn có thể tiện theo dõi, phân biệt được lời giải cuối cùng và những dẫn dắt dẫn đến lời giải đó. Bài 1. Với mỗi n, xét tập hợp Sn = {1, 2,3,…, n} . Tìm số tập hợp con khác rỗng của Sn, không chứa 2 số nguyên liên tiếp tùy theo n. Lời giải: Trước hết, ta bắt đầu với những trường hợp nhỏ +) Với n=1, ta được 1 tập hợp thỏa mãn: {1} Trường THPT Chuyên Thái Bình 2 HỘI CÁC TRƯỜNG THPT CHUYÊN KHU VỰC DUYÊN HẢI - ĐỒNG BẰNG BẮC BỘ HỘI THẢO KHOA HỌC LẦN THỨ VI +) Với n=2, ta được 2 tập thỏa mãn: +) Với n=3, ta được 4 tập thỏa mãn: +) Với n=4, ta được 7 tập thỏa mãn: {4};{1,3};{1,4};{2,4} {1}; {2} {1}; {2}; {3}; {1,3} {1}; {2}; {3}; +) Với n=5, ta được 12 tập thỏa mãn: {1}; {2}; {3};{4};{5};{1,3};{1,4};{1,5};{2,4};{2,5};{3,5};{1,3,5} Bây giờ, quan sát sự thay đổi các tập thu được khi n tăng dần, ta có thể thấy khi n tăng lên 1 đơn vị, ngoài các tập thu được ứng với giá trị n trước đó, thì còn các tập mới được tạo ra bằng cách thêm chính số n vào, tuy nhiên chỉ được phép thêm vào những tập không chứa số n-1. Chẳng hạn: Với n=4, các tập là: {1}; {2}; {3}; {4};{1,3};{1,4};{2,4} Thì với n=5, vẫn có các tập đó, nhưng thêm vào các tập {5}, tạo từ tập ∅ bằng cách thêm số 5 {1,5};{2,5};{3,5}, tạo từ các tập {1};{2};{3} bằng cách thêm vào số 5 {1,3,5}, tạo từ tập {1,3} bằng cách thêm vào số 5 Những tập chứa số 4 không tạo được tập nào thỏa mãn. Như vậy, tổng quát hóa suy nghĩ này, ta thu được lời giải nhờ truy hồi ( từ trường hợp trước tạo ra trường hợp sau) như sau: ******* Gọi an là số tập con tập Sn thỏa mãn đề bài, ta có a1 = 1, a 2 = 2, a3 = 4, a 4 = 7, a5 = 12 Xét với tập Sn+1 , một tập con thỏa mãn thuộc 1 trong 2 loại Loại 1: Không chứa n+1, có an tập loại này Loại 2: Chứa n+1, các tập loại này được tạo ra nhờ thêm vào n+1 từ các tập con của Sn, thỏa mãn đề bài, nhưng các tập con này không chứa n, đó chính là các tập con của Sn−1 thỏa mãn đề bài. Ngoài ra có thêm tập {n+1}. Vậy có an −1 + 1 tập loại này Từ đó ta được: an +1 = an + an −1 + 1, n ≥ 2 , a1 = 1, a2 = 2 Giải phương trình sai phân thu được ở trên, ta được: Trường THPT Chuyên Thái Bình 3 HỘI CÁC TRƯỜNG THPT CHUYÊN KHU VỰC DUYÊN HẢI - ĐỒNG BẰNG BẮC BỘ HỘI THẢO KHOA HỌC LẦN THỨ VI an = 5 − 3 1− 5 n 5 + 3 1+ 5 n ( ) + ( ) −1 2 2 2 5 2 5 Với kiểu lập luận tương tự trên, ta có thể có lời giải cho bài toán sau: Bài 2. Xét 1 hoán vị ( x1, x2 ,…, xn ) của tập S n = {1, 2, … , n} . Vị trí i,1 ≤ i ≤ n được gọi là vị trí cực đại nếu xi > xi −1 , xi > xi +1 ( vị trí 1 và n không phải là vị trí cực đại). Gọi p(n,k) là số hoán vị của Sn có đúng k vị trí cực đại. Chứng minh rằng: p ( n + 1 , k + 1 ) = ( 2 k + 4 ) p ( n , k + 1 ) + ( n − 2 k + 1 ) p ( n , k ) Lời giải: Nhận xét: Nếu i là vị trí cực đại của ( x1, x2 ,…, xn ) thì vị trí cực đại. Gọi p ( n , k ) i − 1, i + 1 không phải là là số hoán vị của Sn có đúng k vị trí cực đại. Xét ( x1, x2 ,…, xn ) là 1 hoán vị của Sn, có k vị trí cực đại i1 < i2 Nếu là số nguyên thì Mà a − b a − 1 − b và [ a ] − [ b ] = a − b a − 1 − [ b ] ≥ 1 1 n ( n + 1) − ( n − 5 ) ( n − 6 ) = 2 n − 5 6 6 [ a ] − [ b ] ≤ a − [ b ] < a − ( b − 1 ) = a − b + 1 là số nguyên nên k − k1 = 2n − 5 Vậy số phần tử màu xanh ( đỏ) ngoài X n−6 đều bằng 2n-5. Có | X n | − | X n − 6 |= 6 n − 15 nên số phần tử màu trắng ngoài X n−6 là 6n-152(2n-5)=2n-5 Khi đó, ta xây dựng các tập An −5 , An − 4 , An −3 , An − 2 , An −1 , An như sau: Tập An − 5 , An chứa toàn phần tử màu xanh Tập An−4 , An−1 chứa toàn phần tử màu đỏ Tập An−2 , An−3 chứa toàn phần tử màu trắng Bài 9. [IMO shortlisted 2012] Cho n ≥ 1 tử của tập { 1 , 2 , … là số nguyên. Tìm số lớn nhất các tập con rời nhau, có 2 phần , n } sao cho i) Tổng các phần tử của 1 tập không vượt quá n ii) Tổng các phần tử của các tập khác nhau là các số nguyên dương khác nhau Trường THPT Chuyên Thái Bình 19 HỘI CÁC TRƯỜNG THPT CHUYÊN KHU VỰC DUYÊN HẢI - ĐỒNG BẰNG BẮC BỘ HỘI THẢO KHOA HỌC LẦN THỨ VI Lời giải: Giả sử k là số lớn nhất các cặp tập rời nhau thỏa mãn đề bài Ví dụ: +) Với n=3, chỉ có thể có 1 tập con duy nhất có 2 phần tử mà tổng các phần tử không quá 3 là tập {1,2} nên k=1 với tập {1,2} +) Với n=4, có 2 tập {1,2}, {1,3} thỏa mãn tổng các phần tử không quá 4, nhưng 2 tập này không rời nhau nên k=1, với tập {1,2} +) Với n=5, có các tập {1,2}; {1,3}; {1,4}; {2,3} thỏa mãn, nhưng 3 tập đầu có chung phần tử nên chỉ chọn được 1 tập, nên k ≤ 2 , nhưng với k=2 chỉ có thể chọn 2 tập {2,3}; {1,4}, 2 tập này có cùng tổng nên cuối cùng k=1, với tập {1,2} +) Với n=6, có các tập {1,2},{1,3},{1,4},{1,5}- chọn được 1 trong 4 tập Và các tập {2,3},{2,4}- chọn được 1 trong 2 tập Nên k=2 với các tập {2,4}; {1,3} hoặc {2,3}; {1,4}, nhưng chỉ cặp {2,4},{1,3} cho tổng khác nhau. Vậy k=2, với các tập {2,4},{1,3} +) Với n=7, có các tập {1,2},{1,3},{1,4}, {1,5}, {1,6}- chọn được 1 tập Các tập {2,3},{2,4},{2,5}- chọn được 1 tập Tập {3,4} nên k ≤ 3 , với các tập {3,4}; {2,5}; {1,6}, tuy nhiên, chỉ chọn được 2 trong 3 tập này vì yêu cầu về tổng nên k ≤ 2 , với k=2, có thể chọn 2 tập {2,4} và {1,3}. Vậy k=2, chọn 2 tập {2,4} và {1,3} +) Với n=8, có các tập {1,2},{1,3}, … , {1,7}- chọn được 1 tập {2,3}, {2,4},{2,5},{2,6} chọn được 1 tập {3,4}, {3,5} chọn được 1 tập Với k=3 với các tập là {3,4}, {2,6},{1,2} chẳng hạn Trường THPT Chuyên Thái Bình 20 HỘI CÁC TRƯỜNG THPT CHUYÊN KHU VỰC DUYÊN HẢI - ĐỒNG BẰNG BẮC BỘ HỘI THẢO KHOA HỌC LẦN THỨ VI Tiếp tục như vậy, ta có bảng sau N 1 2 3 4 5 6 7 8 9 10 11 12 13 14 15 16 17 18 19 20 k 0 0 1 1 1 2 2 3 3 3 4 4 5 5 5 6 6 7 7 7 Theo bảng, có thể dự đoán quy luật lặp lại theo chu kì 5 và với n=5m+1, n=5m+2 cho đáp số k=2m, n=5m+3, 5m+4, 5m+5 cho đáp số 2m+1. Từ đó, ta sẽ nghĩ cách tổng quát để phân hoạch tập { 1 , 2 , … , n } thành các tập 2 phần tử thỏa mãn đề bài trong từng trường hợp và chỉ ra với số k lớn hơn các số đã định thì không thể phân chia được. Ngoài ra, với 2 trường hợp n=5m+1, n=5m+2 có thể chọn chung các tập, 3 trường hợp n=5m+3, 5m+4, 5m+5 cũng có thể chọn chung các tập. Vậy chủ yếu làm việc với 2 trường hợp n=5m+1 và n=5m+3 Ta có lời giải sau: ******* Với n=5m+1,5m+2 ta có thể chọn 2m tập gồm {1,4m}, {3,4m-1}, {5, 4m-2},…, {2m+1,3m}- m tập {2,2m}- 1 tập {4,3m-1}, {6,3m-3},…, {2m-2,2m+2} – m-1 tập Ngoài ra nếu ≥ (1 + Và S S 2 ) + ≤ k ≥ 2m + 1 ( 3 + + ( 5 m 4 ) + 2 + 5 m + thì tổng S của k cặp đó thỏa mãn: + 1 + ( 4 m + 1 ) + + ( 3 m 4 m + 2 ) = + 3 + 3 m ( 2 m + + 1 ) ( 4 m 2 ) = ( 2 m + 3 ) + 1 ) ( 4 m + 2 ) Mâu thuẫn. Với n=5m+3,5m+3,5m+5, ta có thể chọn 2m+1 tập gồm {1,4m+2},{3,4m+3}, …, {2m+1, 3m+2}- m+1 tập {2, 3m+1}, {4, 3m}, …,{2m,2m+2}- m tập Ngoài ra nếu ≥ (1 + Và S S 2 ) + ≤ (5 m ( 3 + k ≥ 2 m + 2 4 ) + + 5 + 5 m + + thì tổng S của k cặp đó thỏa mãn: + 3 + ( 4 m 4 ) + + (3 m 4 m + 4 ) = + 5 + 3 m Mâu thuẫn. Vậy đáp số cuối cùng là Trường THPT Chuyên Thái Bình [ 2n − 1 ] 5 21 ( 2 m + 4 ) = + 2 ) ( 4 m ( 2 m + + 5 ) 2 )( 4 m + 4 ) HỘI CÁC TRƯỜNG THPT CHUYÊN KHU VỰC DUYÊN HẢI - ĐỒNG BẰNG BẮC BỘ HỘI THẢO KHOA HỌC LẦN THỨ VI Bài 10. Tại các đỉnh của một lục giác đều viết 6 số nguyên không âm có tổng bằng 2013. Một người thực hiện thay đổi như sau: Chọn 1 đỉnh, thay số ở đỉnh đó bởi giá trị tuyệt đối của hiệu 2 số viết ở 2 đỉnh kề với đỉnh được chọn. Chứng minh rằng, có thể thực hiện như vậy một số lần sao cho các số thu được ở 6 đỉnh đều bằng 0 Lời giải: Đương nhiên muốn giảm các số về 0 thì tổng các số phải giảm dần Xét vài trường hợp cụ thể. Dễ nhất là trường hợp có 3 số tại 3 đỉnh xen kẽ bằng 0, còn lại có tổng bằng 2013. Khi đó chỉ cần tác động 3 lần thao tác đã cho tại 3 đỉnh khác 0, ta thu được trạng thái 0 Nếu có 2 trong 3 số tại các đỉnh xen kẽ bằng 0, ta cũng dễ dàng thu được đáp số, cụ thể A B F A 0 0 A 0 0 A 0 D A 0 0 A 0 0 0 0 0 0 0 0 Ngoài ra, các số tại các đỉnh chỉ phụ thuộc vào các số tại 3 đỉnh chỉ sau 1 phép biến đổi, nên ta sẽ chỉ xem xét chủ yếu sự biến đổi dựa trên 3 số tại 3 đỉnh,chẳng hạn theo dõi sự biến đổi khi A ≥ C ≥ E như sau C-E A-C C A-E E C-E C-E A-C A A-C C A-E C-E A-E A-C C A-C C-E Ta thấy có phép biến đổi Trường THPT Chuyên Thái Bình 22 |A+E-2C| A-E E A A-C C A-E A-C A C-E E C-E HỘI CÁC TRƯỜNG THPT CHUYÊN KHU VỰC DUYÊN HẢI - ĐỒNG BẰNG BẮC BỘ HỘI THẢO KHOA HỌC LẦN THỨ VI A B F C E A-C A A-E C D E C-E A-E A-C C A-C C-E C-E làm giảm tính lẻ của tổng A+C+E nếu không có số nào bằng 0, là một phép biến đổi đáng lưu ý. Từ đây đặt ra câu hỏi: Khi có 1 số bằng 0 thì có đưa về trường hợp 2 số bằng 0 được 0, không có số nào bằng 0 thì thế nào? Trả lời các câu hỏi đó và xem xét biến đổi cụ thể trên, ta có lời giải sau: ******* Trước hết, tồn tại 3 đỉnh xen kẽ sao cho tổng là số lẻ ( bài toán chỉ sử dụng tính lẻ của số 2013 nên có thể tổng quát 2013 thành số n lẻ bất kỳ), giả sử 3 đỉnh đó là A, C, E ( được điền các số A,C,E luôn) +) Nếu A ≥ C ≥ E Xét >0 B A F A-C A C E đổi biến A-E C-E C D E sau A-C A-E C A-C C-E C-E Ta thấy phép biến đổi này làm giảm tổng A+C+E thành (C-E)+C+(AC)=A+C-E và vẫn giữ nguyên tính lẻ của tổng nếu E khác 0, vậy nếu áp dụng liên tiếp thao tác này, sẽ đi đến trạng thái mà 1 trong 3 số A,C,E bằng 0 hoặc 2 trong 3 số bằng 0 +) Nếu có 2 trong 3 số A,C,E bằng 0. Giả sử A>C=E=0, xét phép biến đổi A B F A 0 0 D A 0 A 0 0 Ta thu được trạng thái toàn số 0 Trường THPT Chuyên Thái Bình 0 23 A 0 0 A 0 0 0 0 0 0 0 HỘI CÁC TRƯỜNG THPT CHUYÊN KHU VỰC DUYÊN HẢI - ĐỒNG BẰNG BẮC BỘ HỘI THẢO KHOA HỌC LẦN THỨ VI +) Nếu A A ≥ C > E = 0 , thì do A lẻ nên A>C, xét phép biến đổi B F A C 0 A-C C A D 0 C A-C A C |A-2C| 0 C Ta thấy phép biến đổi này làm giảm thực sự tổng A+C+0 thành C+|A2C|< A+C ( do C >0 và C < A), trong khi vẫn giữ cho tổng lẻ, tức là nếu tiếp tục áp dụng biến đổi này, ta sẽ đến được trạng thái mà có 2 số bằng 0, lúc đó áp dụng phép biến đổi cho trường hợp trên, ta thu được kết quả. Vậy ta có điều phải chứng minh. Tôi xin tạm dừng bài viết của mình ở đây. Vì thời gian có hạn và kiến thức chuyên môn được nhìn nhận dưới góc nhìn chủ quan, nên có thể còn những điều bất hợp lý và cần phải chỉnh sửa lại, hoặc những điều có thể tiếp tục phát triển được, hay có thêm các ví dụ minh họa tiếp tục làm rõ thêm ý tưởng này, rất mong các bạn bè đồng nghiệp góp ý để bài viết thêm hoàn thiện. Mọi góp ý và bổ sung, minh họa liên quan xin gửi về hòm thư Sinhntsp83@gmail.com (Khi nhận được góp ý và bổ sung xong, tôi sẽ gửi lại bản đã hoàn thiện hơn cho các đồng nghiệp quan tâm, xin chân thành cảm ơn) Tài liệu tham khảo [1] Mathematics Olympiad Coach Seminar, China [2] The IMO Compendium –Springer [3] Web: Mathlinks.ro Trường THPT Chuyên Thái Bình 24 HỘI CÁC TRƯỜNG THPT CHUYÊN KHU VỰC DUYÊN HẢI - ĐỒNG BẰNG BẮC BỘ HỘI THẢO KHOA HỌC LẦN THỨ VI Trường THPT Chuyên Thái Bình 25 HỘI CÁC TRƯỜNG THPT CHUYÊN KHU VỰC DUYÊN HẢI - ĐỒNG BẰNG BẮC BỘ HỘI THẢO KHOA HỌC LẦN THỨ VI Trường THPT Chuyên Thái Bình 26 HỘI CÁC TRƯỜNG THPT CHUYÊN KHU VỰC DUYÊN HẢI - ĐỒNG BẰNG BẮC BỘ HỘI THẢO KHOA HỌC LẦN THỨ VI Trường THPT Chuyên Thái Bình 27 HỘI CÁC TRƯỜNG THPT CHUYÊN KHU VỰC DUYÊN HẢI - ĐỒNG BẰNG BẮC BỘ HỘI THẢO KHOA HỌC LẦN THỨ VI Trường THPT Chuyên Thái Bình 28 HỘI CÁC TRƯỜNG THPT CHUYÊN KHU VỰC DUYÊN HẢI - ĐỒNG BẰNG BẮC BỘ HỘI THẢO KHOA HỌC LẦN THỨ VI Trường THPT Chuyên Thái Bình 29 HỘI CÁC TRƯỜNG THPT CHUYÊN KHU VỰC DUYÊN HẢI - ĐỒNG BẰNG BẮC BỘ HỘI THẢO KHOA HỌC LẦN THỨ VI Trường THPT Chuyên Thái Bình 30 HỘI CÁC TRƯỜNG THPT CHUYÊN KHU VỰC DUYÊN HẢI - ĐỒNG BẰNG BẮC BỘ HỘI THẢO KHOA HỌC LẦN THỨ VI Trường THPT Chuyên Thái Bình 31 HỘI CÁC TRƯỜNG THPT CHUYÊN KHU VỰC DUYÊN HẢI - ĐỒNG BẰNG BẮC BỘ HỘI THẢO KHOA HỌC LẦN THỨ VI Trường THPT Chuyên Thái Bình 32 HỘI CÁC TRƯỜNG THPT CHUYÊN KHU VỰC DUYÊN HẢI - ĐỒNG BẰNG BẮC BỘ HỘI THẢO KHOA HỌC LẦN THỨ VI Trường THPT Chuyên Thái Bình 33 HỘI CÁC TRƯỜNG THPT CHUYÊN KHU VỰC DUYÊN HẢI - ĐỒNG BẰNG BẮC BỘ HỘI THẢO KHOA HỌC LẦN THỨ VI Trường THPT Chuyên Thái Bình 34 HỘI CÁC TRƯỜNG THPT CHUYÊN KHU VỰC DUYÊN HẢI - ĐỒNG BẰNG BẮC BỘ HỘI THẢO KHOA HỌC LẦN THỨ VI Trường THPT Chuyên Thái Bình 35 HỘI CÁC TRƯỜNG THPT CHUYÊN KHU VỰC DUYÊN HẢI - ĐỒNG BẰNG BẮC BỘ HỘI THẢO KHOA HỌC LẦN THỨ VI Trường THPT Chuyên Thái Bình 36 HỘI CÁC TRƯỜNG THPT CHUYÊN KHU VỰC DUYÊN HẢI - ĐỒNG BẰNG BẮC BỘ HỘI THẢO KHOA HỌC LẦN THỨ VI Trường THPT Chuyên Thái Bình 37 HỘI CÁC TRƯỜNG THPT CHUYÊN KHU VỰC DUYÊN HẢI - ĐỒNG BẰNG BẮC BỘ HỘI THẢO KHOA HỌC LẦN THỨ VI Trường THPT Chuyên Thái Bình 38 HỘI CÁC TRƯỜNG THPT CHUYÊN KHU VỰC DUYÊN HẢI - ĐỒNG BẰNG BẮC BỘ HỘI THẢO KHOA HỌC LẦN THỨ VI Trường THPT Chuyên Thái Bình 39 HỘI CÁC TRƯỜNG THPT CHUYÊN KHU VỰC DUYÊN HẢI - ĐỒNG BẰNG BẮC BỘ HỘI THẢO KHOA HỌC LẦN THỨ VI Trường THPT Chuyên Thái Bình 40 HỘI CÁC TRƯỜNG THPT CHUYÊN KHU VỰC DUYÊN HẢI - ĐỒNG BẰNG BẮC BỘ HỘI THẢO KHOA HỌC LẦN THỨ VI Trường THPT Chuyên Thái Bình 41 HỘI CÁC TRƯỜNG THPT CHUYÊN KHU VỰC DUYÊN HẢI - ĐỒNG BẰNG BẮC BỘ HỘI THẢO KHOA HỌC LẦN THỨ VI Trường THPT Chuyên Thái Bình 42 HỘI CÁC TRƯỜNG THPT CHUYÊN KHU VỰC DUYÊN HẢI - ĐỒNG BẰNG BẮC BỘ HỘI THẢO KHOA HỌC LẦN THỨ VI Trường THPT Chuyên Thái Bình 43 HỘI CÁC TRƯỜNG THPT CHUYÊN KHU VỰC DUYÊN HẢI - ĐỒNG BẰNG BẮC BỘ HỘI THẢO KHOA HỌC LẦN THỨ VI Trường THPT Chuyên Thái Bình 44 HỘI CÁC TRƯỜNG THPT CHUYÊN KHU VỰC DUYÊN HẢI - ĐỒNG BẰNG BẮC BỘ HỘI THẢO KHOA HỌC LẦN THỨ VI Trường THPT Chuyên Thái Bình 45 HỘI CÁC TRƯỜNG THPT CHUYÊN KHU VỰC DUYÊN HẢI - ĐỒNG BẰNG BẮC BỘ HỘI THẢO KHOA HỌC LẦN THỨ VI Trường THPT Chuyên Thái Bình 46 HỘI CÁC TRƯỜNG THPT CHUYÊN KHU VỰC DUYÊN HẢI - ĐỒNG BẰNG BẮC BỘ HỘI THẢO KHOA HỌC LẦN THỨ VI Trường THPT Chuyên Thái Bình 47 HỘI CÁC TRƯỜNG THPT CHUYÊN KHU VỰC DUYÊN HẢI - ĐỒNG BẰNG BẮC BỘ HỘI THẢO KHOA HỌC LẦN THỨ VI Trường THPT Chuyên Thái Bình 48 HỘI CÁC TRƯỜNG THPT CHUYÊN KHU VỰC DUYÊN HẢI - ĐỒNG BẰNG BẮC BỘ HỘI THẢO KHOA HỌC LẦN THỨ VI Trường THPT Chuyên Thái Bình 49 HỘI CÁC TRƯỜNG THPT CHUYÊN KHU VỰC DUYÊN HẢI - ĐỒNG BẰNG BẮC BỘ HỘI THẢO KHOA HỌC LẦN THỨ VI Trường THPT Chuyên Thái Bình 50 HỘI CÁC TRƯỜNG THPT CHUYÊN KHU VỰC DUYÊN HẢI - ĐỒNG BẰNG BẮC BỘ HỘI THẢO KHOA HỌC LẦN THỨ VI Trường THPT Chuyên Thái Bình 51 HỘI CÁC TRƯỜNG THPT CHUYÊN KHU VỰC DUYÊN HẢI - ĐỒNG BẰNG BẮC BỘ HỘI THẢO KHOA HỌC LẦN THỨ VI Trường THPT Chuyên Thái Bình 52 HỘI CÁC TRƯỜNG THPT CHUYÊN KHU VỰC DUYÊN HẢI - ĐỒNG BẰNG BẮC BỘ HỘI THẢO KHOA HỌC LẦN THỨ VI Trường THPT Chuyên Thái Bình 53 HỘI CÁC TRƯỜNG THPT CHUYÊN KHU VỰC DUYÊN HẢI - ĐỒNG BẰNG BẮC BỘ HỘI THẢO KHOA HỌC LẦN THỨ VI Trường THPT Chuyên Thái Bình 54 HỘI CÁC TRƯỜNG THPT CHUYÊN KHU VỰC DUYÊN HẢI - ĐỒNG BẰNG BẮC BỘ HỘI THẢO KHOA HỌC LẦN THỨ VI Trường THPT Chuyên Thái Bình 55 HỘI CÁC TRƯỜNG THPT CHUYÊN KHU VỰC DUYÊN HẢI - ĐỒNG BẰNG BẮC BỘ HỘI THẢO KHOA HỌC LẦN THỨ VI Trường THPT Chuyên Thái Bình 56 HỘI CÁC TRƯỜNG THPT CHUYÊN KHU VỰC DUYÊN HẢI - ĐỒNG BẰNG BẮC BỘ HỘI THẢO KHOA HỌC LẦN THỨ VI Trường THPT Chuyên Thái Bình 57 HỘI CÁC TRƯỜNG THPT CHUYÊN KHU VỰC DUYÊN HẢI - ĐỒNG BẰNG BẮC BỘ HỘI THẢO KHOA HỌC LẦN THỨ VI Trường THPT Chuyên Thái Bình 58 HỘI CÁC TRƯỜNG THPT CHUYÊN KHU VỰC DUYÊN HẢI - ĐỒNG BẰNG BẮC BỘ HỘI THẢO KHOA HỌC LẦN THỨ VI Trường THPT Chuyên Thái Bình 59 HỘI CÁC TRƯỜNG THPT CHUYÊN KHU VỰC DUYÊN HẢI - ĐỒNG BẰNG BẮC BỘ HỘI THẢO KHOA HỌC LẦN THỨ VI Trường THPT Chuyên Thái Bình 60 HỘI CÁC TRƯỜNG THPT CHUYÊN KHU VỰC DUYÊN HẢI - ĐỒNG BẰNG BẮC BỘ HỘI THẢO KHOA HỌC LẦN THỨ VI Trường THPT Chuyên Thái Bình 61 HỘI CÁC TRƯỜNG THPT CHUYÊN KHU VỰC DUYÊN HẢI - ĐỒNG BẰNG BẮC BỘ HỘI THẢO KHOA HỌC LẦN THỨ VI Trường THPT Chuyên Thái Bình 62 HỘI CÁC TRƯỜNG THPT CHUYÊN KHU VỰC DUYÊN HẢI - ĐỒNG BẰNG BẮC BỘ HỘI THẢO KHOA HỌC LẦN THỨ VI Trường THPT Chuyên Thái Bình 63 HỘI CÁC TRƯỜNG THPT CHUYÊN KHU VỰC DUYÊN HẢI - ĐỒNG BẰNG BẮC BỘ HỘI THẢO KHOA HỌC LẦN THỨ VI Trường THPT Chuyên Thái Bình 64 HỘI CÁC TRƯỜNG THPT CHUYÊN KHU VỰC DUYÊN HẢI - ĐỒNG BẰNG BẮC BỘ HỘI THẢO KHOA HỌC LẦN THỨ VI Trường THPT Chuyên Thái Bình 65 HỘI CÁC TRƯỜNG THPT CHUYÊN KHU VỰC DUYÊN HẢI - ĐỒNG BẰNG BẮC BỘ HỘI THẢO KHOA HỌC LẦN THỨ VI Trường THPT Chuyên Thái Bình 66 HỘI CÁC TRƯỜNG THPT CHUYÊN KHU VỰC DUYÊN HẢI - ĐỒNG BẰNG BẮC BỘ HỘI THẢO KHOA HỌC LẦN THỨ VI Trường THPT Chuyên Thái Bình 67 HỘI CÁC TRƯỜNG THPT CHUYÊN KHU VỰC DUYÊN HẢI - ĐỒNG BẰNG BẮC BỘ HỘI THẢO KHOA HỌC LẦN THỨ VI Trường THPT Chuyên Thái Bình 68 HỘI CÁC TRƯỜNG THPT CHUYÊN KHU VỰC DUYÊN HẢI - ĐỒNG BẰNG BẮC BỘ HỘI THẢO KHOA HỌC LẦN THỨ VI Trường THPT Chuyên Thái Bình 69 HỘI CÁC TRƯỜNG THPT CHUYÊN KHU VỰC DUYÊN HẢI - ĐỒNG BẰNG BẮC BỘ HỘI THẢO KHOA HỌC LẦN THỨ VI Chuyên đề xếp loại A MỘT SỐ BÀI TOÁN VỀ LƯỚI VÀ BẢNG Nguyễn Hữu Nhân Giáo viên THPT Chuyên Quốc Học – Huế MỞ ĐẦU Trong những năm gần đây, ở các kì thi Olympic Toán quốc gia và quốc tế thường có các bài toán tổ hợp và rời rạc. Trong các bài toán tổ hợp và rời rạc này, có các bài toán liên quan đến đến lưới và bảng. Lớp bài toán này khá phong phú về nội dung và đa dạng về hình thức thể hiện. Trong bài viết này tôi xin giới thiệu một số bài toán liên quan đến vấn đề này. Các ví dụ này đã được trình bày cho học sinh trong các đợt bồi dưỡng của các năm qua. Một số bài toán. Bài toán 1. Cho bảng vuông kích thước ( n 2 + n + 1) × ( n 2 + n + 1) . Mỗi ô vuông của bảng được ghi số 0 hoặc số 1, sao cho không có bốn ô có ghi số 1 nào là đỉnh của một hình chữ nhật. Chứng minh số số 1 không vượt quá ( n + 1) × ( n 2 + n + 1) . Giải Gọi xi là số số 1 ở hàng thứ S= ( i = 1, 2, i n2 + n +1 ∑ i =1 Xét tập M , n 2 + n + 1 ). Ta cần chứng minh xi ≤ (n + 1)(n 2 + n + 1) . mà mỗi phần tử là một cặp (k, l) với 1 ≤ k < l ≤ n2 + n +1. Ta có M = Cn22 +n+1 . Với mỗi i = 1, 2, , n 2 + n + 1 , xét tập M i mà mỗi phần tử là một cặp (k, l) với 1 ≤ k < l ≤ n2 + n +1 và hai cột k và Trường THPT Chuyên Thái Bình l đều có số 1 ở hàng i . Ta có Mi ⊂ M và 70 HỘI CÁC TRƯỜNG THPT CHUYÊN KHU VỰC DUYÊN HẢI - ĐỒNG BẰNG BẮC BỘ HỘI THẢO KHOA HỌC LẦN THỨ VI M i = C x2i = x i ( x i − 1) 2 (chú ý rằng nếu xi < 2 thì C x2i = 0 ). Do không có bốn số 1 nào là đỉnh của một hình chữ nhật nên M i ∩ M j = ∅ nếu i ≠ j . Suy ra n 2 + n +1 ∑ i =1 Mi ≤ M tức là n 2 + n +1 ∑ i =1 xi ( xi − 1) ( n 2 + n )( n 2 + n + 1) ≤ ⇔ 2 2 n 2 + n +1 ∑ i =1 x − 2 i n 2 + n +1 ∑ i =1 xi ≤ ( n 2 + n )( n 2 + n + 1) . Theo bất đẳng thức Cauchy-Schwarz 2 n + n +1  n + n +1  2 ( 1) x n n xi2 ⇒ ≤ + +  ∑ i ∑ i =1  i =1  2 2 n 2 + n +1 ∑ i =1 2  n + n +1  1 x ≥ 2  ∑ xi  . n + n + 1  i =1  2 2 i Suy ra 2  n + n +1  n + n +1 1 2 2  ∑ xi  − ∑ xi ≤ ( n + n )( n + n + 1) . 2 n + n + 1  i =1 i =1  2 2 Hay S 2 − ( n 2 + n + 1) S − ( n 2 + n )( n 2 + n + 1) 2 ≤ 0 . Từ đó S ≤ ( n + 1)( n 2 + n + 1) . Bài toán 2. Mỗi ô của bảng vuông kích thước n × n được ghi 0 hoặc số 1, sao cho với mỗi ô ghi số 0 thì có ít nhất n ô cùng hàng hoặc cùng cột với nó được ghi số Trường THPT Chuyên Thái Bình 71 HỘI CÁC TRƯỜNG THPT CHUYÊN KHU VỰC DUYÊN HẢI - ĐỒNG BẰNG BẮC BỘ HỘI THẢO KHOA HỌC LẦN THỨ VI n2  1. Chứng minh rằng có ít nhất   số 1 được ghi. 2 Giải Với n = 4 ta có một cách ghi như sau, với số số 1 được ghi là 8 : 0 1 1 0 1 0 0 1 0 1 1 0 1 0 0 1 Trong trường hợp tổng quát, xây dượng đồ thị hai phía gồm 2 n đỉnh, mà n đỉnh bên trái là n hàng và n đỉnh bên phải là n cột của bảng. Đỉnh H i được nối với đỉnh Cj nếu ô (i, j) được ghi số 1. Theo giả thiết nếu đỉnh H i không nối với đỉnh Cj thì d ( H i ) + d (C j ) ≥ n trong đó d ( H i ) là số số 1 ở hàng i , d(Ci ) là số số 1 ở hàng j . n2 Ta chứng minh số cạnh của đồ thị là e ≥ . 2 Thật vậy, ta có S = ∑ (i, j )= 0  d ( H i ) + d ( C j )  ≥ ( n 2 − e ) n . Trong tổng trên với mỗi i , số hạng d ( H i ) xuất hiện n − d ( H i ) lần; với mỗi j , số hạng d(Ci ) xuất hiện n − d ( C j ) lần. Mà Trường THPT Chuyên Thái Bình 72 HỘI CÁC TRƯỜNG THPT CHUYÊN KHU VỰC DUYÊN HẢI - ĐỒNG BẰNG BẮC BỘ HỘI THẢO KHOA HỌC LẦN THỨ VI n n i =1 j =1 ∑ d ( H i ) = ∑ d (C j ) = e . Suy ra n n n n i =1 j =1 i =1 j =1 S = ∑ d ( H i ) [ n − d ( H i ) ] + ∑ d (C j )  n − d (C j )  = 2ne − ∑ d 2 ( H i ) − ∑ d 2 (C j ) . Theo bất đẳng thức Schwarz 2 1 n e2  ( ) ( ) d H d H ≥ = ∑ ∑ i  n , i n  i =1 i =1 n 2 n ∑ d 2 (C j ) ≥ j =1 e2 . n Suy ra (n2 − e)n ≤ 2ne − 2 e2 n2 ⇔ 2e2 − 3n2e + n4 ≤ 0 ⇔ ≤ e ≤ n2 (đpcm). 2 n Bài toán 3. Cho bảng vuông kích thước 2 0 1 2 × 2 0 1 2 . Người ta ghi vào mỗi ô (i, j) ( i, j = 1, 2, , 2012 ) một số tự nhiên aij thỏa các điều kiện : (1) ai1 + ai 2 + + ai 2012 = 2011 , với i = 1, 2, , 2012; (2) nếu aij akl > 0 thì (k − i)(l − j) ≥ 0. Hỏi có bao nhiêu cách ghi như vậy ? Giải Theo giả thiết (1), mỗi ô được ghi số nguyên dương và tổng các số trên mỗi hàng bằng 2011. Theo giả thiết (2), nếu a ij > 0 và a kl > 0 thì k ≥ i và l ≥ j ; nghĩa là từ một ô có ghi số dương chỉ có thể đến một ô có ghi số dương ở hàng lớn hơn hoặc cột lớn hơn. Trường THPT Chuyên Thái Bình 73 HỘI CÁC TRƯỜNG THPT CHUYÊN KHU VỰC DUYÊN HẢI - ĐỒNG BẰNG BẮC BỘ HỘI THẢO KHOA HỌC LẦN THỨ VI Từ đó có thể xây dựng bảng như sau: • Xuất phát từ ô (1,1) để đến ô (2012,2012) bằng cách sang phải hoặc xuống dưới. • Tại mỗi ô có thể đặt một viên sỏi hoặc không. • Nếu đã đặt đủ 2011 viên sỏi trên một hàng thì xuống dưới, nếu chưa thì có thể đặt một viên sỏi hoặc sang phải. Sau khi hoàn tất, số sỏi trong mỗi ô là số cần ghi vào ô đó. Ô không có sỏi ghi số 0. Như vậy có 2011 thao tác “đặt viên sỏi” trên mỗi hàng. Do có 2012 hàng nên số thao tác “đặt viên sỏi” là 2011× 2012 = 4 046132 . Ngoài ra có 2011 thao tác “sang phải”. Vì vậy tổng cộng có 4 046132 + 2011 = 4 048143 . Chú ý rằng thao tác “xuống dưới” không được tính do được tính theo thao tác “đặt viên sỏi”. Suy ra số cách thành lập bảng là số cách bố trí 2011 thao tác “sang phải” trong 2011 dãy 4 048143 thao tác nói trên. Vậy số cách thành lập bảng là C4048143 . Bài toán 4. Tìm số đường đi dọc theo cạnh lưới ô vuông từ đỉnh (0, 0) đến đỉnh (n, n), sao cho không vượt qua đường chéo chính y = x và mỗi bước đi là sang phải hoặc lên trên. (n,n) (0,0) Trường THPT Chuyên Thái Bình 74 HỘI CÁC TRƯỜNG THPT CHUYÊN KHU VỰC DUYÊN HẢI - ĐỒNG BẰNG BẮC BỘ HỘI THẢO KHOA HỌC LẦN THỨ VI Giải Ta gọi một đường đi từ đỉnh (0, 0) đến đỉnh (n, n) theo hướng sang phải hoặc đi lên là đường đi tiến. y = x+1 (n,n) A (-1,1) (0,0) Mỗi đường đi tiến gồm 2 n bước, với n bước sang phải và n bước lên trên. Như vậy mỗi đường đi tiến là một cách chọn n bước sang phải trong số 2 n bước. Do đó số đường đi tiến là C2nn . Ta lại gọi một đường đi tiến không vượt qua đường chéo chính là một đường đi tốt, ngược lại là một đường đi không tốt. Ta sẽ tìm số đường đi không tốt. Cho P là một đường đi không tốt. Khi đó P sẽ gặp đường thẳng y = x + 1 lần đầu tiên tại một điểm A . Lấy đối xứng đoạn đường của P từ điểm O đến điểm A qua đường thẳng y = x + 1 ta được một đoạn đường đi từ điểm ( −1, 1) đến điểm A . Đoạn đường này cũng là một đường đi tiến. Kết hợp đoạn đường này với phần còn lại của từ điểm A đến điểm (n, n) ta được một đường đi tiến từ điểm ( −1, 1) đến điểm (n, n). Ngược lại, cho Q là một đường đi tiến từ điểm ( −1, 1) đến điểm (n, n). Khi đó sẽ gặp đường thẳng y = x + 1 lần đầu tiên tại một điểm của Q từ điểm ( −1, 1) đến điểm A qua đường thẳng y = A. x + P Q Lấy đối xứng đoạn đường 1 ta được một đoạn đường đi từ điểm O đến điểm A . Đoạn đường này cũng là một đường đi tiến. Kết hợp đoạn đường này với phần còn lại của Q từ điểm A đến điểm (n, n) ta được một đường đi tiến từ điểm O đến điểm (n, n). Đường đi này là một đường đi không tốt. Trường THPT Chuyên Thái Bình 75 HỘI CÁC TRƯỜNG THPT CHUYÊN KHU VỰC DUYÊN HẢI - ĐỒNG BẰNG BẮC BỘ HỘI THẢO KHOA HỌC LẦN THỨ VI Như vậy số đường đi không tốt từ điểm O đến điểm (n, n) đúng bằng số đường đi tiến từ điểm ( −1, 1) đến điểm (n, n). Số đường đi này bằng C 2nn−1 . Suy ra số đường đi tốt từ điểm O đến điểm (n, n) là 1 C 2n n n +1 C 2n n − C 2n n− 1 = . Bài toán 5. Một con xe được đặt trên bàn cờ kích thước 3 × n , với n ∈ * . Con xe đi từ vị trí (1, 1) đến vị trí (3,1) bằng một đường đi không tự cắt. Hỏi có bao nhiêu đường đi như thế trên bàn cờ ? Giải Gọi số đường đi là rn. Có 6 cách đi như các hình vẽ sau : 1) Đường đi qua các ô (1,1), (2,1), (3,1) . Có 1 đường đi loại này. 2) Đường đi không qua ô (2,1) . Mỗi đường đi loại này bắt đầu là (1,1) →(1,2) và kết thúc là (3, 2) → (3,1) . Có rn−1 đường đi loại này. 3) Đường đi bắt đầu là (1,1) → (2,1) → (2,2) và không trở lại hàng 1. Mỗi đường đi như vậy đến hàng 3 từ ô (2,k), với 2 ≤ k ≤ n và di dọc theo hàng 3 đến ô (3,1) . Có n − 1 đường đi loại này. 4) Đường đi bắt đầu là (1,1) → (2,1) → (3, k + 1) → (3, k ) → → (3,1) , với 2 ≤ loại này. k → (2, k ) → (1, k ) → (1, k + 1) , kết thúc là + r1 đường đi ≤ n − 1 . Có rn − 2 + rn − 3 + 5) Đường đi bắt đầu là (1,1) →(1,2) và kết thúc là (2,1) →(3,1) . Có Trường THPT Chuyên Thái Bình 76 n − 1 đường đi loại HỘI CÁC TRƯỜNG THPT CHUYÊN KHU VỰC DUYÊN HẢI - ĐỒNG BẰNG BẮC BỘ HỘI THẢO KHOA HỌC LẦN THỨ VI này. 6) Đường đi là (1,1) → (1,2) → → (1, k ) → (1, k + 1) → (2, k + 1) → (3, k + 1) → (3, k ) → (2, k ) → → (2,1) → (3,1) . Có rn −3 + rn − 2 + + r1 đường đi loại này. Vậy rn = 1 + rn −1 + 2( n − 1) + 2( rn − 2 + rn −3 + + r1 ) = 2 n − 1 + rn −1 + 2( rn − 2 + rn −3 + + r1 ) . Suy ra rn +1 = 2 n + 1 + rn + 2( rn −1 + rn − 2 + + r1 ) . Do đó rn +1 − rn = 2 + rn + rn −1 ⇒ rn +1 = 2 + 2 rn + rn −1 ⇒ rn +1 + 1 = 2( rn + 1) + rn −1 + 1 . Dễ thấy r1 = 1, r2 =4. Sử dụng phương trình đặc trưng tìm được rn = ( 1  1+ 2 2 2  ) n +1 ( − 1− 2 ) n +1  −1.  Bài toán 7. Cho bàn cờ kích thước 2 0 1 1 × 2 0 1 2 . Bỏ bớt hai ô khác màu tùy ý. Hãy xếp đầy bàn cờ còn lại bằng các đôminô kích thước 1 × 2 , sao cho các đôminô đó không chờm lên nhau (có thể xoay các đôminô). Giải lẻ, n Ta giải bài toán trong trường hợp tổng quát, bàn cờ có kích thước chẵn. m ×n với m Xây dựng một chu trình Hamilton đi qua tất cả các ô, mỗi ô một lần. Do tổng số ô là chẵn nên điều này luôn tìm được. Một chu trình Hamilton được chỉ ra như hình vẽ. Có hai trường hợp • Nếu hai ô bỏ đi là kề nhau trên chu trình Hamilton thì xếp các đôminô liên tiếp trên phần còn lại. Trường THPT Chuyên Thái Bình 77 HỘI CÁC TRƯỜNG THPT CHUYÊN KHU VỰC DUYÊN HẢI - ĐỒNG BẰNG BẮC BỘ HỘI THẢO KHOA HỌC LẦN THỨ VI • Nếu hai ô bỏ đi không kề nhau thì các ô này chia chu trình Hamilton thành hai phần, mỗi phần có một số chẵn ô. Ta xếp các đôminô liên tiếp trên mỗi phần đó. Vậy luôn xếp đầy bàn cờ còn lại bằng các đôminô. Bài toán 6. Cho các quân triminô hình chữ L gồm 3 hình vuông đơn vị như hình vẽ sau Phủ hình vuông kích thước 5 × 5 bằng các quân triminô hình chữ L này, sao cho chúng không chờm lên nhau, thì còn thừa một ô vuông đơn vị không được phủ (có thể xoay các triminô). Hỏi ô không được phủ có thể nằm ở vị trí nào trên hình vuông đã cho ? Giải Tô màu các ô như hình vẽ. Trường THPT Chuyên Thái Bình 78 HỘI CÁC TRƯỜNG THPT CHUYÊN KHU VỰC DUYÊN HẢI - ĐỒNG BẰNG BẮC BỘ HỘI THẢO KHOA HỌC LẦN THỨ VI Nếu ô có màu trắng không được phủ thì cả 9 ô đen đều được phủ. Mà mỗi triminô chỉ phủ đúng một ô đen. Suy ra có ít nhất 9 triminô được dùng. Khi đó số ô ít nhất được phủ là 9 × 3 = 2 7 > 2 5 . Vô lí. Vậy không được phủ phải là ô đen. Do tính đối xứng nên chỉ xét ba trường hợp: ô đen không được phủ là ô trung tâm, ô cạnh, ô góc. Ba trường hợp này có cách phủ như sau : Bài toán 7. Mỗi ô vuông đơn vị của bảng vuông kích thước n × n được tô bởi màu đen hoặc màu trắng. Giả sử tất cả các cách tô màu của hình vuông kích thước 2 × 2 đều có mặt trong bảng. a) Hãy tìm giá trị nhỏ nhất của n . b) Với n tìm được hãy tìm một cách tô mà số ô đen là ít nhất. Giải a) Mỗi ô vuông đơn vị của bảng có hai cách tô màu. Suy ra mỗi hình vuông kích thước 2 × 2 có 24 = 16 cách tô màu. Mặt khác với hình vuông kích thước kích thước 2 × 2 khác nhau. n × n có thể chọn ra ( n − 1) 2 hình vuông Do đó ( n − 1) 2 ≥ 16 ⇔ n ≥ 5 . Với n = 5 ta có một cách tô màu như sau thỏa điều kiện bài toán : Trường THPT Chuyên Thái Bình 79 HỘI CÁC TRƯỜNG THPT CHUYÊN KHU VỰC DUYÊN HẢI - ĐỒNG BẰNG BẮC BỘ HỘI THẢO KHOA HỌC LẦN THỨ VI Vậy giá trị nhỏ nhất là n = 5 . b) Hình vuông kích thước 5 × 5 có 4 hình vuông đơn vị ở góc, 12 hình vuông đơn vị ở cạnh và 9 hình vuông đơn vị ở trong. Mỗi hình vuông đơn vị ở góc chỉ thuộc đúng 1 hình vuông kích thước 2 × 2 , mỗi hình vuông đơn vị ở cạnh thuộc 2 hình vuông kích thước 2 × 2 , mỗi hình vuông đơn vị ở trong thuộc 4 hình vuông kích thước 2 × 2 . 5 × 5 Chú ý rằng 16 hình vuông kích thước 2 × 2 có được từ hình vuông kích thước gồm tổng cộng 64 hình vuông đơn vị, trong đó có 32 ô đen. Gọi k là số ô trắng trên hình vuông kích thước cạnh và c ô trong. Ta chứng minh k = 1 0 . Giả sử k < 1 0 5 × 5 , trong đó có a ô góc, b ô . Ta có a+b+c = k (1) a + 2b + 4c = 32 (2) Từ (2) ta có + Nếu c = 8 4 c ≤ 3 2 ⇔ thì a = b = 0 c ≤ 8 . . Khi đó có một trong các hình vẽ sau (các hình đối xứng) Trường THPT Chuyên Thái Bình 80 HỘI CÁC TRƯỜNG THPT CHUYÊN KHU VỰC DUYÊN HẢI - ĐỒNG BẰNG BẮC BỘ HỘI THẢO KHOA HỌC LẦN THỨ VI (a) (b) (c) • Trong hình (a) thiếu một trong các hình vuông kích thước trắng và 1 ô đen. • Trong hình (a) và (b) thiếu hình vuông kích thước 2 × 2 2 × 2 gồm 3 ô gồm 4 ô trắng. Vậy trường hợp này không thể xảy ra. + Nếu c = 7 thì a = 0, b = 2 . Trường hợp này có ít nhất hai cạnh của hình vuông kích thước 5 × 5 không có ô trắng. Có thể xem một trong hai cạnh này là cạnh trên của hình vuông đó. Chú ý rằng mỗi cách tô màu hình vuông kích thước 2 × 2 chỉ xuất hiện đúng một lần trong hình vuông kích thước 5 × 5 . Trong 16 cách tô màu hình vuông kích thước 2 × 2 chỉ có 4 cách mà 2 ô ở hàng trên có màu trắng. Suy ra 4 hình vuông kích thước 2 × 2 tương ứng phải nằm ở hai hàng đầu của hình vuông kích thước 5 × 5 . Trong số này có 1 hình vuông kích thước 2 × 2 gồm 4 ô trắng. Lý luận tương tự với cạnh còn lại gồm 5 ô trắng. Suy ra hình vuông kích thước 2 × 2 gồm 4 ô trắng phải ở một góc của hình vuông kích thước 5 × 5 . Giả sử đó là góc trái trên. Khi đó 2 ô bên cạnh hình vuông này là hai ô đen. Như vậy có 2 hình vuông có dạng giống nhau (gồm 3 ô trắng và 1 ô đen ở góc phải dưới). Trường THPT Chuyên Thái Bình 81 HỘI CÁC TRƯỜNG THPT CHUYÊN KHU VỰC DUYÊN HẢI - ĐỒNG BẰNG BẮC BỘ HỘI THẢO KHOA HỌC LẦN THỨ VI Vậy trường hợp này không thể xảy ra. + Nếu c ≤ Có ba khả năng 6 thì ta luôn có • a = 0, b = 4 ⇒ a + b + c = 10 • a = 2, b = 3 ⇒ a + b + c = 11 • a = 4, b = 2 ⇒ a + b + c = 12 k = a + b + c ≥ 1 0 . Chẳng hạn với c = 6 thì a + 2 b = 8 . Như vậy trường hợp này cũng không thể xảy ra. Vậy giá trị nhỏ nhất của số ô đen là hình vẽ đầu tiên. k = 1 0 . Một cách tô màu được cho trong Bài toán 9. Một con tốt được đặt trên một ô của bảng vuông kích thước n × n , với n ≥ 2 . Con tốt đó có thể đi từ một ô sang 8 ô xung quanh, sao cho hai bước đi liên tiếp phải khác kiểu (chéo, ngang hoặc dọc). Xác định các giá trị của n sao cho có thể chọn một ô xuất phát và một dãy các bước đi để con tốt có thể đi khắp bảng, mỗi ô qua đúng một lần. Giải Xét hai trường hợp • n = 2 k : Chia bàn cờ thành các hình vuông kích thước 2 × 2 . Ban đầu đặt con tốt ở vị trí (1, 1) của bàn cờ. Di chuyển như hình vẽ thì con tốt sẽ đi khắp bàn cờ, mỗi ô qua đúng một lần và hai bước đi liên tiếp là khác kiểu. Trường THPT Chuyên Thái Bình 82 HỘI CÁC TRƯỜNG THPT CHUYÊN KHU VỰC DUYÊN HẢI - ĐỒNG BẰNG BẮC BỘ HỘI THẢO KHOA HỌC LẦN THỨ VI 2 • n = 2 k + 1 : Tổng số bước đi là n −1. Tô màu đen các hàng chẵn. Sô ô đen là n2 − n . Khi đó mỗi bước đi theo kiểu chéo thì con tốt phải đi qua hai ô khác màu. 2 Do con tốt không đi qua ô nào quá một lần và không có hai bước đi liên tiếp theo kiểu chéo, nên mỗi bước đi theo kiểu chéo chỉ qua đúng một ô đen. Suy ra số bước đi theo kiểu chéo nhiều nhất là nhiếu nhất là n ≥ 3 n2 − n và số bước đi theo kiểu ngang hoặc dọc 2 n2 − n + 1 . Suy ra tổng số bước đi nhiều nhất là n2 − n +1. Với 2 n lẻ và thì n2 − n + 1 < n2 −1. Do đó con tốt con tốt không thể đi khắp bàn cờ trong trương hợp này. Vậy n = 2 k , với k ∈ * . Bài toán 10. Một hình chữ nhật kích thước 2 0 1 3 × 2 0 1 2 được tô toàn bộ bởi bốn màu xanh, trắng, vàng, đỏ theo quy tắc (i) Mỗi ô tô đúng một màu (ii) Các màu xanh, trắng, đỏ, vàng lần lượt được tô cho các mảng có dạng (1) xanh (2) trắng (3) đỏ (4) vàng Sau khi tô xong một người đếm được 2.013.021 ô xanh, 1.113.006 ô trắng và tiếp tục đếm các ô màu còn lại. Hỏi kết quả đếm được là đúng hay sai ? Trường THPT Chuyên Thái Bình 83 HỘI CÁC TRƯỜNG THPT CHUYÊN KHU VỰC DUYÊN HẢI - ĐỒNG BẰNG BẮC BỘ HỘI THẢO KHOA HỌC LẦN THỨ VI Giải Đánh số các ô theo quy tắc: ô (i, j) được đánh số (i + j)mod3 . 2 0 1 2 0 1 0 1 2 0 1 2 1 2 0 1 2 0 2 0 1 2 0 1 0 1 2 0 1 2 1 2 0 1 2 0 Khi đó + Hình (1) chiếm các ô mà tổng các số trong các ô đó chia 3 dư 2. + Hình (2) chiếm các ô mà tổng các số trong các ô đó chia 3 dư 1. + Hình (3) chiếm các ô mà tổng các số trong các ô đó chia 3 dư 0. + Hình (4) chiếm các ô mà tổng các số trong các ô đó chia 3 dư 0. Giả sử sau khi tô xong, có N1 mảng hình (1), N 2 mảng hình (2), N 3 mảng hình (3), N 4 mảng hình (4). Tổng các số ghi trong hình chữ nhật là 3k + 2 N1 + N 2 . Do tổng các số trong hình chữ nhật chia hết cho 3 nên 2N1 + N2 chia hết cho 3. Theo giả thiết 3N1 = 2.013.021, 3N2 = 1.113.006 . Suy ra N1 = 671.007 , N 2 = 371.002 . Do đó 2 N1 + N 2 = 1.713.016 . Số này không chia hết cho 3. Vậy kết quả đếm được là sai. Bài toán 10. Cho tam giác đều cạnh n được chia thành các tam giác đều cạnh 1. a) Hỏi có bao nhiêu hình hình bình hành được tạo thành ? Trường THPT Chuyên Thái Bình 84 HỘI CÁC TRƯỜNG THPT CHUYÊN KHU VỰC DUYÊN HẢI - ĐỒNG BẰNG BẮC BỘ HỘI THẢO KHOA HỌC LẦN THỨ VI b) Người ta phủ kín tam giác đều đã cho bằng các hình được ghép bởi 6 tam giác đều cạnh 1 có dạng như sau sao cho các hình này không chờm lên nhau (có thể xoay hoặc lật các hình này). Hãy xác định các giá trị của n để có thể thực hiện được điều đó. Giải a) Gọi tam giác đã cho là A B C . Chia tập các hình bình hành tạo thành thành ba tập con A C B M • và • và • A C B A N TBC là tập các hình bình hành có hai cạnh song song (hoặc ở trên) hai cạnh . TCA là tập các hình bình hành có hai cạnh song song (hoặc ở trên) hai cạnh A B B C . TAB là tập các hình bình hành có hai cạnh song song (hoặc ở trên) hai cạnh C A và C B Do tính đối xứng của tam giác đều nên TBC = TCA = TAB . Suy ra số hình bình hành tạo thành là S = 3 TBC . Kéo dài AB, AC thêm các đoạn B M = C N = 1 . Cho ( H ) là một hình bình hành trong tập TBC . Kéo dài 4 cạnh của ( H ) cắt đoạn thẳng M N tại 4 điểm phân biệt. Ngược lại với 4 điểm phân biệt có tọa độ nguyên trên đoạn thẳng M N ta xây dựng được duy nhất một hình bình hành thuộc tập TBC . Như vậy số hình bình hành thuộc Trường THPT Chuyên Thái Bình 85 HỘI CÁC TRƯỜNG THPT CHUYÊN KHU VỰC DUYÊN HẢI - ĐỒNG BẰNG BẮC BỘ HỘI THẢO KHOA HỌC LẦN THỨ VI TBC là số cách chọn 4 điểm có tọa độ nguyên trên đoạn thẳng thẳng M N có n + 1 M N . Mà trên đoạn điểm như vậy nên TBC = C n4+1 . Vậy S = 3Cn4+1 . b) Tam giác đều cạnh n 2 được chia thành n tam giác đều cạnh 1. Mỗi hình đã 2 cho phủ được 6 tam giác đều cạnh 1. Suy ra n chia hết cho 6, do đó n cũng chia hết cho 6. Xét hai trường hợp • n = 1 2 k : Chia tam giác đều cạnh thành k2tam giác đều cạnh 12. Mỗi tam đều n cạnh 12 được phủ bởi các hình đã cho theo cách như hình vẽ sau • n = 1 2 k + 6 : Tô màu tam giác đều cạnh giác đều cạnh 1 ở biên đều được tô màu đen. Tổng số tam giác đen bằng 1 + 2 + n + n = theo kiểu bàn cờ, sao cho các tam n ( n + 1) = ( 6 k + 3 ) (1 2 k + 7 ) 2 , là một số lẻ. Mỗi hình đã cho phủ được 2 hoặc 4 tam giác đen, nên số tam giác đen được phủ là một số chẵn. Điều này chứng tỏ trường hợp này không thể phủ được tam giác đều cạnh n bằng các hình đã cho. Vậy để phủ tam giác đều cạnh Trường THPT Chuyên Thái Bình n bằng các hình đã cho thì 86 n = 1 2 k , với k ∈ * . HỘI CÁC TRƯỜNG THPT CHUYÊN KHU VỰC DUYÊN HẢI - ĐỒNG BẰNG BẮC BỘ HỘI THẢO KHOA HỌC LẦN THỨ VI Các bài toán tương tự 1. Tìm số đường đi từ điểm (0, 0) đến điểm ( m , n ) trên mặt phẳng tọa độ, dọc theo lưới ô vuông theo hướng đi lên hoặc sang phải. 2. Tìm số đường đi từ điểm (0, 0) đến điểm (2n, 0) trên mặt phẳng tọa độ theo các véc tơ a = (1, 1) , b = (1, − 1) và không vượt qua trục hoành. 3. Ta nói một hình chữ nhật kích thước nguyên là tách được nếu nó có thể chia thành hai hay nhiều hơn các hình vuông có cạnh song song hoặc nằm trên cạnh hình chữa nhật đã cho, với độ dài nguyên và có duy nhất một hình vuông có cạnh nhỏ nhất. Tìm kích thước của hình chữ nhật để nó có thể tách được. 4. Cho phủ được hình H T là triminô hình chữ L. Chứng minh rằng với mọi n∈ đồng dạng với hình H theo hệ số n , sao cho các hình H * đều có thể không chờm lên nhau (có thể xoay hoặc lật các hình H). 5. Cho bảng chữ nhật kích thước H 1 0 × 1 1 . Ta gọi một minô hình chữ thập là hình gồm 5 hình vuông đơn vị như hình vẽ sau a) Hãy chỉ ra một cách xếp 15 hình H vào bảng, sao cho các hình này không chờm lên nhau. b) Chứng minh rằng không thể xếp được 16 hình H vào bàn cờ, sao cho các hình này không chờm lên nhau. 6. Ta gọi n − boomerang là hình hợp bởi L. Tìm tất cả các số nguyên n ≥ 2 2n − 1 hình vuông đơn vị có dạng chữ sao cho tồn tại một hình chữ nhật có kích thước nguyên có thể phủ kín bằng các n − boomerang mà các n − boomerang này không thể chờm lên nhau (có thể xoay các n − boomerang ). 7. Cho bảng hình chữ nhật n × 2 0 0 7 , gồm n hàng và 2007 cột. Tìm n nhỏ nhất sao cho với bất kỳ cách viết vào tất cả ô của bảng mỗi ô một số thực, mà trên mỗi hàng có ít nhất 2 ô được viết là số hữu tỷ, thì tìm được dãy ô các số hữu tỷ a1 , a 2 , , a2 m ( m ≥ 2 ) thỏa mãn a1, a2 ở trên cùng một hàng và a2, a3 ở trên cùng một cột, …, a2m−1, a2m ở trên cùng một hàng và a 2 m , a1 ở trên cùng một cột. Trường THPT Chuyên Thái Bình 87 HỘI CÁC TRƯỜNG THPT CHUYÊN KHU VỰC DUYÊN HẢI - ĐỒNG BẰNG BẮC BỘ HỘI THẢO KHOA HỌC LẦN THỨ VI TÀI LIỆU THAM KHẢO [1] Đề thi Olympic Toán Việt Nam, các nước và quốc tế. [2] Các trang web : Diendantoanhoc.net, Artofproblemsolving, Kalva, Animath. Mathscope.org, Mathlinks.ro.uk, [3] Các tạp chí : Toán học và Tuổi trẻ, Mathematical Excalibur, Mathematical Reflections, Krux Math, Sigma, Kvant, Komal. [4] A path to combinatorics for undergraduates Counting Strategies – Titu Andreescu & Zuming Feng (Birkhauser Boston, 2004) [5] Principles and Techniques in Combinatorics – Chen Chuan-Chong & Koh Khee-Meng (World Scientific, 1999). Trường THPT Chuyên Thái Bình 88 HỘI CÁC TRƯỜNG THPT CHUYÊN KHU VỰC DUYÊN HẢI - ĐỒNG BẰNG BẮC BỘ HỘI THẢO KHOA HỌC LẦN THỨ VI Chuyên đề xếp loại A ĐỊNH LÝ HELLY Lê Đức Thịnh THPT Chuyên Trần Phú – Hải Phòng MỞ ĐẦU Tổ hợp là chủ đề khó trong chương trình trung học phổ thông chuyên. Trong các kì thi học sinh giỏi Toán các cấp, Tổ hợp thường chiếm tới 20 – 30% tổng số bài. Tuy nhiên, học sinh Việt Nam nói chung còn tương đối yếu về mảng toán này. Nguyên nhân chính là các bài toán này thường không yêu cầu nhiều kiến thức nhưng mỗi bài toán lại đòi hỏi những suy luận, sáng tạo riêng để giải quyết. Trong các dạng toán Tổ hợp, mảng kiến thức về hình lồi, bao lồi có rất nhiều bài toán khó. Các bài toán này đòi hỏi sự vận dụng linh hoạt các khái niệm, kiến thức liên quan. Để việc giảng dạy, học tập mảng kiến thức này có hiệu quả, điều rất quan trọng là giáo viên cần phải hệ thống được các nội dung lý thuyết và xây dựng được một nhóm các bài tập điển hình. Một mảng nội dung quan trọng của phần này là hệ thống bài tập liên quan đến định lý Helly. Trong đề tài này, tôi sẽ xây dựng nội dung lý thuyết và đưa ra một hệ thống bài tập điển hình về định lý Helly. Hệ thống bài tập bắt đầu từ các bài đơn giản đến các bài phức tạp hơn, các bài có mối liên quan, móc xích với nhau. Điều này tạo điều kiện cho học sinh có thể dễ dàng hơn trong việc tiếp thu, kích thích tốt hơn khả năng giải quyết vấn đề, thậm chí còn có thể nâng cao hơn khả năng sáng tạo các bài toán. Trường THPT Chuyên Thái Bình 89 HỘI CÁC TRƯỜNG THPT CHUYÊN KHU VỰC DUYÊN HẢI - ĐỒNG BẰNG BẮC BỘ HỘI THẢO KHOA HỌC LẦN THỨ VI 1. Định lý Radon: Cho A = { X 1 ; X 2 ;… ; X d + 2 } là d + 2 điểm trong không gian d . Khi đó luôn tồn tại cách chia A = A1 ∪ A2 sao cho A1 , A2 ≠ ∅ , A1 ∩ A2 = ∅ thỏa mãn bao lồi của A và bao lồi của A2 có giao khác ∅ . 1 Chứng minh: Gọi tọa độ của x k = ( x1k ; x2k ;…; xdk ) ⊂ Xét hệ d + 1 điểm này lần lượt là x1 , x 2 ,…, x d + 2 trong đó d + 2 d . phương trình Ở đây bên trên có d d + 2 d+2 i ∑ai x = 0  ẩn a1 , a 2 , … , a d + 2 : di=+12  a =0 ∑i   i=1 phương trình. Hệ có nghiệm khác ( 0;0;… ;0 ) . Chia a1 , a 2 , … , a d + 2 thành 2 nhóm I1 = {ai | ai > 0} , I 2 = {ai | ai ≤ 0} .  ∑ ai x i = − ∑ a j x j j∈ I 2 Ta có  i∈ I1 ⇒ = − = > 0 a a S ∑ j ∑ i j∈ I 2  i∈ I1 ∑ i∈ I 1 ai i x = S ∑ j∈ I 2 −a j S xj = X . Khi đó X sẽ thuộc vào bao lồi của cả hai tập A và A2. 1 Cách chứng minh trên đây mang một dáng vẻ "không tự nhiên" trong việc tạo ra hệ phương trình trên, ngoài ra còn có vẻ "khô khan" và "cứng nhắc" vì đã biến một khẳng định có tính hình học thành một chứng minh đại số. Tuy nhiên đây cũng là điều khá dễ hiểu vì đó là chứng minh trong trường hợp tổng quát – không gian d . Để dễ hình dung hơn chứng minh của định lý, dưới đây tôi xin trình bày cách mô tả hình học trong trường hợp hay gặp nhất – trường hợp trong mặt phẳng. Trường THPT Chuyên Thái Bình 90 HỘI CÁC TRƯỜNG THPT CHUYÊN KHU VỰC DUYÊN HẢI - ĐỒNG BẰNG BẮC BỘ HỘI THẢO KHOA HỌC LẦN THỨ VI Chứng minh định lý Radon cho d = 2 : Xét 4 điểm trong mặt phẳng. Có thể xảy ra 3 trường hợp: Trường hợp 1: A, B, C, D tạo thành tứ giác lồi. Trong trường hợp này chỉ cần chia 4 điểm A, B, C, D thành 2 nhóm A, C và B, D với điểm chung của hai bao lồi là giao điểm của 2 đường chéo AC, BD. Trường hợp 2: A, B, C, D tạo thành tam giác A B C với điểm D nằm trong hoặc trên các cạnh của tam giác. Trong trường hợp này chỉ cần chia 4 điểm A, B, C, D thành 2 nhóm A, B, C và D với điểm chung của hai bao lồi chính là điểm D . Trường hợp 3: A, B, C, D thẳng hàng theo thứ tự đó. Trong trường hợp này chỉ cần chia 4 điểm A, B, C, D thành 2 nhóm A, D và B, C với điểm chung của hai bao lồi là toàn bộ đoạn B C . 2. Định lý Helly: Cho d hình lồi trong không gian n ≥ d + 1 đó có giao khác ∅ . Khi đó n . Biết d + 1 hình bất kỳ trong hình này có giao khác ∅ . Chứng minh: Ta chứng minh với trường hợp Giả sử có d + 2 n = d tập F1 , F2 , … , Fd + 2 và Do đó ∃ x i ∉ F i , x i ∈ Fj trước. + 2 d + 2 ∀ i = 1, d + 2 tập này không có điểm chung. . j≠i Theo Radon có thể chia x1 , x 2 , … , x d + 2 vào 2 tập A, B sao cho bao lồi của chúng cắt nhau tại P nào đó. Ta sẽ chứng minh P chính là điểm cần tìm.  P ∈ Fi d +2  xi ∉A ⇒ P ∈ Fi Thật vậy  i =1  P ∈ Fi xi ∉B  Bây giờ ta chứng minh cho trường hợp Trường THPT Chuyên Thái Bình 91 n > d + 2 bằng quy nạp. HỘI CÁC TRƯỜNG THPT CHUYÊN KHU VỰC DUYÊN HẢI - ĐỒNG BẰNG BẮC BỘ HỘI THẢO KHOA HỌC LẦN THỨ VI Giả sử ta đã có khẳng định trong trường hợp khẳng định trong trường hợp Xét k + 1 Gi = Fi ∩ Fk +1 ∀i = 1, k . Do d + 1 ≥ 2 d + 1 d + 1 nên Gi ≠ ∅ ∀i = 1, k . Xét d + 2 d + 2 d + 1 tập Gi bất kỳ, tập F1 , F2 , … , Fd +1 và Fk +1 tập bất kỳ có giao khác ∅ , theo chứng minh trên cho d +1 có tập, cần chứng minh tập bất kỳ có giao khác ∅ . Đặt không mất tổng quát giả sử là G 1 , G 2 , … , G d +1 . Xét thỏa mãn + 2 tập. tập F1 , F2 , … , Fk +1 trong đó k + 1 k ≥ d tập này có giao khác ∅ , tức là d + 2 tập ta Gi ≠ ∅ . Lại theo quy nạp cho k tập i =1 Gi ∀i = 1, k với d + 1 tập bất kỳ có giao khác Gi ∀i = 1, k có giao khác ∅ , hay tức là toàn bộ ∅ k + 1 ta suy ra toàn bộ k tập tập Fi ∀i = 1, k + 1 có giao khác ∅ . Định lý được chứng minh. Ở đây ta cũng sẽ đưa ra một cách khác chứng minh định lý Helly cho trường hợp d = 1 . Đây sẽ là cách chứng minh "dễ chịu hơn", mang tính hình học hơn, hoàn toàn không cần phải sử dụng đến định lý Radon nhưng đương nhiên là không thể chứng minh tương tự cho các trường hợp tổng quát hơn. Chứng minh định lý Helly cho Với n ≥ 2 : d = 1 đoạn thẳng trên một đường thẳng sao cho hai đoạn thẳng bất kỳ có điểm chung, ta cần chứng minh tất cả n đoạn thẳng đó cũng có điểm chung. Xét các đoạn thẳng cùng nằm trên trục đầu mút bên trái phải B A O x nằm ngang. Xét đoạn thẳng có có hoành độ lớn nhất (cũng có thể tương tự với đầu mút bên có hoành độ nhỏ nhất). Khi đó tất cả các đoạn thẳng do có giao với đoạn thẳng đã cho nên đều phải chứa A . Đó chính là điểm cần tìm. Chứng minh ở trên rất đơn giản nhưng có thể được sử dụng một cách tương tự trong một số bài toán biến dạng khác của Helly. Ngoài ra trong thực hành trường hợp d = 2 cũng là trường hợp ta rất hay gặp. Nếu kết hợp cách chứng minh hình học của Radon trong trường hợp này với Trường THPT Chuyên Thái Bình 92 HỘI CÁC TRƯỜNG THPT CHUYÊN KHU VỰC DUYÊN HẢI - ĐỒNG BẰNG BẮC BỘ HỘI THẢO KHOA HỌC LẦN THỨ VI cách chứng minh tổng quát của Helly ta cũng có thể có một cách chứng minh Helly thuần túy hình học. Định lý Helly có thể được mở rộng cho vô hạn tập lồi nhưng với điều kiện các tập lồi này phải là tập compact hay hiểu một cách đơn giản hơn là các tập lồi đóng và bị chặn. Chứng minh khi đó của định lý thông qua tính chất giao hữu hạn của không gian compact: giao của một họ các tập hợp là khác rỗng khi và chỉ khi mọi bộ phận hữu hạn các tập trong họ đó có giao khác rỗng. 3. Bài tập áp dụng: Bài tập 1 (Ôlympic toán Canada 2009): Cho n ≥ 4 điểm trong mặt phẳng, trong đó 3 điểm bất kỳ luôn có thể được phủ kín bởi một hình tròn bán kính bằng đơn vị. Chứng minh rằng: Có hình tròn bán kính bằng đơn vị phủ kín cả n điểm này. Giải: Xét n hình tròn bán kính 1 có tâm là n điểm đã cho. Điều kiện "3 điểm bất kỳ luôn có thể được phủ kín bởi một hình tròn bán kính 1" tương đương với điều kiện "3 đường tròn bất kỳ kể trên luôn có giao khác rỗng" (chúng có ít nhất 1 điểm chung là tâm của hình tròn phủ 3 điểm đó). Theo Helly, n có giao khác rỗng, hay tức là có hình tròn bán kính 1 phủ kín cả đường tròn kể trên n điểm ban đầu. Bài tập 2 (Định lý Jung): Trong mặt phẳng cho n điểm, trong đó hai điểm bất kỳ có khoảng cách không 1 phủ tất cả n điểm này. vượt quá 1. Chứng minh rằng: Có hình tròn bán kính 3 Giải: Trước hết ta chứng minh rằng: 3 điểm bất kỳ luôn có thể được phủ kín bởi một hình tròn bán kính 1 . Thật vậy: 3 Xét 3 điểm A, B, C bất kỳ trong đó thoả mãn AB, BC,CA≤1. Có hai trường hợp xảy ra. Trường THPT Chuyên Thái Bình 93 HỘI CÁC TRƯỜNG THPT CHUYÊN KHU VỰC DUYÊN HẢI - ĐỒNG BẰNG BẮC BỘ HỘI THẢO KHOA HỌC LẦN THỨ VI Trường hợp 1: Tam giác A B C nhọn. Không mất tính tổng quát ta giả sử ∠A = max{∠A; ∠B; ∠C} , từ đó 600 ≤ ∠A ≤ 900 . Khi đó R( ABC ) = BC BC 1 1 . ≤ ≤ = 0 0 2sin ∠ A 2sin 60 2sin 60 3 Trường hợp 2: Tam giác ABC tù. Không mất tính tổng quát ta giả sử ∠ A > 90 0 . Khi đó đường tròn đường kính B C Ở đây đường tròn này có bán kính là sẽ phủ kín toàn bộ tam giác A B C . 1 BC 1 . ≤ < 2 2 3 Vậy 3 điểm bất kỳ luôn có thể được phủ kín bởi một hình tròn bán kính 1 . 3 Áp dụng bài tập trên ta có điều phải chứng minh. Bài tập 3: Cho hữu hạn điểm trên mặt phẳng sao cho nếu vẽ một hình tròn bán kính đơn vị có tâm là một điểm tùy ý trong đó thì hình tròn này sẽ chứa nhiều hơn một nửa các điểm còn lại. Hình tròn có bán kính nhỏ nhất là bao nhiêu có thể chứa tất cả các điểm đã cho (hình tròn này không nhất thiết phải có tâm là một trong số các điểm kể trên)? Giải: Xét 2 điểm A, B bất kỳ trong số các điểm đã cho. Vẽ hai hình tròn bán kính 1 có tâm là 2 điểm A, B . Do mỗi hình tròn chứa nhiều hơn một nửa các điểm còn lại nên chắc chắn chúng phải có điểm chung là một trong số các điểm còn lại, giả sử là C khác A và B . Khi đó ta có A B ≤ A C + B C ≤ 2 . Như vậy hai điểm bất kỳ trong số các điểm đã cho luôn có khoảng cách không vượt quá 2. Theo định lý Jung, tồn tại một hình tròn bán kính tất cả các điểm đã cho. Trường THPT Chuyên Thái Bình 94 2 chứa 3 HỘI CÁC TRƯỜNG THPT CHUYÊN KHU VỰC DUYÊN HẢI - ĐỒNG BẰNG BẮC BỘ HỘI THẢO KHOA HỌC LẦN THỨ VI Bài tập 4: Trên đường thẳng có nhất k 2 k + 1 đoạn thẳng sao cho mỗi đoạn thẳng cắt ít đoạn thẳng khác. Chứng minh rằng: Tồn tại đoạn thẳng cắt tất cả các đoạn thẳng còn lại. Giải: Với mỗi đoạn thẳng cho trước ta xây dựng một tập lồi là đoạn thẳng được tạo thành bằng cách lấy hợp của đoạn thẳng đó với tất cả các đoạn thẳng có giao với nó. Mỗi tập lồi như vậy là hợp của ít nhất cộng 2 k + 1 k + 1 đoạn thẳng, và do ta có tổng đoạn thẳng nên hai tập lồi bất kỳ như vậy chắc chắn có đoạn thẳng chung. Do đó theo Helly, tất cả các tập lồi đó có đoạn thẳng chung. Đây chính là đoạn thẳng cần tìm. Bài tập 5 (Ôlympic toán Nga 2003): Trên đường thẳng có 2 k − 1 đoạn thẳng màu trắng và màu đen sao cho mỗi đoạn thẳng cắt ít nhất k 2 k − 1 đoạn thẳng đoạn thẳng khác màu. Chứng minh rằng: Có đoạn thẳng màu trắng cắt tất cả các đoạn thẳng màu đen và có đoạn thẳng màu đen cắt tất cả các đoạn thẳng màu trắng. Giải: Ta chứng minh có đoạn thẳng màu đen cắt tất cả các đoạn thẳng màu trắng. Với mỗi đoạn thẳng màu trắng cho trước ta xây dựng một tập lồi là đoạn thẳng được tạo thành bằng cách lấy hợp của đoạn thẳng đó với tất cả các đoạn thẳng màu đen có giao với nó. Mỗi tập lồi như vậy gồm ít nhất màu đen, và do ta có tổng cộng 2 k − 1 k đoạn thẳng đoạn thẳng màu đen nên hai tập lồi bất kỳ như vậy chắc chắn có đoạn thẳng màu đen chung. Do đó theo Helly, tất cả các tập lồi đó có đoạn thẳng màu đen chung. Đây chính là đoạn thẳng màu đen cần tìm. Hoàn toàn tương tự ta có đoạn thẳng màu trắng cắt tất cả các đoạn thẳng màu đen. Trường THPT Chuyên Thái Bình 95 HỘI CÁC TRƯỜNG THPT CHUYÊN KHU VỰC DUYÊN HẢI - ĐỒNG BẰNG BẮC BỘ HỘI THẢO KHOA HỌC LẦN THỨ VI Bài tập 6 (Định lý điểm trung tâm): Một điểm O được gọi là điểm trung tâm của hệ nếu mỗi nửa mặt phẳng đóng chứa O đều chứa ít nhất n điểm trong mặt phẳng n điểm của hệ đã cho ( O 3 có thể không thuộc hệ điểm). Chứng minh rằng mọi hệ n điểm trong mặt phẳng đều có ít nhất một điểm trung tâm. Giải: Xét các nửa mặt phẳng đóng chứa ít hơn phẳng đóng như vậy chứa ít hơn n n điểm của hệ đã cho. 3 nửa mặt 3 điểm của hệ đã cho. Do đó phần bù của 3 nửa mặt phẳng đóng đó có ít nhất một điểm chung. Theo Helly, phần bù của tất cả các nửa mặt phẳng đóng đó có điểm chung, giả sử là điểm ngoài tất cả các nửa mặt phẳng đóng chứa ít hơn nửa mặt phẳng đóng chứa O đều chứa ít nhất O . Do O nằm n điểm của hệ đã cho nên mỗi 3 n điểm của hệ đã cho, tức là O chính là 3 điểm trung tâm. Bài tập 7: Cho hữu hạn các đa giác (có thể không lồi) trên mặt phẳng sao cho hai đa giác bất kỳ có điểm chung. Chứng minh rằng: Có vô số đường thẳng cắt tất cả các đa giác đó. Giải: Chiếu vuông góc tất cả các đa giác trên xuống một đường thẳng d bất kỳ. Mỗi đa giác sẽ có hình chiếu vuông góc là một đoạn thẳng. Do hai đa giác bất kỳ có điểm chung nên hai đoạn thẳng hình chiếu bất kỳ cũng có điểm chung. Theo Helly tất cả các đoạn thẳng hình chiếu cũng có điểm chung. Từ điểm Trường THPT Chuyên Thái Bình 96 HỘI CÁC TRƯỜNG THPT CHUYÊN KHU VỰC DUYÊN HẢI - ĐỒNG BẰNG BẮC BỘ HỘI THẢO KHOA HỌC LẦN THỨ VI chung này ta kẻ đường thẳng d' vuông góc với ban đầu. Do có vô số đường thẳng d d thì d' sẽ cắt tất cả các đa giác nên cũng có vô số đường thẳng d' cần tìm. Bài tập 8: Cho hữu hạn các đoạn thẳng song song trong mặt phẳng sao cho với 3 đoạn thẳng bất kỳ luôn tồn tại một đường thẳng cắt cả 3 đoạn thẳng đó. Chứng minh rằng tồn tại đường thẳng cắt tất cả các đoạn thẳng đã cho. Giải: Ta xét bài toán trong mặt phẳng tọa độ Oxy . Gọi các đoạn thẳng là Mi Ni và giả sử chúng song song với trục Oy . Xây dựng các tập hợp như sau: Pi = {( a , b ) ∈ 2 đường thẳng y = ax + b cắt đoạn thẳng M i N i } Dễ dàng chứng minh được Pi là các tập lồi thỏa mãn 3 tập lồi bất kỳ có giao khác rỗng. Do đó theo Helly, tất cả các tập lồi trên có giao khác rỗng, tức là tồn tại đường thẳng cắt tất cả các đoạn thẳng đã cho. Bài tập 9: Trên đường tròn cho hữu hạn cung tròn có số đo nhỏ hơn 1800 sao cho hai cung bất kỳ luôn có điểm chung. Chứng minh rằng: Tồn tại đường thẳng đi qua tâm đường tròn cắt tất cả các cung đã cho. Giải: Ta xét bài toán trong mặt phẳng tọa độ Oxy và giả sử đường tròn có tâm là gốc tọa độ O và bán kính bằng 1. Xây dựng các tập hợp như sau: Pi = {k ∈ đường thẳng y = kx cắt cung thứ i} Hay cụ thể hơn: -) Đường thẳng cắt cung tròn tại điểm A ( x 0 ; y 0 ) với x0 ≠ 0 thì k = -) Đường thẳng cắt cung tròn tại điểm B(0; −1) thì Trường THPT Chuyên Thái Bình 97 k = − ∞ . y0 . x0 HỘI CÁC TRƯỜNG THPT CHUYÊN KHU VỰC DUYÊN HẢI - ĐỒNG BẰNG BẮC BỘ HỘI THẢO KHOA HỌC LẦN THỨ VI -) Đường thẳng cắt cung tròn tại điểm C (0;1) thì k = + ∞ . Với cách xây dựng như vậy thì mỗi cung tròn sẽ được chuyển thành các khoảng có dạng ( −∞; a ],[ a; b ],[ a; +∞ ) . Đây đều là các tập lồi. Do hai cung bất kỳ luôn có điểm chung nên hai khoảng bất kỳ cũng có điểm chung. Do đó theo Helly, tất cả các khoảng trên có điểm chung, chẳng hạn là k . Khi đó đường thẳng y = kx cắt tất cả các cung đã cho. Bài tập 10: Phủ đường tròn bằng một số hữu hạn các nửa đường tròn đóng. Chứng minh rằng: Có 3 nửa đường tròn đóng phủ kín đường tròn đã cho. Giải: Để tạo ra các hình lồi ta sẽ chuyển bài toán về dạng: "Phủ hình tròn bằng một số hữu hạn các nửa hình tròn đóng. Chứng minh rằng: Có 3 nửa hình tròn đóng phủ kín hình tròn đã cho." Ta chứng minh bằng phản chứng: Giả sử 3 nửa hình tròn đóng bất kỳ không thể phủ kín hình tròn đã cho. Khi đó với mỗi 3 nửa hình tròn đóng bất kỳ luôn tồn tại một điểm tròn đã cho sao cho A A trong hình không nằm trong nửa hình tròn đóng nào trong số 3 nửa hình tròn đóng đó. Khi đó điểm A sẽ nằm trong cả 3 nửa hình tròn mở là "phần bù" của 3 nửa hình tròn đóng đó. Do đó theo Helly tất cả các nửa hình tròn mở "phần bù" đó đều có điểm chung, chẳng hạn là B . Khi đó điểm B sẽ không nằm trong một nửa hình tròn đóng nào cả. Mâu thuẫn này chứng minh bài toán. Bài tập 11: Chứng minh rằng: Với mọi đa giác lồi cho trước luôn tồn tại một điểm trong đa giác sao cho với mỗi đường thẳng tùy ý qua và B ta luôn có 1 OA ≤ ≤ 2. 2 OB Trường THPT Chuyên Thái Bình 98 O cắt biên của đa giác tại O A HỘI CÁC TRƯỜNG THPT CHUYÊN KHU VỰC DUYÊN HẢI - ĐỒNG BẰNG BẮC BỘ HỘI THẢO KHOA HỌC LẦN THỨ VI Giải: Với mỗi điểm vị tự tâm P trọng tâm P trên biên của đa giác gọi S P là ảnh của đa giác qua phép 2 tỉ số . Xét 3 điểm P1 , P2 , P3 bất kỳ trên biên của đa giác ta luôn có 3 G của tam giác P1 P2 P3 là điểm chung của 3 hình lồi compact S P1 , S P2 , S P3 . Khi đó theo Helly tất cả các hình lồi S P ở trên đều có điểm chung là O . Và O chính là điểm cần tìm. Bài tập 12: Cho tập S gồm A n ≥ 5 bộ 3 số ( xi , yi , zi ) ∀i = 1, n sao cho với bất kỳ tập con gồm 4 bộ 3 số trong đó luôn tồn tại bộ 3 số (a, b, c) thỏa mãn ax i + by i + cz i > 0 ∀ ( x i , y i , z i ) ∈ A . Tồn tại không bộ 3 số (d , e, f ) thỏa mãn dx i + ey i + fz i > 0 ∀ ( x i , y i , z i ) ∈ S ? Giải: Câu trả lời của bài toán là "có". Với mỗi vectơ xét nửa không gian chứa vectơ đó tạo bởi một mặt phẳng đi qua O và nhận vectơ đó là vectơ pháp tuyến. Do 4 vectơ bất kỳ luôn nằm trong cùng một nửa không gian nên 4 nửa không gian ứng với 4 vectơ đó luôn có vectơ chung (chúng cùng chứa vectơ pháp tuyến của nửa không gian chứa 4 vectơ đó). Theo Helly, tất cả các nửa không gian đó đều có vectơ chung. Từ đó tất cả các vectơ đã cho đều nằm trong cùng một nửa không gian có vectơ pháp tuyến là vectơ chung đó. Bài toán này có thể được mô tả hoàn toàn tương tự nếu ta thay các vectơ chung gốc O thành các điểm đầu mút của chúng. Khi đó bài toán mới sẽ có dạng: Trường THPT Chuyên Thái Bình 99 HỘI CÁC TRƯỜNG THPT CHUYÊN KHU VỰC DUYÊN HẢI - ĐỒNG BẰNG BẮC BỘ HỘI THẢO KHOA HỌC LẦN THỨ VI Bài tập 13: Trong không gian cho n ≥ 5 điểm, trong đó 4 điểm bất kỳ luôn nằm trong cùng một nửa không gian tạo bởi một mặt phẳng đi qua O . Tồn tại không một nửa không gian tạo bởi một mặt phẳng đi qua O chứa tất cả n điểm đã cho?" Bài tập 14 (Chọn đội tuyển Iran thi Ôlympic toán Quốc tế 2002): Cho một đường cong kín trên một mặt cầu đơn vị sao cho mỗi đường tròn lớn đều có giao với đường cong đó. Chứng minh rằng: đường cong đó có độ dài ít nhất là 2 . Giải: Theo bài tập 13, nếu 4 điểm bất kỳ của đường cong luôn nằm trong một nửa không gian tạo bởi mặt phẳng qua tâm cầu thì toàn bộ đường cong cũng nằm trong một nửa không gian tạo bởi mặt phẳng qua tâm cầu, mâu thuẫn. Do đó sẽ tồn tại 4 điểm A, B, C, D trên đường cong sao cho tâm cầu khối tứ diện A B C D O nằm trong . Phần đường cong nối hai điểm A, B có độ dài nhỏ nhất là cung tròn AB . Khi đó đường cong kín ban đầu có độ dài nhỏ nhất là tổng độ dài các cung tròn AB, BC , CD, DA . Gọi D ' là điểm đối xứng tâm của D . Khi đó bởi đường tròn ngoại tiếp tam giác A B C D ' sẽ nằm trong chỏm cầu giới hạn . Dễ dàng chứng minh được AB + BC ≥ AD ′ + D ′C . Do đó AB + BC + CD + DA ≥ AD′ + D′C + CD + DA = 2 . Bài toán được giải quyết xong. Bài tập 15 (Ôlympic toán Iran 2006): Trên bầu trời đêm có hữu hạn ngôi sao. Mỗi ngôi sao tỏa sáng trong một khoảng thời gian nhất định. Biết rằng với k > 1 ngôi sao bất kỳ, có ít nhất hai ngôi sao tỏa sáng cùng một thời điểm. Chứng minh rằng có thể chụp k − 1 ảnh về bầu trời sao cho mỗi ngôi sao tỏa sáng trong ít nhất một bức ảnh. Trường THPT Chuyên Thái Bình 100 bức HỘI CÁC TRƯỜNG THPT CHUYÊN KHU VỰC DUYÊN HẢI - ĐỒNG BẰNG BẮC BỘ HỘI THẢO KHOA HỌC LẦN THỨ VI Giải: Ta chứng minh bằng quy nạp theo k . Với k = 2 , bài toán đúng theo định lý Helly. Giả sử ta đã có khẳng định với k ≥ 2 , ta sẽ chứng minh khẳng định với k + 1 . Thật vậy, trong tất cả các đoạn kể trên, chọn đoạn I n = [ a n ; bn ] sao cho an lớn nhất. Chọn một điểm là an. Khi đó tất cả các đoạn có giao với In đều chứa an. Bỏ tất cả các đoạn này đi và xét các đoạn còn lại. Nếu trong các đoạn còn lại có k đoạn đôi một không giao nhau thì cộng thêm cả đoạn In ta sẽ có một không giao nhau, mâu thuẫn. Do đó với k k + 1 đoạn đôi đoạn bất kỳ trong các đoạn còn lại này luôn có hai đoạn giao nhau. Từ đó theo quy nạp ta có thể xác định k − 1 điểm sao cho mỗi đoạn trong các đoạn còn lại này chứa ít nhất một điểm trong đó. Cộng thêm điểm an ta có k điểm cần tìm. Bài tập 16: Trên mặt bàn hình chữ nhật có các hình vuông bằng nhau và có các cạnh song song với cạnh bàn sao cho với k hình vuông bất kỳ luôn tồn tại hai hình vuông có thể đóng chặt vào bàn chỉ bằng một chiếc đinh. Chứng minh rằng có thể đóng chặt tất cả các hình vuông vào bàn bằng 2 k − 3 chiếc đinh. Giải: Ta chứng minh bằng quy nạp theo k . Trường hợp O x k = 2 : Chiếu vuông góc các hình vuông đã cho xuống trục . Khi đó hình chiếu của các hình vuông là các đoạn thẳng bằng nhau trong đó hai đoạn thẳng bất kỳ luôn có điểm chung. Theo Helly suy ra tất cả các đoạn thẳng hình chiếu đó cũng có điểm chung, giả sử là M ( x0 ;0) nằm trên O x . Hoàn toàn tương tự khi chiếu các hình vuông đã cho xuống trục Oy ta cũng thu được tất cả các đoạn thẳng hình chiếu đó cũng có điểm chung, giả sử là N ( 0; y 0 ) nằm Trường THPT Chuyên Thái Bình 101 HỘI CÁC TRƯỜNG THPT CHUYÊN KHU VỰC DUYÊN HẢI - ĐỒNG BẰNG BẮC BỘ HỘI THẢO KHOA HỌC LẦN THỨ VI trên Oy . Khi đó các hình vuông đã cho có điểm chung là P ( x 0 ; y 0 ) . Vậy trong trường hợp này ta chỉ cần một chiếc đinh tại P . Giả sử ta đã có khẳng định với k + 1 k ≥ 2 , ta sẽ chứng minh khẳng định với . Thật vậy, xét trong mặt phẳng tọa độ Oxy với các trục Ox, Oy song song với cạnh bàn. Giả sử mỗi hình vuông có độ dài cạnh bằng đơn vị và phương trình các cạnh của hình vuông thứ i có dạng: x = x i , x = x i − 1, y = y i , y = y i − 1 . Trong tất cả các hình vuông kể trên, chọn hình vuông thứ n sao cho yn lớn nhất (hình vuông ở vị trí cao nhất). Đóng hai chiếc đinh tại hai điểm là A ( x n − 1; y n − 1 ) và B ( x n ; y n − 1 ) (hai góc phía dưới của hình vuông). Khi đó tất cả các hình vuông có giao với hình vuông này đều chứa A hoặc B . Bỏ tất cả các hình vuông này đi và xét các hình vuông còn lại. Nếu trong các hình vuông còn lại có k hình vuông đôi một không giao nhau thì cộng thêm cả hình vuông thứ ở trên ta sẽ có k k + 1 n hình vuông đôi một không giao nhau, mâu thuẫn. Do đó với hình vuông bất kỳ trong các hình vuông còn lại này luôn có hai hình vuông giao nhau. Từ đó theo quy nạp ta có thể xác định 2 k − 3 điểm đóng đinh khác sao cho mỗi hình vuông trong các hình vuông còn lại này chứa ít nhất một điểm trong đó. Cộng thêm điểm A và Trường THPT Chuyên Thái Bình B ta có 2 k − 1 102 điểm đóng đinh cần tìm. HỘI CÁC TRƯỜNG THPT CHUYÊN KHU VỰC DUYÊN HẢI - ĐỒNG BẰNG BẮC BỘ HỘI THẢO KHOA HỌC LẦN THỨ VI KẾT LUẬN Tổ hợp, rời rạc là một chủ đề khó nhưng rất quan trọng trong các kỳ thi học sinh giỏi các cấp. Đây cũng là mảng mà học sinh Hải Phòng nói riêng, học sinh Việt Nam nói chung còn yếu. Một phần điều này là do các em thường có tư tưởng "sợ" tổ hợp, cứ nhìn thấy bài toán có hơi hướng tổ hợp, rời rạc là cho rằng đấy là bài toán khó, không dám dành thời gian thỏa đáng để học tập, nghiên cứu và giải quyết. Một phần khác cũng là do nhiều thầy cô giáo mới chỉ giảng dạy tổ hợp, rời rạc cho các em theo hướng ra các bài toán có tính riêng lẻ, chưa mang tính hệ thống, chưa làm toát lên được đường lối, định hướng, phương pháp giải cho nhóm các bài toán có liên quan. Trong phạm vi đề tài này tác giả đã đưa ra một cách xây dựng nội dung lý thuyết và hệ thống bài tập về định lý Helly, một mảng bài tập hay gặp trong phần hình lồi, bao lồi. Nội dung này ít nhiều đã thể hiện được tính hiệu quả khi áp dụng vào thực tế giảng dạy, góp phần nâng cao kết quả học tập, nghiên cứu, giải quyết, thậm chí là sáng tạo các bài toán. Dù đã rất cố gắng song tác giả vẫn còn rất cần sự góp sức, nhận xét, bổ sung của các thầy cô, bạn đồng nghiệp và độc giả. Tác giả xin sẵn sàng lắng nghe và cám ơn các ý kiến đóng góp quý báu! Xin chân thành cảm ơn! Trường THPT Chuyên Thái Bình 103 HỘI CÁC TRƯỜNG THPT CHUYÊN KHU VỰC DUYÊN HẢI - ĐỒNG BẰNG BẮC BỘ HỘI THẢO KHOA HỌC LẦN THỨ VI Chuyên đề xếp loại B CHUYÊN ĐỀ DUYÊN HẢI NĂM 2013 MỘT SỐ BÀI TOÁN CỰC TRỊ LIÊN QUAN ĐẾN TẬP HỢP TRẦN NGỌC THẮNG GV THPT CHUYÊN VĨNH PHÚC, TỈNH VĨNH PHÚC Chuyên ngành toán tổ hợp là một bộ phận quan trọng, hấp dẫn và lí thú của Toán học nói chung và toán rời rạc nói riêng. Nội dung của toán tổ hợp phong phú và được ứng dụng nhiều trong thực tế đời sống. Trong toán sơ cấp, tổ hợp cũng xuất hiện trong rất nhiều bài toán với độ khó rất cao. Tổ hợp có vị trí đặc biệt trong toán học không chỉ như là những đối tượng để nghiên cứu mà còn đóng vai trò như một công cụ đắc lực của các mô hình rời rạc của giải tích, đại số, hình học... Với vai trò quan trọng trong toán học như vậy nên trong hầu hết các kì thi học sinh giỏi quốc gia, thi Olimpic toán quốc tế, thi Olimpic sinh viên giữa các trường đại học và cao đẳng, các bài toán liên quan đến tổ hợp thường là các bài toán rất khó, là những bài tập phân loại học sinh rất tốt. Phương pháp giải các bài toán tổ hợp thường rất phong phú và đa dạng. Nhìn chung để giải một bài toán tổ hợp thông thường học sinh phải sáng tạo ra phương pháp và cách thức tiếp cận bài toán. Do đó khi giảng dạy phần tổ hợp thì điều quan trọng là với mỗi bài toán giáo viên nên phân tích, định hướng lời giải một cách cụ thể để học sinh hiểu được ý tưởng cũng như mục đích của bài toán. Để cho việc giảng dạy toán phần tổ hợp đạt được kết quả tốt, chúng tôi mạnh dạn viết chuyên đề "sử dụng số phức để giải một số dạng toán tổ hợp" để trao đổi với các thầy, cô giáo về phương pháp giảng dạy các bài toán tổ hợp. Trong chuyên đề này, một số dạng bài tập được chọn lọc là các đề ra của các kì thi học sinh giỏi quốc gia, quốc tế, Olimpic sinh viên giữa các trường đại học trên thế giới những năm gần đây. Trường THPT Chuyên Thái Bình 104 HỘI CÁC TRƯỜNG THPT CHUYÊN KHU VỰC DUYÊN HẢI - ĐỒNG BẰNG BẮC BỘ HỘI THẢO KHOA HỌC LẦN THỨ VI Chuyên đề được chia làm hai phần chính: I. Phần bài tập minh họa II. Phần bài tập tương tự Những bài toán tổ hợp xuất hiện trong các đề thi chọn học sinh giỏi những năm gần đây thường là các bài tập hay và khó, có độ phân hóa cao giữa các đối tượng học sinh. Với thời gian ngắn thì học sinh thường rất khó để giải quyết được các bài toán dạng này và đây cũng là vấn đề rất nan giải trong công tác ôn luyện học sinh giỏi của đa số giáo viên. Số lượng cũng như số dạng bài toán tổ hợp là rất nhiều (có thể nói là vô hạn) nên giáo viên không thể dạy hết tất cả được, mà cần phải có phương pháp hiệu quả nhất để trang bị cho học sinh cách tiếp cận cũng như các kiến thức cơ sở trong việc giải quyết các bài toán tổ hợp. Chuyên đề được hoàn thành với sự giúp đỡ nhiệt tình cả về nội dụng và hình thức của các thầy, cô giáo trong tổ toán - tin, BGH trường THPT chuyên Vĩnh Phúc. Do thời gian và trình độ có hạn nên trong bài viết chỉ đề cập đến một khía cạnh rất nhỏ của dạng toán tổ hợp, rất mong nhận được góp ý và các phương pháp hiệu quả để việc giảng dạy phân môn này có hiệu quả hơn. I. MỘT SỐ BÀI TẬP MINH HỌA Bài 1. Cho tập hợp và 3 phần tử phân biệt bất kì trong nhất của M là tập hợp con của tập M M X có tính chất T nếu: tích của đều không là số chính phương. Tìm số phần tử lớn . Lời giải. Xét 4 tập hợp rời nhau có 3 phần tử và tích của 3 phần tử đều là số chính phương: {1, 4, 9} , {2, 7,14} , {5,12,15} , {3, 6,8} Nếu tập hợp trên không thuộc M M có tính chất T thì có ít nhất một phần tử trong mỗi tập suy ra M ≤ 11 . Giả sử M = 11 : Do M có tính chất T nên mỗi tập trong 4 tập hợp {1, 4, 9} , {2, 7,14} , {5,12,15} , {3, 6,8} phải có đúng hai phần tử thuộc M và các phần tử 1 0 ,1 1,1 3 ∈ M . Khi đó với tập hợp {5,12,15} ta xét các trường hợp sau: Trường THPT Chuyên Thái Bình 105 HỘI CÁC TRƯỜNG THPT CHUYÊN KHU VỰC DUYÊN HẢI - ĐỒNG BẰNG BẮC BỘ HỘI THẢO KHOA HỌC LẦN THỨ VI +) M 5 ,1 2 ∈ M ⇒ 2 ∉ M ⇒ 7 ,1 4 ∈ M ⇒ 8 ∉ M ⇒ . Do 3,6 ∈ M 3 .1 2 = 6 2 nên không được chứa một phần tử nào của {1, 4, 9} vô lí. +) 5 ,1 5 ∈ M +) 1 2 , 1 5 ⇒ ∈ M 3 ∉ M ⇒ ⇒ 6 ∉ M 6,8 ∈ M ⇒ ⇒ 2 ∉ M . Do 3,8 ∈ M ⇒ 3 .1 2 = 6 phần tử nào của {1, 4, 9} vô lí. Vậy . Do 7 ,1 4 ∈ M nên 2 M ≤ 10 . M 7 .1 4 .8 = 2 8 2 vô lí. không được chứa một Mặt khác nếu ta lấy M = {1, 4,5, 6, 7,10,11,12,13,14} Vậy số phần tử lớn nhất của tập hợp M Bài 2. Cho tập hợp X = {1, 2,3,...,16} và tính chất T nếu: M là 10. là tập hợp con của tập M M . Lời giải. Xét tập hợp số A = {1, 2,3,5,7,11,13} , ta thấy nếu tập hợp T thì M có không chứa ba phần tử nào đôi một nguyên tố cùng nhau. Tìm số phần tử lớn nhất của mãn tính chất X chỉ chứa nhiều nhất 2 phần tử của tập hợp gồm 11 phần tử sau thỏa mãn tính chất Vậy số phần tử lớn nhất của A ⇒ M ≤ 11 . M thỏa Mặt khác : {2, 3, 4, 6,8, 9,10,12,14,15,16} . T là 11. M Bài 3(VMO 2004). Cho tập hợp A = {1, 2,3,...,16} . Hãy tìm số nguyên dương nhỏ nhất sao cho trong mỗi tập con gồm k hai số phân biệt a,b thỏa mãn a 2 + b 2 k phần tử của A đều tồn tại là một số nguyên tố. Lời giải. Xét tập hợp B = {2, 4, 6,8,10,12,14,16} , ta thấy a , b ∈ B ⇒ a 2 + b 2 là một số chẵn lớn hơn 2 nên minh A k = 9 k thỏa mãn yêu cầu bài toán thì k ≥ 9 . Ta sẽ chứng sẽ là số nhỏ nhất thỏa mãn yêu cầu bài toán. Thật vậy, ta chia tập hợp thành 8 cặp hai phần tử (a, b) sao cho a 2 + b 2 là một số nguyên tố: (1, 4 ) , ( 2, 3) , ( 5,8 ) , ( 6,11) , ( 7,10 ) , ( 9,16 ) , (12,13 ) , (14,15 ) Do đó theo nguyên tắc Dirichlet thì trong 9 phần tử phân biệt của tập hợp A phải tồn tại hai số thuộc cùng một cặp. Vậy Trường THPT Chuyên Thái Bình 106 k nhỏ nhất bằng 9. HỘI CÁC TRƯỜNG THPT CHUYÊN KHU VỰC DUYÊN HẢI - ĐỒNG BẰNG BẮC BỘ HỘI THẢO KHOA HỌC LẦN THỨ VI Bài 4. Cho tập hợp M = {1, 2,..., n} , n ≥ 2 . Hãy tìm số m nhỏ nhất sao cho trong mỗi tập con chứa m phần tử của M đều tồn tại ít nhất hai số a,b mà số này là bội của số kia. Lời giải. Xét tập con M 1 =   n + 1  ,  n + 1  + 1,..., n  của M. Do  2   2   nên trong M1 không có hai số mà số này chia hết cho số kia. Do đó n +1 m =  +1  2  ta sẽ chứng minh TH1. Nếu n = 2k  2  +1   , ta viết có đúng k số lẻ nên tồn tại i ≠ n +1 m ≥  +1,  2  là số nhỏ nhất thỏa mãn yêu cầu bài toán. Thật   vậy, xét tập con  a1 , a2 ,..., a n +1  của M. Ta xét hai trường hợp   n + 1 2 > n  2   a i = 2 bi ci , j trong đó ci là số lẻ, sao cho ci = c j ⇒ một n chẵn và i = 1 , 2 , ..., k + 1 trong hai số ai , a j n lẻ. . Do chỉ có một số chia hết cho số kia. TH2. Nếu n = 2k + 1 , ta viết a i = 2 bi ci , chỉ có nhiều nhất k+1 số lẻ nên tồn tại ai , a j trong đó ci là số lẻ, i ≠ j sao cho i = 1, 2 , ..., k + 2 ci = c j ⇒ một . Do trong hai số có một số chia hết cho số kia. Vậy số nhỏ nhất thỏa mãn yêu cầu là n +1 m =  +1.  2  Bài 5. Cho X là một tập con của tập {1, 2,3,...,10000} , sao cho nếu trong X thì ab a,b nằm không nằm trong X. Tìm số phần tử lớn nhất của tập X. Lời giải. Xét tập hợp con M = {101,102,...,10000} của {1, 2,3,...,10000} , do 1012 > 10000 nên tập hợp M thỏa mãn yêu cầu bài toán, tập hợp M có 9900 phần tử. Ta sẽ chứng minh 9900 là số lớn nhất thỏa mãn yêu cầu bài toán. Thật vậy, xét tập con A gồm có 9901 phần tử, ta chứng minh tập A không thỏa mãn yêu cầu bài toán. Xét 100 bộ số sau : (100 − i,100 + i, (100 − i )(100 + i ) ) , 0 Trường THPT Chuyên Thái Bình 107 ≤ i ≤ 99 . Dễ thấy nếu cả 3 số của HỘI CÁC TRƯỜNG THPT CHUYÊN KHU VỰC DUYÊN HẢI - ĐỒNG BẰNG BẮC BỘ HỘI THẢO KHOA HỌC LẦN THỨ VI mỗi bộ trên thuộc A thì vô lý suy ra phải có ít nhất một số trong mỗi bộ đó không thuộc A ⇒ A ≤ 10000 − 100 = 9900 , vô lí. Vậy số phần tử lớn nhất của X bằng 9900. Bài 6. Cho A là một tập con của tập hợp {1; 2; 3;...;100} , nhất là 1 và phần tử lớn nhất là 100. Giả sử của A , thì x ≠ 1 x A A có phần tử nhỏ có tính chất: Với mỗi phần tử hoặc bằng tổng của hai phần tử thuộc A x hoặc bằng hai lần một phần tử thuộc A . Tìm số phần tử nhỏ nhất có thể của tập hợp A . Lời giải. Giả sử tập hợp A gồm Với mỗi số ta có: 2 ≤ i ≤ n x i = x j + x s ≤ 2 x i −1 . Do đó x2 ≤ 2 x1 = 2, x3 ≤ 2 x2 = 4, x4 ≤ 2 x3 = 8 , vậy n ≥ 8 phần tử là 1 = x1 < x2 < ... < xn −1 < xn = 100 . n x5 ≤ 2 x4 = 16, x6 ≤ 2 x5 = 32, x7 ≤ 2 x6 = 64 . Vì . Nếu n = 8 ⇒ x8 = 100 , kết hợp với x6 + x7 ≤ 64 + 32 = 96 ⇒ x8 = 2 x7 ⇒ x7 = 50 . Do x5 + x6 ≤ 48 ⇒ x7 = 2 x6 ⇒ x6 = 25 . Mặt khác x4 + x5 ≤ 2 4 ⇒ x6 = 2 x5 ⇒ x5 = 25 2 vô lý. Do đó n ≥ 9 , với n = 9 ta lấy tập hợp A = {1, 2, 3, 5,10, 20, 25, 50,100} thỏa mãn yêu cầu bài toán. Vậy số phần tử nhỏ nhất của tập hơp A là 9. Bài 7. Cho tập hợp X i ≠ X j , ∀ i ≠ j, X i đó ta có X 2k n ≥ 2 phần tử . Xét k k ≥ 2 tập con của X thỏa mãn . Tìm giá trị lớn nhất có thể có của k . tập Y1 = X \ X 1 , Y2 = X \ X 2 ,..., Yk = X \ X k . Do X j ≠ ∅ , ∀ i , j = 1, 2, ..., k nên Yi ≠ Y j , ∀ i ≠ j , Yi X j , ∀ i , j = 1, 2, ..., k . Do tập con đôi một phân biệt của tập X . Mặt khác số tập con của tập hợp là 2 . Do đó n Với có X j ≠ ∅ , ∀ i , j = 1, 2, ..., k Lời giải. Xét X i ≠ X j , ∀ i ≠ j, X i X 2 k ≤ 2 k = 2 A1 , A2 ,..., A2n−1 . n −1 n ⇔ k ≤ 2 n −1 . , ta xét phần tử a ∈ X và gọi 2 n − 1 tập con của X \ {a} là Khi đó X 1 = A1 {a} , X 2 = A2 {a} ,..., X 2 = A2 n −1 n −1 của X thỏa mãn yêu cầu bài toán. Vậy giá trị lớn nhất của Trường THPT Chuyên Thái Bình 108 {a} là k = 2 n −1 . 2 n − 1 tập con HỘI CÁC TRƯỜNG THPT CHUYÊN KHU VỰC DUYÊN HẢI - ĐỒNG BẰNG BẮC BỘ HỘI THẢO KHOA HỌC LẦN THỨ VI là các số nguyên dương, Bài 8. Cho sao cho bất kì một họ gồm nhất . Tìm số nguyên dương nhỏ tập hợp con của tập đều tìm được ba tập hợp phân biệt khác rỗng mà một trong chúng là hợp của hai tập hợp còn lại. Lời giải. Trước hết ta chứng minh hai bổ đề sau đây: Bổ đề 1. Cho tập con của tập hợp là một số nguyên . Khi đó luôn tồn tại một họ sao cho bất kì ba tập hợp phân biệt khác rỗng của họ này đều không thỏa mãn tính chất một trong chúng là hợp của hai tập hợp còn lại. Chứng minh. Ta sẽ chứng minh bổ đề này bằng quy nạp như sau: đễ thấy thỏa mãn. +) Khi +) Ta giả sử bổ đề này đúng đến tồn tại tập hợp , tức là từ tập hợp luôn sao cho không có tập hợp nào là hợp của hai tập hợp phân biệt khác nó. Ta xét tập hợp . Khi đó tập sau: Thỏa mãn không có tập hợp nào là hợp của hai tập hợp phân biệt khác nó. Vậy bổ đề 1 được chứng minh. Bổ đề 2. Cho là một số nguyên tập hợp con của tập hợp nhất . Chứng minh rằng mọi họ gồm ít đều tìm được ba tập hợp phân biệt khác rỗng mà một trong chúng là hợp của hai tập hợp còn lại. Chứng minh. Ta chứng minh bằng quy nạp toán học. +) Khi đễ thấy bổ đề 2 là đúng. Trường THPT Chuyên Thái Bình 109 HỘI CÁC TRƯỜNG THPT CHUYÊN KHU VỰC DUYÊN HẢI - ĐỒNG BẰNG BẮC BỘ HỘI THẢO KHOA HỌC LẦN THỨ VI +) Giả sử bổ đề 2 đúng đến , tức là mọi họ tập con của tập luôn tồn tại ba tập hợp phân biệt khác rỗng mà một trong chúng là hợp hợp của hai tập hợp còn lại. tập con của tập hợp Ta chứng minh trong bất kì luôn tồn tại ba tập hợp phân biệt khác rỗng mà một trong chúng là hợp của hai tập hợp còn lại. Thật vậy, số tập con chứa bằng tập con của tập hợp . Gọi là số tập con trong đã xét ở trên. Khi đó ta xét các trường hơp: thì theo giả thiết quy nạp ta có đpcm. TH1. Nếu TH2. Nếu tập con chứa thì có ít nhất tập đã xét ở trên. Giả sử ta xét trong tập dạng này là . Đặt . i) Nếu thì theo giả thiết quy nạp ta có đpcm ii) Nếu tồn tại sao cho thì tập đã . Trong giả sử nó là . chọn ở trên phải có một tập con khác rỗng của Nếu thì theo giả thiết quy nạp ta có đpcm. Nếu thì ba tập thỏa mãn yêu cầu bài toán. Vậy bổ đề 2 được chứng minh. Trở lại BÀI 3 th́ dễ thấy nhỏ nhất bằng . Bài 9 (Bài toán về khoảng cách Hamming). Cho có phần tử. Gọi và tập hợp là tập hợp chứa các xâu (mỗi kí tự là một phần tử của X hoặc phần tử 0) có độ dài bằng và khoảng cách Hamming có độ dài không nhỏ là số phần tử lớn nhất của tập hợp . Khi đó hơn . Kí hiệu i) Nếu chẵn và Trường THPT Chuyên Thái Bình thì 110 HỘI CÁC TRƯỜNG THPT CHUYÊN KHU VỰC DUYÊN HẢI - ĐỒNG BẰNG BẮC BỘ HỘI THẢO KHOA HỌC LẦN THỨ VI ii) Nếu lẻ và iii) Nếu chẵn thì iv) Nếu lẻ thì thì Chứng minh. Giả sử . Ta đồng nhất mỗi phần tử của C với một xâu nhị phân dạng xâu là , trong đó nếu ở vị trí thứ của nếu vị trí thứ của xâu là . và i) Ta sẽ đánh giá theo 2 cách khác nhau, ở đây kí hiệu giữa hai xâu là khoảng cách Hamming . +) Số cách chọn bộ có thứ tự +) Xét ma trận là suy ra: , trong đó mỗi dòng là một phần tử của . Gọi là số số 0 ở cột thứ và tương ứng ở cột thứ đó có hai xâu , có tính đối xứng nên suy Do đó từ hai cách đánh giá ở trên ta được: Trường THPT Chuyên Thái Bình 111 số 1. Do xét quan hệ HỘI CÁC TRƯỜNG THPT CHUYÊN KHU VỰC DUYÊN HẢI - ĐỒNG BẰNG BẮC BỘ HỘI THẢO KHOA HỌC LẦN THỨ VI . Kết hợp với giả thiết suy ra đpcm. Hay ii) Lặp lại cách chứng minh tương tự như trên nhưng đối với tập Một số bài toán áp dụng bài toán khoảng cách Hamming. Bài 10 (Vĩnh Phúc 2012, vòng 2) Có 7 em học sinh được lập thành nhóm hoạt động ngoại khóa, mỗi học sinh có thể tham gia nhiều nhóm hoạt động. Biết rằng với hai nhóm tùy ý thì có ít nhất 4 học sinh chỉ tham gia vào một trong hai nhóm đó.Tìm giá trị lớn nhất có thể của (Gợi ý: Giả sử 7 học sinh là nhóm đó không chứa và đặt , trong đó nhóm là một xâu dạng . . Ta coi mỗi nếu nhóm đó chứa , nếu . Khi đó theo giả thiết khoảng cách Hamming không nhỏ hơn . Áp dụng kết quả phần 2.i ở trên với ta được và trong đánh giá dễ dàng chỉ ra được dấu bằng) Bài 11 (PTNK TPHCM 2012). Cho tập hợp con và của được gọi là không giống nhau nếu . Cho tập hợp của gồm a) . Hai tập , trong đó mà mỗi phần tử là một tập con phần tử đôi một không giống nhau. Chứng minh rằng b) Chứng minh rằng . (Gợi ý: ta coi mỗi tập là một xâu dạng , trong đó trong đó nếu không chứa . Khi đó theo giả thiết khoảng cách Hamming không nhỏ hơn . Áp dụng kết quả phần 2.ii ở trên với nếu chứa , Trường THPT Chuyên Thái Bình 112 HỘI CÁC TRƯỜNG THPT CHUYÊN KHU VỰC DUYÊN HẢI - ĐỒNG BẰNG BẮC BỘ HỘI THẢO KHOA HỌC LẦN THỨ VI ta được và trong đánh giá dễ dàng chỉ ra được dấu bằng) Nhận xét. Như vậy với mỗi cách tạo ra khoảng cách Hamming của hai đối tương nào đó ta được một dạng bài tập tương đối khó. Trong tất cả các dạng bài tập liên quan đến khoảng cách Hamming thì ví dụ 2 thật tinh tế và sâu sắc. II. BÀI TẬP T¦¥NG Tù Bài 12. Cho A1 , A2 ,..., An là các tập hợp có hữu hạn phần tử sao cho A1 = A2 = ... An và n Ai = S . Giả sử có số nguyên dương 1 ≤ k ≤ n thỏa mãn hợp i =1 của bất kì k tập hợp của họ trên bằng S, hợp của nhiều nhất k − 1 tập của họ đã cho là một tập con thực sự của S. Tìm số phần tử nhỏ nhất của S. Bài 13. Cho ( X i )1≤i≤k là một họ các tập con có h phần tử của tập hợp X. k Chứng minh rằng min Xi bằng số nguyên dương m nhỏ nhất sao cho k ≤ C mh . i =1 Bài 14. Cho n là một A = ( Ai )1≤i ≤3 n , B = ( Bi )1≤i ≤3 n , C = ( Ci )1≤i ≤3 n sử ta có i, j , k = 1, 2 , ..., 3 n : Ai bất B j + Ai số nguyên dương cho trước và là ba phân hoạch của tập hợp hữu hạn X . Giả đẳng thức Ck + B j C k ≥ 3n . sau đúng với mọi Tìm số phần tử nhỏ nhất có thể có của tập hợp X . Bài 15 (Định lí Sperner). Cho X là một tập hợp có n phần tử, và G = { A1 , A2 , ..., A p } là một họ các tập con của X thỏa mãn tính chất Ai ⊄ A j , ∀ i , j = 1, 2, ..., p , i ≠ j . Chứng minh rằng n   max p = C n 2  . Bài 16 (Định lí Erdos – Ko - Rado). Cho X là một tập hợp có n phần tử, và G = { A1 , A2 , ..., A p } là một họ các tập con của X thỏa mãn các điều kiện sau: a) Ai = r ≤ n , ∀ i = 1, 2 , ..., p 2 Trường THPT Chuyên Thái Bình 113 HỘI CÁC TRƯỜNG THPT CHUYÊN KHU VỰC DUYÊN HẢI - ĐỒNG BẰNG BẮC BỘ HỘI THẢO KHOA HỌC LẦN THỨ VI b) A j ≠ ∅ , ∀ i , j = 1, 2,..., p . Ai Chứng minh rằng max p = C nr −−11 . Bài 17. Cho X là một tập hợp có n phần tử, và Y là một tập con có k phần tử của X. Chứng minh rằng số lớn nhất các tập con đôi một khác nhau của tập X, mỗi tập có đúng r phần tử của Y và hai tập bất kì thì không chứa nhau bằng n−k     C kr C n − k2 . Bài 18(Balkan MO 2005). Cho con của tập hợp {1,2,...,n} sao cho S là một số nguyên và n ≥ 2 S là một tập hoặc chứa hai phần tử mà phần tử này là bội của phần tử kia, hoặc chứa hai phần tử nguyên tố cùng nhau. Tìm số phần tử lớn nhất của tập hợp S . Bài 19(Balkan MO 1997). Cho tập hợp A có n phần tử và S = { A1 , A2 ,..., Ak } kì x, y ∈ A là một họ các tập hợp con của tập hợp A. Nếu với hai phần tử bất có một tập con Ai ∈S chứa đúng một phần tử trong hai phần tử Chứng minh rằng n ≤ 2k x, y . . Bài 20(Balkan MO 1996). Cho tập X = {1, 2, ..., 2 1996 − 1} , chứng minh rằng tồn tại tập con A của X thỏa mãn đồng thời các điều kiện sau: a) 1 ∈ A, 21996 − 1 ∈ A; b) Với mọi phần tử khác 1 của A đều viết thành tổng của hai (có thể bằng nhau) phần tử thuộc A; c) Số phần tử lớn nhất của tập A là 2012. Bài 21(Balkan MO 1989). Cho F là một họ các tập con của tập hợp {1,2,...,n} và thỏa mãn đồng thời các điều kiện sau: a) Nếu A thuộc F, khi đó A có 3 phần tử; b) Nếu A và B là hai phần tử khác nhau của S, khi đó A và B có nhiều nhất một phần tử chung. Trường THPT Chuyên Thái Bình 114 HỘI CÁC TRƯỜNG THPT CHUYÊN KHU VỰC DUYÊN HẢI - ĐỒNG BẰNG BẮC BỘ HỘI THẢO KHOA HỌC LẦN THỨ VI Kí hiệu f (n) là số phần tử lớn nhất có thể có của F. Chứng minh rằng n2 − 4n n2 − n ≤ f (n) ≤ 6 6 . Bài 22. Cho S là một tập con của tập hợp {1, 2,...,1989} và S thỏa mãn tính chất trong S không có hai phần tử mà hiệu của chúng bằng 4 hoặc 7. Tìm số phần tử lớn nhất của S? Bài 23 (Iran TST 2013) Cho F = { A1 , A2 , ..., A p } tập {1, 2,3,..., n} thỏa mãn tính chất: nếu Ai ⊂ A j thì là một họ các tập con của Ai − A j ≥ 3 . Tìm số lớn nhất có thể có của p . Bài 24 (Moldova TST 2013) Tìm số lớn nhất các cặp phân biệt ( xi , yi ) sao cho xi , yi ∈{1, 2,..., 2013} , x i + y i ≤ 2013, ∀ i ≠ j , x i + y i ≠ x j + y j Bài 25 (China 1996). Cho 11 tập hợp M 1 , M 2 ,..., M 11 , mỗi tập có 5 phần tử và thỏa mãn Mi M j ≠ ∅ , ∀ i ≠ j ; i , j = 1, 2, ...,11 . Gọi m là số lớn nhất sao cho tồn tại các tập M i1 , M i2 ,..., M im Trong số các tập đã cho sao cho m k =1 M ik ≠ ∅ Hỏi giá trị lớn nhất của mbằng bao nhiêu? Bài 26 (AIME 1989). Cho tập hợp X = {1, 2,3,...,1989} . Xét tập con thỏa mãn tính chất: không có hai phần tử nào của vị. Hỏi số phần tử lớn nhất của Trường THPT Chuyên Thái Bình S là bao nhiêu? 115 S S của X hơn kém nhau 4 hoặc 7 đơn HỘI CÁC TRƯỜNG THPT CHUYÊN KHU VỰC DUYÊN HẢI - ĐỒNG BẰNG BẮC BỘ HỘI THẢO KHOA HỌC LẦN THỨ VI Bài 27. Cho số nguyên dương nguyên dương nhau A , B n n ≥ 5 và tập hợp X = {1, 2,..., n} . Tìm số nhỏ nhất sao cho với mọi cách chia tập hợp X thành hai tập rời thì luôn tồn tại một tập chứa ba số lập thành một cấp số cộng. Bài 28 (IMO 1991). Cho tập hợp S = {1, 2,..., 280} . Tìm số nguyên dương nhỏ nhất n sao cho mọi tập con n phần tử của đều chứa 5 số đôi một nguyên tố cùng S nhau. Bài 29. Cho m ,n là các số tự nhiên sao cho nguyên dương k lớn nhất sao cho tồn tại {1,2,...,n} thỏa mãn Ai ∈ {m, m − 1} Trường THPT Chuyên Thái Bình với mọi k và n > 2 m . Tìm số tập con rời nhau A1 , A2 ,..., Ak của tập i = 1, 2 , ..., k 116 m > 1 . HỘI CÁC TRƯỜNG THPT CHUYÊN KHU VỰC DUYÊN HẢI - ĐỒNG BẰNG BẮC BỘ HỘI THẢO KHOA HỌC LẦN THỨ VI TÀI LIỆU THAM KHẢO [1] Nguyễn Văn Mậu (chủ biên), Chuyên đề chọn lọc Tổ hợp và Toán rời rạc, NXB Giáo dục, 2008. [2] Nguyễn Văn Mậu (chủ biên), Toán Rời rạc và một số vấn đề liên quan, Tài liệu bồi dưỡng giáo viên hè 2007, Trường ĐHKHTN - ĐHQG Hà Nội. [3] Trần Nam Dũng (chủ biên), Chuyên đề toán học số 8, 9, Trường PTNK - ĐHQG TP. Hồ Chí Minh. [4] Le Hai Chau - Le Hai Khoi, Selected Problems of the Vietnamese Maththematical Olympiad (1962 - 2009), World Scientific. [5] Tạp chí Toán học tuổi trẻ, Crux - Canada, AMM - USA [6] Titu Andresscu - Zuming Feng, A path to combinatorics for underfrduates, Birkhauser. [7] Arthur Engel, Problem - Solving Strategies, Springer. [8] Titu Andreescu and Zuming Feng. 102 combinatorial problems from the training of the USA IMO team. [9] Phạm Minh Phương. Một số chuyên đề toán học tổ hợp bồi dưỡng học sinh giỏi trung học phổ thông. NXB Giáo dục Việt Nam. [10] Các nguồn tài liệu từ internet www.mathscope.org; www.mathlinks.org; www.imo.org.yu Trường THPT Chuyên Thái Bình 117 HỘI CÁC TRƯỜNG THPT CHUYÊN KHU VỰC DUYÊN HẢI - ĐỒNG BẰNG BẮC BỘ HỘI THẢO KHOA HỌC LẦN THỨ VI Chuyên đề xếp loại B Chuyên đề sử dụng đại lượng bất biến và đơn biến giải bài toán tổ hợp Tổ Toán - Trường THPT Chuyên Hưng Yên A. MỞ ĐẦU 1.Lý do chọn đề tài: Bài toán tổ hợp là bài toán nằm trong cấu trúc bắt buộc của các đề thi học sinh giỏi. Các vấn đề liên quan đến lí thuyết tổ hợp là một bộ phận quan trọng, hấp dẫn và lí thú của Toán học nói chung và toán rời rạc nói riêng, bởi vì nó có nội dung phong phú và được ứng dụng nhiều trong thực tiễn đời sống. Trong chuyên đề này, tôi tập trung khai khác sử dụng đại lượng bất biến và đại lượng đơn biến để giải bài toán tổ hợp nhằm giúp học sinh có thêm một công cụ khi đứng trước một bài toán tổ hợp. 2. Mục đích nghiên cứu: Chuyên đề nhằm hệ thống kiến thức về đại lượng bất biến trong toán tổ hợp, trình bày các kết quả qua quá trình nghiên cứu đại lượng bất biến. Giúp học sinh có kiến thức nền tảng và có thêm một định hướng cho các dạng bài toán tổ hợp. B. NỘI DUNG 1.KIẾN THỨC CƠ BẢN Trong một loạt bài toán ta thường gặp tình huống sau : Một hệ thống nào đó thay đổi liên tục trạng thái của mình và cần phải chỉ ra một điều gì đó về trạng thái cuối cùng của nó. Khảo sát cục bộ sau đó tất cả các lần thay đổi như vậy là một việc làm rất phức tạp và khó khăn. Nhưng ta lại có thể trả lời câu hỏi mà bài toán yêu cầu nhờ tính một đại lượng đặc biệt nào đó đặc trưng cho tất cả Trường THPT Chuyên Thái Bình 118 HỘI CÁC TRƯỜNG THPT CHUYÊN KHU VỰC DUYÊN HẢI - ĐỒNG BẰNG BẮC BỘ HỘI THẢO KHOA HỌC LẦN THỨ VI các trạng thái của hệ thống đó. Hai đại lượng thường được sử dụng là bất biến và đơn biến. Bất biến là một đại lượng (hay tính chất) không thay đổi trong quá trình chúng ta thực hiện các phép biến đổi. Đơn biến là một đại lượng (hay tính chất) thay đổi, nhưng chỉ theo một chiều (tức là tăng lên hoặc giảm xuống). Dựa vào đại lượng bất biến hoặc đơn biến, ta chỉ ra được một số tính chất của trạng thái cuối cùng, từ đó giải quyết được bài toán. Trên thực tế phương pháp sử dụng đại lượng đơn biến hoặc bất biến được tiến hành như sau : Tính một đại lượng nào đó bằng 2 cách: đầu tiên nó được tính ở trạng thái ban đầu và trạng thái cuối cùng, sau đó khảo sát sự thay đổi của nó qua một số lần thay đổi nhỏ liên tiếp. Để thiết lập các bất biến hoặc đơn biến đôi khi người ta còn sử dụng sự tô màu, tức là chia các đối tượng đang xét ra làm các nhóm (mỗi nhóm gồm các đối tượng được đánh dấu cùng một màu). 2. SỬ DỤNG ĐẠI LƯỢNG BẤT BIẾN ĐỂ GIẢI BÀI TOÁN TỔ HỢP 2.1 Bất biến liên quan đến tính chia hết (hoặc số dư trong một phép chia) a) Bất biến là tính chẵn, lẻ Ví dụ 1.1 Cho một bàn cờ kích thích 8 x 8, tô đen một ô bất kì. Mỗi bước cho phép đổi màu tất cả các ô trên cùng một hàng hoặc một cột ( ô đen thay bằng ô trắng và ngược lại). Hỏi có khi nào tất cả các ô trên bàn cờ cùng màu không? Nhận xét: + Việc khảo sát tất cả các phương án đổi màu trong bài toán là không thực hiện được, ta thử xem có quy luật nào chi phối tất cả các phương án này không? Ta xem xét sự thay đổi số lượng ô đen và ô trắng : Ban đầu, có 1 ô đen, 63 ô trắng Trường THPT Chuyên Thái Bình 119 HỘI CÁC TRƯỜNG THPT CHUYÊN KHU VỰC DUYÊN HẢI - ĐỒNG BẰNG BẮC BỘ HỘI THẢO KHOA HỌC LẦN THỨ VI Kết thúc có 64 ô đen, 0 ô trắng (hoặc 64 ô trắng, 0 ô đen) Mỗi bước thực hiện: sẽ đổi màu 8 ô (1 hàng hoặc 1 cột), giả sử trước bước đổi màu thứ k, có xk ô đen, và 64 – xk ô trắng . Bước đổi màu k biến a ô đen thành a ô trắng, 8 – a ô trắng thành 8 – a ô đen. Như vậy, sau bước đổi màu thứ k, số ô đen là xk – a + 8 – a = xk + (8 – 2a), số ô trắng 64 – xk + (2a - 8). Từ đó, ta có được nhận xét về sự thay đổi số lượng ô trắng, đen sau mỗi phép đổi màu. Lời giải: Mỗi bước thực hiện: sẽ đổi màu 8 ô (1 hàng hoặc 1 cột), giả sử trước bước đổi màu thứ k, có xk ô đen, và 64 – xk ô trắng . Bước đổi màu k biến a ô đen thành a ô trắng, 8 – a ô trắng thành 8 – a ô đen. Như vậy, sau bước đổi màu thứ k, số ô đen là xk – a + 8 – a = xk + (8 – 2a), số ô trắng 64 – xk + (2a - 8). Như vậy, sau mỗi lần thực hiện, thì số ô đen tăng hoặc giảm một số chẵn. Ban đầu có 1 ô đen, như vậy, sau mỗi bước đổi màu, thì số lượng ô đen vẫn là một số lẻ. Như vậy, không thể đạt được trạng thái có 64 ô đen, 0 ô trắng hoặc 0 ô đen, 64 ô trắng. Ví dụ 2.1: Cho một bàn cờ kích thước 9 x 9, gồm 1 ô đen và 80 ô trắng. Thực hiện thuật toán : mỗi lần thay đổi màu tất cả các ô trên cùng 1 hàng hoặc 1 cột (ô đen thay bằng ô trắng và ngược lại). Hỏi có khi nào tất cả các ô trên bàn cờ cùng màu đen không? Nhận xét: Thuật toán tương tự như bài trước, tuy nhiên, mỗi lần thực hiện thuật toán sẽ đổi màu 9 ô , như vậy tính chẵn lẻ của số lượng ô đen không bất biến. Tuy nhiên, để ý kĩ một chút, ta hoàn toàn có thể đưa bài toán về bài toán 1: Xét hình vuông 8 x 8 ở 1 góc mà chứa ô đen. Khi đó, việc thực hiện thuật toán của đề bài sẽ : hoặc không thay đổi hình vuông 8x 8 này, hoặc thay màu tất cả các ô trên cùng một hàng hoặc một cột. Trường THPT Chuyên Thái Bình 120 HỘI CÁC TRƯỜNG THPT CHUYÊN KHU VỰC DUYÊN HẢI - ĐỒNG BẰNG BẮC BỘ HỘI THẢO KHOA HỌC LẦN THỨ VI Lời giải: Giả sử ô màu đen nằm ở phần hình vuông 8 x 8 được tô đậm trên hình vẽ. Khi đó, mỗi lần thực hiện thuật toán thì hoặc không làm thay đổi hình vuông 8 x8 này, hoặc sẽ đổi màu 1 hàng hoặc 1 cột của hình vuông 8 x 8 này. Như vậy, theo bài toán 1 thì số ô đen trong hình vuông 8 x 8 này luôn là số lẻ. Tức là không xảy ra trường hợp cả 64 ô đều màu đen. Vậy không xảy ra trường hợp cả bàn cờ màu đen Ví dụ 3.1. Hai người chơi cờ. Sau mỗi ván người thắng được 2 điểm, người thua được 0 điểm, nếu hòa thì mỗi người được 1 điểm. Hỏi sau một số ván liệu có thể xảy ra trường hợp một người được 7 điểm và người kia được 10 điểm được không? Lời giải: Gọi S(n) là tổng số điểm của cả hai người sau ván thứ n. Trường THPT Chuyên Thái Bình 121 HỘI CÁC TRƯỜNG THPT CHUYÊN KHU VỰC DUYÊN HẢI - ĐỒNG BẰNG BẮC BỘ HỘI THẢO KHOA HỌC LẦN THỨ VI Ta có S(n + 1) = S(n) + 2, ∀ n ≥ 0 Do đó, S(n) bất biến theo modun 2. Suy ra S(n) ≡ S(0) ≡ 0(mod 2), ∀ n ≥ 0 Vậy không thể xảy ra trường hợp một người được 7 điểm, một người được 10 điểm. Nhận xét: Tính bất biến ở đây là tính chẵn lẻ của tổng số điểm của hai ngươi chơi Ví dụ 4.1. Viết các số 1, 2, 3, …, 2014 lên bảng. Thực hiện thuật toán: Mỗi lần xóa đi hai số a, b bất kì và viết thêm số c = |a - b| . Chứng minh rằng số còn lại cuối cùng trên bảng là một số lẻ. Lời giải: Vì a + b + (a - b) = 2a nên a + b và a – b cùng tính chẵn, lẻ . Gọi S(n) là tổng các số trên bảng sau bước thứ n. Vì sau mỗi bước, tổng a + b được thay bởi c = |a - b| nên S(n) giữ nguyên tính chẵn, lẻ, hay bất biến theo modun 2. Mặt khác S(0) = 1 + 2 + … + 2014 = 1007. 2015 là số lẻ Nên S(n) là lẻ với mọi n. Vậy số cuối cùng còn lại trên bảng là một số lẻ Nhận xét: Tính bất biến ở đây là tính chẵn lẻ của tổng các số trên bảng Bài tập tương tự : Bài 1.1: Trên bảng, người ta viết 2013 số 0 và 2013 số 1. Thực hiện thuật toán: Mỗi lần số đi 2 số bất kì: nếu hai số đó giống nhau thì viết thêm số 0, nếu 2 số đó khác nhau thì viết thêm số 1. Hỏi số cuối cùng còn lại trên bảng là số nào? Bài 2.1: Trên bảng ta viết một số chữ số 0, một số chữ số 1 và một số chữ số 2. Sau đó ta cứ xóa đi một cặp hai chữ số khác nhau và thay thế vào đó một chữ số khác với hai chữ số đã xóa. Chứng minh rằng : Nếu trong kết quả cuối cùng trên Trường THPT Chuyên Thái Bình 122 HỘI CÁC TRƯỜNG THPT CHUYÊN KHU VỰC DUYÊN HẢI - ĐỒNG BẰNG BẮC BỘ HỘI THẢO KHOA HỌC LẦN THỨ VI bảng chỉ còn một chữ số thì kết quả đó không phụ thuộc vào thứ tự các cặp chữ số được xóa (Trích đề thi Vô địch Liên Xô - 1975) Bài 3.1: Cho dãy số 1, 2, 3, 4, …, 2013. Mỗi lần thay hai số a , b bởi số a – b. Hỏi có khi nào thu được toàn số 0 hay không? Thay 2013 bằng số tự nhiên N, tìm điều kiện của N để kết quả cuối cùng thu được toàn số 0? Bài 4.1: Cho một bàn cờ 7 x 7, tô đen 4 ô ở 4 góc. Thực hiện thuật toán: mỗi lần thay đổi màu 1 hàng hoặc một cột (ô đen thay bằng ô trắng và ngược lại). Hỏi có khi nào tất các ô trên bàn cờ cùng màu không? b) Bất biến liên quan đến tính chia hết cho một số lớn hơn 2 hoặc số dư trong một phép chia cho một số lớn hơn 2 Ví dụ 5.1. Cho 1000 số từ 1 đến 1000 trên bảng, mỗi lần thay 1 hoặc một vài số bởi tổng các chữ số của nó. Hỏi cuối cùng có nhiều số 1 hay nhiều số 2 hơn? Lời giải: Một số chia cho 9 dư k thì tổng các chữ số của nó chia 9 cũng dư k. Các số chia 9 dư 1 trong dãy từ 1 đến 1000 là : 1; 10; 19; 28; ….; 1000, như vậy có 1000 − 1 + 1 = 112 số 9 Các số chia 9 dư 2 trong dãy từ 1 đến 1000 là : 2; 11; 20; 29; ….; 992, như vậy có 992 − 2 + 1 = 111 số 9 Vậy, cuối cùng có nhiều số 1 hơn số 2. Nhận xét: trong lời giải trên ta sử dụng tính chất một số chia cho 9 dư k thì tổng các chữ số của nó chia 9 cũng dư k. Đây chính là đại lượng bất biến của bài toán. Ví dụ 6.1. (Đề thi chọn đội tuyển dự thi HSGQG tỉnh Bắc Ninh , năm 2007) Trường THPT Chuyên Thái Bình 123 HỘI CÁC TRƯỜNG THPT CHUYÊN KHU VỰC DUYÊN HẢI - ĐỒNG BẰNG BẮC BỘ HỘI THẢO KHOA HỌC LẦN THỨ VI Trên bàn có 2007 viên bi gồm 667 bi xanh, 669 bi đỏ, 671 bi vàng. Thực hiện thuật toán như sau : Mỗi lần lấy đi 2 viên bi khác màu và đặt thêm 2 viên bi có màu còn lại. Hỏi có thể nhận được trạng thái mà trên bàn chỉ còn lại các viên bi cùng màu được không? Lời giải: Gọi X(n), D(n) và V(n) tương ứng là số bi màu xanh, số bi màu đỏ và số bi màu vàng sau bước thứ n. Xét các đại lượng X(n) – D(n), D(n) – V(n) , V(n) – X(n) Vì mỗi lần lấy đi 2 viên bi khác màu và đặt thêm 2 viên bi có màu còn lại nên các đại lượng X(n) – D(n), D(n) – V(n) , V(n) – X(n) bất biến theo mođun 3. Ta có X(0) – D(0) = - 2 , D(0) – V(0) = 2; V(0) – X(0)= 4 Nên số dư của các đại lượng này khi chia cho 3 sau mỗi lần thực hiện thuật toán là 1; 2; 1. Ta lại thấy, số bi ở trên bàn luôn không thay đổi (mỗi lần thực hiện, lấy đi 2 viên và đặt thêm 2 viên khác), vậy nếu trên bàn chỉ còn các viên bi cùng màu tức là số viên bi đỏ, xanh, vàng còn lại là một hoán vị của tập {2007, 0, 0}, vậy các đại lượng X(n) – D(n), D(n) – V(n) , V(n) – X(n) đều chia hết cho 3 (mâu thuẫn) Từ đó, suy ra không thể nhận được trạng thái mà trên bàn chỉ còn lại các viên bi cùng màu Nhận xét: Việc lấy thêm đi 2 viên bi khác nhau và đặt thêm 2 viên bi có màu còn lại tạo ra bất biến về hiệu số các viên bi khi chia cho 3 ở bài toán. Ví dụ 7.1. Mỗi bước cho phép chọn 1 số a, phân tích a thành tích hai số m, n và viết lên bảng m ± 2, n ± 2 tùy ý (ví dụ a = 99 = 9.11, 9 – 2 = 7, 11 + 2 = 13, như vậy có thể viết lên bảng số 7 và số 13 thay cho số 99). Hỏi sau một số bước như vậy,từ số 99…..99 (2012 chữ số 9) có thu được trên bảng một dãy gồm toàn các số 9 không? Trường THPT Chuyên Thái Bình 124 HỘI CÁC TRƯỜNG THPT CHUYÊN KHU VỰC DUYÊN HẢI - ĐỒNG BẰNG BẮC BỘ HỘI THẢO KHOA HỌC LẦN THỨ VI Lời giải: Nếu a là một số chia cho 4 dư 3 thì trong 2 số m, n có một số chia 4 dư 1, một số chia 4 dư 3. Một số chia 4 dư 1 thì cộng hay trừ 2 đều chia 4 dư 3. Vậy, sau khi thực hiện phép thay đổi thì từ 1 số chia 4 dư 3 luôn tồn tại 1 số chia 4 dư 3 còn lại. Số 99…..99 (2012 chữ số 9) chia 4 dư 3, như vậy, sau một số bước biến đổi được chỉ ra ở đề bài thì cuối cùng, luôn tồn tại một số chia 4 dư 3 Một dãy gồm toàn số 9, tức là dãy gồm toàn số chia 4 dư 1. Vậy không thể thu được dãy số như trên. Nhận xét: Bất biến trong bài toán là sự tồn tại một số chia 4 dư 3 trong dãy Ví dụ 8.1. Trên bảng có hai số 1 và 2. Thực hiện việc ghi số theo quy tắc sau: nếu trên bảng có hai số a, b thì được phép ghi thêm số c = a + b + ab. Hỏi bằng cách đó, có thể ghi được các số 2010 và 11111 hay không? Lời giải: Dãy các số được viết là 1, 2, 5, 11, 17,… Dễ dàng chứng minh được các số được viết thêm trên bảng đều chia cho 3 dư 2. Bất biến trên cho phép ta loại trừ được số 2010 trong dãy các số được viết trên bảng. Tuy nhiên, bất biến đó không cho phép ta loại trừ số 11111. Ta đi tìm một bất biến khác. Quan sát các số được viết và quy tắc viết thêm số, ta có c = a + b + ab ⇒ c + 1 = (a + 1)(b + 1) Và nếu cộng thêm 1 vào các số thuộc dãy trên, ta có dãy mới 2, 3, 6, 12, 18, … Như vậy, nếu cộng thêm 1 vào các số viết thêm thì các số này đều có n m dạng 2.3 với n, m ∈ Ν . Do 11111 + 1 = 11112 = 3. 8. 463 nên không thuộc dãy các số viết được. Trường THPT Chuyên Thái Bình 125 HỘI CÁC TRƯỜNG THPT CHUYÊN KHU VỰC DUYÊN HẢI - ĐỒNG BẰNG BẮC BỘ HỘI THẢO KHOA HỌC LẦN THỨ VI Do đó không thể viết được các số 2010 và 11111. Nhận xét: Bài toán trên sử dụng 2 bất biến Bài tập tương tự Bài 5.1: Các số tự nhiên 0, 1, 2, 3, … được viết trong các ô của một bảng ô vuông kích thước 2003 x 2003 theo vòng xoáy trôn ốc (xoáy ngược chiều kim đồng hồ) sao cho số 0 nằm ở ô trung tâm (tâm của bảng). Các dòng và cột của bảng được đánh số tăng dần từ dưới lên trên và từ trái sang phải (bắt đầu từ số 1). a) Số 2004 nằm ở dòng nào, cột nào? Tại sao? b) Thực hiện thuật toán sau : lần đầu tiên, thay số 0 ở ô trung tâm bởi 1998; mỗi lần tiếp theo, cho phép lấy ra 12 số trong 12 ô liên tiếp trong cùng một hàng hoặc trong cùng một cột hoặc trong cùng một hình chữ nhật 3 x 4 rồi tăng mỗi số đó lên một đơn vị. Hỏi sau một số lần như vậy ta có thể làm cho tất cả các số trong bảng đều là bội của 2004 hay không? Tại sao? 20 19 18 17 16 21 6 5 4 15 22 7 0 3 14 23 8 1 2 13 24 9 10 11 12 Bài 6.1 : Ở xứ sở nọ, nàng công chúa bị một con rồng hung hãn 100 đầu bắt đi. Chàng hoàng tử lên đường đi cứu công chúa, chàng có 2 thanh kiếm, thanh 1 chặt được 21 đầu rồng, thanh 2 : chặt được 3 đầu rồng nhưng rồng lại mọc thêm 2012 đầu. Nếu hoàng tử chặt được hết đầu của rồng thì cứu được Trường THPT Chuyên Thái Bình 126 HỘI CÁC TRƯỜNG THPT CHUYÊN KHU VỰC DUYÊN HẢI - ĐỒNG BẰNG BẮC BỘ HỘI THẢO KHOA HỌC LẦN THỨ VI công chúa. Hỏi hoàng tử có cứu được công chúa không? Nếu số lượng đầu rồng ban đầu là N, N thỏa mãn điều kiện gì thì hoàng tử cứu được công chúa? 2. 2. Bất biến là công thức đại số Ví dụ 1.2. Trên bảng người ta viết các số tự nhiên liên tiếp từ 1 đến 2013 sau đó thực hiện trò chơi như sau: mỗi lần xóa hai số bất kì và viết một số mới bằng tổng hai số đã xóa. Việc làm này thực hiện liên tục cho đến khi còn một số trên bảng. Hỏi số cuối cùng còn lại trên bảng là bao nhiêu? Tại sao? Lời giải: Vì mỗi lần thực hiện trò chơi thì thay hai số bằng tổng của chúng nên số lượng số trên bảng giảm đi 1 và tổng các số trên bảng không thay đổi trong mọi thời điểm. Như vậy, sau 2012 lần thực hiện thì trên bảng còn 1 số. các Tổng 1+ 2 + 3 + + 2012 + 2013 = số lúc đầu là: 2013.(2013 + 1) = 2027091 2 Vậy số cuối cùng còn lại trên bảng là 2027091 Nhận xét: Bất biến trong bài toán trên là tổng của các số trên bảng không thay đổi sau thuật toán. Ví dụ 2.2. Một dãy gồm có 19 phòng. Ban đầu mỗi phòng có một người. Sau đó, cứ mỗi ngày có hai người nào đó chuyển sang hai phòng bên cạnh nhưng theo hai chiều ngược nhau, Hỏi sau một số ngày, có hay không trường hợp mà: (a) Không có ai ở phòng có thứ tự chẵn (b) Có 10 người ở phòng cuối. Lời giải: Đánh số các phòng theo thứ tự từ 1 đến 19 1 2 3 4 5 6 7 8 Trường THPT Chuyên Thái Bình 9 10 11 12 13 14 15 16 17 18 19 127 HỘI CÁC TRƯỜNG THPT CHUYÊN KHU VỰC DUYÊN HẢI - ĐỒNG BẰNG BẮC BỘ HỘI THẢO KHOA HỌC LẦN THỨ VI Ta cho mỗi vị khách một thẻ ghi số phòng mình đang ở. Gọi S(n) là tổng các số ghi trên thẻ của tất cả các vị khách trong ngày thứ n. Vì mỗi ngày có hai người nào đó chuyển sang hai phòng bên cạnh nhưng theo hai chiều ngược nhau nên S(n) không hề thay đổi. Vậy S(n) = S(1) = 1 + 2 + 3 + …+ 19 = 190, ∀ n ≥ 1 a) Vì có lẻ người nên nếu không ai ở phòng có thứ tự chẵn thì S(n) là tổng của 19 số lẻ, tức là S(n) là số lẻ , mâu thuẫn. Vậy trường hợp này không xảy ra. b) Nếu có 10 người ở phòng cuối (phòng 19) thì S(n) > 19. 10 = 190, mâu thuẫn.Vậy trường hợp này cũng không xảy ra Nhận xét: Bất biến của bài toán là tổng các số ghi trên thẻ của tất cả các vị khách. Ví dụ 3.2. Xét một bảng vuông 4 x 4 ô. Tại mỗi ô của bảng vuông có chứa dấu “+” hoặc dấu “ - ”. Mỗi lần thực hiện, cho phép đổi dấu của tất cả các ô trên cùng một hàng hoặc cùng một cột. Giả sử bảng hình vuông ban đầu có 1 dấu “+” và 15 dấu “-”. Hỏi có thể đưa bảng ban đầu về bảng có toàn dấu “+” được không? Lời giải: Thay tất cả các dấu “+” bằng 1 và dấu “ - ” bằng – 1. Mỗi lần thực hiện đổi dấu tất cả các ô trên cùng 1 hàng hoặc 1 cột, tức là đổi dấu của 4 số nên tích của 16 số trên bảng là không thay đổi. Tích của 16 số ban đầu là – 1, sau mỗi lần biến đổi vẫn là -1. Nếu bảng toàn dấu “+” tức là tích của 16 số trên bảng là 1. Như vậy, dù có dù có thực hiện bao nhiêu lần thì từ bảng vuông ban đầu không thể đưa về bảng vuông toàn dấu “+”. Nhận xét: Bất biến trong bài toán là tích tất cả các số trên các ô của bảng. Trường THPT Chuyên Thái Bình 128 HỘI CÁC TRƯỜNG THPT CHUYÊN KHU VỰC DUYÊN HẢI - ĐỒNG BẰNG BẮC BỘ HỘI THẢO KHOA HỌC LẦN THỨ VI Ví dụ 4.2. Trên bảng có các số 1 2 3 80 ; ; ; ; . Mỗi lần thực hiện, cho phép xóa 80 80 80 80 đi hai số a, b bất kì và thay bởi a + b – 2ab. Hỏi sau 1987 lần thực hiện phép xóa, số còn lại trên bảng là số nào? Lời giải Giả sử các số trên là a1; a2; a3; …; ak. Xét tích P = (2a1 - 1)(2a2 - 1)…(2ak - 1) Khi đó, sau mỗi lần biến đổi, tích trên bị mất đi hai thừa số (2a - 1)(2b 1) và được thêm vào thừa số 2(a + b – 2ab) – 1 = - (2a - 1)(2b – 1) Tức là sau mỗi lần biến đổi giá trị tuyệt đối của tích P là không thay đổi. Vì tích ban đầu bằng 0 (do bảng ban đầu có chứa số 40 1 = ) nên sau mỗi 80 2 lần biến đổi thì tích này luôn bằng 0. 1 Vậy số còn lại cuối cùng trên bảng là s thỏa mãn 2s – 1 = 0, hay s = . 2 Nhận xét: Bất biến trong bài toán là tích tất cả các giá trị của hàm số y = 2x – 1 tại các số trên bảng. Ví dụ 5.2. Cho đa thức P(x) = ax2 + bx + c, có thể thực hiện một trong hai phép biến đổi: a) Đổi chỗ a và c b) Đổi biến x bởi x + t với t ∈ R. Hỏi từ x 2 – 31x - 3 có thể thu được x2 – 20x – 12 không? Tìm mối liên hệ của hai đa thức P(x) và Q(x) sao cho từ đa thức này có thể thu được đa thức kia bởi hai phép biến đổi nói trên. Lời giải. Xét biểu thức P(x+t) = a(x + t)2 + b(x + t) + c = ax2 + (2at + b) x + at2 + c Trường THPT Chuyên Thái Bình 129 HỘI CÁC TRƯỜNG THPT CHUYÊN KHU VỰC DUYÊN HẢI - ĐỒNG BẰNG BẮC BỘ HỘI THẢO KHOA HỌC LẦN THỨ VI có ∆ = (2at + b)2 – 4a(at2 + c) = 4a2t2 + 4abt + b2 – 4a2t2 – 4ac = b2 – 4ac Xét biểu thức P1(x) = cx2 + bx + a có ∆ = b2 – 4ac Như vậy, cả 2 phép biến đổi trên không làm thay đổi đại lượng ∆ ( ∆ bất biến với hai phép biến đổi) Xét P(x) = x 2 – 31x – 3 có ∆ = 312 + 4. 3 = 973 Q(x) = x2 – 20x – 12 có ∆ = 202 + 4. 12 = 448 Vậy từ P(x) ta không thể thu được Q(x) thông qua hai phép biến đổi trên. Hai đa thức P(x) và Q(x) mà từ đa thức này có thể thu được đa thức kia bởi hai phép biến đổi nói trên khi chúng có giá trị của biệt thức Nhận xét: đại lượng bất biến là biệt thức ∆ bằng nhau. ∆ Bài tập tương tự: Bài 1.2: Tại mỗi đỉnh của đa giác lồi A1A2…A1993 ta ghi một dấu “+” hoặc một dấu “-”sao cho trong 1993 dấu đó có cả dấu “+” và dấu “-”. Thực hiện việc thay dấu như sau: mỗi lần, thay dấu đồng thời tại tất cả các đỉnh của đa giác theo quy tắc: - Nếu dấu tại Ai và Ai+1 là như nhau thì dấu tại Ai được thay là dấu “+” - Nếu dấu tại Ai và Ai+1 là khác nhau thì dấu tại Ai được thay là dấu “+” (Quy ước A1994 là A1) Chứng minh rằng, tồn tại số nguyên k ≥ 2 sao cho khi thực hiện liên tiếp k lần phép thay dấu nói trên, ta được đa giác A1A2…A1993 mà dấu tại mỗi đỉnh Ai ( i = 1,1993 ) trùng với dấu của đỉnh đó ngay sau lần thay dấu thứ nhất Bài 2.2: Trên bảng cho đa thức f (x) = x 2 + 4x + 3 . Thực hiện trò chơi sau, nếu trên bảng đã có đa thức P(x) thì được phép viết thêm lên bảng một trong hai đa thức sau 1   1  Q(x) = x 2 f  + 1 ,R(x) = (x − 1)2 .f   x   x −1 Hỏi sau một số bước ta có thể viết được đa thức g(x) = x 2 + 10x + 9 Trường THPT Chuyên Thái Bình 130 HỘI CÁC TRƯỜNG THPT CHUYÊN KHU VỰC DUYÊN HẢI - ĐỒNG BẰNG BẮC BỘ HỘI THẢO KHOA HỌC LẦN THỨ VI Bài 3.2: Cho n cái cây, mỗi cây có một con chim đậu. Thực hiện thuật toán: Mỗi lần, một con bay theo chiều xuôi qua a cây thì sẽ có một con bay theo chiều ngược qua a cây. Hỏi có khi nào cả n con chim đều đậu trên 1 cây không? Bài 4.2: Cho dãy số : 1, 2, 3, …, 2013. Mỗi bước thay hai số a, b bởi a.b + a + b. Hỏi sau 2012 bước, số còn lại là số nào? 2. 3. Bài toán tô màu Ví dụ 1.3. Bàn cờ vua 8 x 8 bị mất hai ô ở hai góc đối diện. Hỏi có thể lát phần còn lại của bàn cờ bởi các quân Domino 2 x 1 được không? Lời giải: Mỗi quân Domino lát vào bàn cờ luôn chiếm một ô trắng và một ô đen. Do đó, nếu lát được phần còn lại của bàn cờ thì số ô trắng và số ô đen bằng nhau. Nhưng do hai ô ở đối diện của bàn cờ là hai ô cùng màu nên số ô màu trắng và số ô màu đen trong phần còn lại của bàn cờ không bằng nhau. Vậy không lát được phần còn lại của bàn cờ bằng các quân Domino. Trường THPT Chuyên Thái Bình 131 HỘI CÁC TRƯỜNG THPT CHUYÊN KHU VỰC DUYÊN HẢI - ĐỒNG BẰNG BẮC BỘ HỘI THẢO KHOA HỌC LẦN THỨ VI Nhận xét: Tô các ô của bàn cờ đan xen bằng 2 màu, xét số lượng các ô của từng màu Ví dụ 2.3 : Cho bàn cờ 10 x 10. Có thể sử dụng các quân 1 x 4 để lát kín bàn cờ được không? Lời giải: Ghi số trên bàn cờ như sau : 1 2 3 4 1 2 3 4 1 2 2 3 4 1 2 3 4 1 2 3 3 4 1 2 3 4 1 2 3 4 4 1 2 3 4 1 2 3 4 1 1 2 3 4 1 2 3 4 1 2 2 3 4 1 2 3 4 1 2 3 3 4 1 2 3 4 1 2 3 4 4 1 2 3 4 1 2 3 4 1 1 2 3 4 1 2 3 4 1 2 2 3 4 1 2 3 4 1 2 3 Ta tô bàn cờ bởi 4 màu, các ô ghi số 1 được tô màu đỏ, ô ghi số 2 được tô màu xanh, ô ghi số 3 được tô màu vàng, ô ghi số 4 được tô màu đen. Như vậy có 25 ô đỏ, 26 ô xanh, 25 ô vàng, 24 ô đen. Khi lát mỗi quân 1 x 4 lên bàn cờ thì mỗi quân ấy sẽ chiếm 1 ô đỏ, 1 ô xanh, 1 ô vàng và 1 ô đen. Như vậy số lượng các ô đỏ, xanh, vàng, đen sau khi Trường THPT Chuyên Thái Bình 132 HỘI CÁC TRƯỜNG THPT CHUYÊN KHU VỰC DUYÊN HẢI - ĐỒNG BẰNG BẮC BỘ HỘI THẢO KHOA HỌC LẦN THỨ VI phủ bởi các quân 1 x 4 lên bàn cờ là bằng nhau. Do đó, không thể phủ kín được bàn cờ. Nhận xét: Tô các ô bằng 4 màu khác nhau và nhận xét số lượng ô của từng màu Ví dụ 3.3. Cho một bảng ô vuông kích thước kích thước 8 x 8 được điền số như sau: 2013 2 3 4 5 6 7 8 9 10 11 12 13 14 15 16 17 18 19 20 21 22 23 24 25 26 27 28 29 30 31 32 33 34 35 36 37 38 39 40 41 42 43 44 45 46 47 48 49 50 51 52 53 54 55 56 57 58 59 60 61 62 63 64 Cho phép thực hiện việc thay đổi các số trong bảng theo quy tắc: Mỗi lần lấy tất cả các số nằm trong hình vuông kích thước 3 x 3 hoặc 4 x 4 rồi tăng mỗi số lên 1 đơn vị. Hỏi khẳng định sau đúng hay sai ? Với mỗi cách điền số ban đầu, nhờ việc thực hiện liên tiếp phép thay số nói trên đối với bảng số ban đầu ta sẽ nhận được bảng 8 x 8 mà ở mỗi ô vuông con của bảng đó là một số chia hết cho 3. Trường THPT Chuyên Thái Bình 133 HỘI CÁC TRƯỜNG THPT CHUYÊN KHU VỰC DUYÊN HẢI - ĐỒNG BẰNG BẮC BỘ HỘI THẢO KHOA HỌC LẦN THỨ VI Lời giải: Tô màu bảng như hình vẽ. Nhận xét: Bất kể hình vuông con 3 x 3 nào cũng đều chứa đúng 6 ô vuông đen hoặc 9 ô vuông đen. Bất kể hình vuông con 4 x 4 nào cũng đều chứa đúng 12 ô đen. Do đó, sau mỗi lần thay số, ta không làm thay đổi số dư trong phép chia cho 3 của tổng các số trong các ô của phần gạch chéo. Tổng các số trong các ô ở phần bôi đen ở trên chia 3 dư 2, nên tổng các số sau các bước biến đổi cũng chia 3 dư 2, tức là không thể đạt được trạng thái mà số trong mọi ô vuông con đều chia hết cho 3. Ví dụ 4.3. Một hình tròn được chia thành 2010 hình quạt. Trong mỗi hình quạt có một viên bi. Thực hiện trò chơi sau: mỗi lần cho phép lấy ra hai viên bi trong hai hình quạt nào đó và chuyển chúng sang các ô bên cạnh nhưng theo hai chiều ngược nhau. Hỏi sau một số lần có thể chuyển hết các viên bi vào một hình quạt được không? Trường THPT Chuyên Thái Bình 134 HỘI CÁC TRƯỜNG THPT CHUYÊN KHU VỰC DUYÊN HẢI - ĐỒNG BẰNG BẮC BỘ HỘI THẢO KHOA HỌC LẦN THỨ VI Lời giải: Tô màu các hình quạt bởi hai màu đen, trắng như hình vẽ sao cho hình quạt kề nhau thì khác màu. Gọi S(n) và T(n) tương ứng là số viên bi trong các hình quạt màu đen và số viên bi trong các hình quạt màu trắng sau bước chuyển bi thứ n. Ta có S(n) và T(n) bất biến theo modun 2. Do S(0) = T(0) = 1005 nên S(n) và T(n) lẻ với mọi n. Do đó không thể có trạng thái mà tất cả các viên bi ở trong cùng một hình quạt Ví dụ 5.3. Hình tròn được chia thành 2011 hình dẻ quạt. Xếp 2012 viên kẹo vào các phần dẻ quạt. Mỗi bước, cho phép chuyển hai viên ở cùng một phần sang hai phần kề khác hướng. Chứng minh rằng đến lúc nào đó có ít nhất 1006 phần có chứa kẹo. Lời giải: Nhận xét 1: Quá trình trên là không dừng lại, vì việc thực hiện mỗi bước không làm thay đổi số viên kẹo ban đầu. Có 2011 hình dẻ quạt, 2012 viên kẹo, nên sau mỗi bước luôn tồn tại một hình dẻ quạt có lớn hơn hoặc bằng 2 viên kẹo (nguyên lí đirichle), nói cách khác, sau mỗi bước thì luôn có thể thực hiện được bước tiếp theo. Nhận xét 2: đến một lúc nào đó, 2 phần cạnh nhau bất kì có kẹo (ít nhất là một ô có kẹo) Giả sử điều này không đúng, tức là tồn tại hai hình dẻ quạt kề nhau không bao giờ có kẹo. Trường THPT Chuyên Thái Bình 135 HỘI CÁC TRƯỜNG THPT CHUYÊN KHU VỰC DUYÊN HẢI - ĐỒNG BẰNG BẮC BỘ HỘI THẢO KHOA HỌC LẦN THỨ VI Bỏ 2 hình dẻ quạt này đi, có thể coi 2009 hình dẻ quạt còn lại là một chuỗi 2009 ô hình chữ nhật thẳng hàng. Như vậy quá trình này dừng lại được. Vậy điều giả sử là sai. Tức là đến một lúc nào đó 2 phần cạnh nhau bất kì thì có kẹo. Lúc này, chia 2011 ô thành 1006 phần như sau: 1 ô có kẹo, 1005 cặp ô kề nhau. Khi đó, mỗi cặp ô sẽ có ít nhất 1 ô có kẹo, như vậy sẽ có ít nhất 1006 ô có kẹo. Nhận xét 3: 2 ô kề nhau có kẹo sau bước nhảy vẫn luôn có kẹo. Xét 2 ô kề nhau có kẹo, nếu lấy kẹo từ 2009 ô còn lại để thực hiện phép nhảy thì 2 ô này vẫn luôn có kẹo, nếu lấy kẹo từ 1 trong 2 ô này để thực hiện phép nhảy, vì lấy 2 viên kẹo và chuyển sang 2 phần bên nó, nên sẽ chắc chắn sẽ chuyển kẹo vào ô còn lại. Vậy trong mọi tình huống, 2 ô này luôn có kẹo Ví dụ 6.3. Điền 29 số nguyên dương đầu tiên vào các ô vuông con của bảng 6 x 5 như sau (bảng 1 ): 1 6 2 3 4 5 7 8 9 10 13 14 11 12 15 16 17 18 19 20 21 22 23 24 25 26 27 28 29 Cho phép đổi vị trí các số trong bảng theo quy tắc : Mỗi lần, lấy 1 số nằm ở ô kề với ô trống rồi chuyển số đó sang ô trống. Hỏi nhờ việc thực hiện liên tiếp một số hữu hạn lần phép chuyển số nói trên đối với bảng ban đầu, ta có thể nhận được bảng số sau (bảng 2) không? Trường THPT Chuyên Thái Bình 136 HỘI CÁC TRƯỜNG THPT CHUYÊN KHU VỰC DUYÊN HẢI - ĐỒNG BẰNG BẮC BỘ HỘI THẢO KHOA HỌC LẦN THỨ VI 29 2 3 4 5 6 7 8 9 10 11 12 13 14 15 16 17 18 19 20 21 22 23 24 25 26 27 28 29 Lời giải: Giả sử nhờ phép chuyển số theo quy tắc của đề bài, từ bảng 1 ta có thể nhận được bảng 2.(*) Ta coi ô trống của mỗi bảng là ô được điền số 0. Với mỗi bảng số nhận được trong quá trình chuyển số, ta liệt kê tất cả các số trong bảng theo thứ tự từ trái qua phải, từ trên xuống dưới. Khi đó, ứng với mỗi bảng số ta sẽ có một hoán vị của 30 số tự nhiên đầu tiên. Và do đó, từ giả thiết (*) cho thấy, từ hoán vị (1, 2, 3, 4, 5, 6, 7, 8, 9, 10, 11, 12, 0, 13, 14, 15, 16, 17, 18, 19, 20, 21, 22, 23, 24, 25, 26, 27, 28, 29 ) (gọi là hoán vị I) ta có thể nhận được hoán vị (29, 2, 3, 4, 5, 6, 7, 8, 9, 10, 11, 12, 0, 13, 14, 15, 16, 17, 18, 19, 20, 21, 22, 23, 24, 25, 26, 27, 28, 1 ) (gọi là hoán vị 2) nhờ việc thực hiện liên tiếp 1 số hữu hạn lần phép đổi chỗ các số hạng trong hoán vị theo quy tắc : Mỗi lần, lấy một số khác 0 của hoán vị rồi đổi vị trí của số hạng đó và 0 cho nhau. (1) Giả sử (a1, a2, …, a30) là một hoán vị của 30 số tự nhiên đầu tiên. Ta gọi cặp số (ai, aj) là cặp số ngược của hoán vị vừa nêu nếu ai > aj và i < j . Dễ thấy, sau mỗi lần thực hiện phép đổi chỗ các số hạng theo quy tắc (1) đối với hoán vị (a1, a2, …, a30) thì số cặp số ngược của hoán vị sẽ tăng hoặc giảm một số lẻ đơn vị.(2) Ta có số cặp số ngược của hoán vị I là 12, số cặp số ngược của hoán vị II là 67 . Từ đó kết hợp với (2) suy ra từ hoán vị I ta chỉ có thể nhận được hoán vị II sau 1 số lẻ lần thực hiện phép đổi chỗ các số hạng. Điều này cho thấy, nếu từ bảng 1 ta nhận được bảng 2 thì số lần chuyển số phải là số lẻ. (3) Trường THPT Chuyên Thái Bình 137 HỘI CÁC TRƯỜNG THPT CHUYÊN KHU VỰC DUYÊN HẢI - ĐỒNG BẰNG BẮC BỘ HỘI THẢO KHOA HỌC LẦN THỨ VI Tô màu tất cả các ô vuông con của bảng 6 x 5 bởi 2 màu xanh, đỏ sao cho 2 ô kề nhau có 2 màu khác nhau. Thế thì, sau mỗi lần chuyển số, số 0 sẽ được chuyển từ ô có màu này sang ô có màu kia. Và vì thế, do số 0 ở bảng 1 và số 0 ở bảng 2 nằm ở hai ô có màu giống nhau nên từ bảng 1 chỉ có thể nhận được bảng 2 sau một số chẵn lần chuyển số. Điều này mâu thuẫn với (3). Vậy từ bảng (1) ta không thể nhận được bảng 2 nhờ phép chuyển số theo quy tắc của đề bài. Nhận xét: Nhờ việc tô màu các ô vuông con trong bảng, ta tìm được số lần chuyển số là một số chẵn. Từ các suy luận về cặp số ngược, ta tìm được số lần chuyển số là một số lẻ. Từ đó có được kết quả của bài toán. Trong bài toán này, phát hiện và sử dụng cặp số ngược có vai trò quan trọng. Cặp số ngược cũng được sử dụng trong bài toán sau: Ví dụ 7.3. Ở các vị trí khác nhau của một đường đua ô tô vòng tròn cùng một thời gian có 25 ô tô xuất phát theo cùng một hướng. Theo thể lệ cuộc đua, các ô tô có thể vượt lẫn nhau, nhưng cấm không được vượt đồng thời hai xe cùng một lúc. Các ô tô đến đích là các điểm mà chúng xuất phát ban đầu cùng một lúc. Chứng minh rằng trong suốt cuộc đua có một số chẵn lần vượt nhau của các ô tô. Lời giải: Ta sơn 1 trong số 25 ô tô thành màu vàng, còn các ô tô khác được đánh số thứ tự là 1, 2, 3, …, 24 theo thứ tự mà chúng ở thời điểm ban đầu sau ô tô màu vàng (theo chiều chuyển động của các ô tô). Ở tâm của đường đua ta sẽ đặt một cái bảng để ghi số thứ tự của các ô tô sắp xếp sau ô tô vàng sau mỗi lần các ô tô vượt nhau, tức là ta được một hoán vị của {1, 2, …, 24}. Trường hợp 1: Mỗi lần 2 ô tô trong các ô tô từ 1 đến 24 vượt nhau thì trên bảng sẽ có 2 số liền nhau đổi chỗ cho nhau. Trường hợp 2: Nếu trước khi có lần vượt của một ô tô nào với ô tô vàng , các số trên bảng lập thành một hoán vị a1, a2, …, a24 thì sau lần vượt đó sẽ có Trường THPT Chuyên Thái Bình 138 HỘI CÁC TRƯỜNG THPT CHUYÊN KHU VỰC DUYÊN HẢI - ĐỒNG BẰNG BẮC BỘ HỘI THẢO KHOA HỌC LẦN THỨ VI hoán vị a2, a3, …, a24, a1. Từ hoán vị trên có thể chuyển xuống hoán vị dưới bằng 23 phép chuyển vị , tức là phép đổi chỗ 2 số đứng liền nhau. Trường hợp 3: Nếu ô tô vàng vượt một ô tô nào đó thì từ hoán vị a1, a2, …, a24 ta có hoán vị a24, a1, a2, …, a23 . Lần di chuyển này cũng có thể thay bằng 23 phép chuyển vị như trường hợp 2. Như vậy, mỗi lần các ô tô vượt nhau, đều dẫn đến việc thực hiện một số lẻ lần phép chuyển vị. Ta sẽ chứng tỏ nếu số lần vượt nhau là số lẻ thì khi về đích các ô tô không được sắp xếp như cũ. Thật vậy, giả sử a1, a2, …, a24 là một cách sắp xếp tùy ý của các số 1, 2, …, 24. Ta sẽ nói rằng các số ai, aj lập thành một nghịch thế nếu i < j nhưng ai > aj. Khi đổi vị trí của 2 số đứng liền nhau, tức là thực hiện một phép chuyển vị thì sẽ tăng hay giảm số nghịch thế đi 1. Do đó nếu các ô tô vượt nhau một số lẻ lần thì từ cách sắp xếp thứ tự của các ô tô ban đầu, đến cuối cùng ta đã thực hiện 1 số lẻ các phép chuyển vị, tức là số nghịch thế của lần sắp xếp cuối cùng là lẻ, nghĩa là các ô tô không thể sắp xếp như cũ. Mâu thuẫn Vậy các ô tô vượt nhau một số chẵn lần. Bài tập tương tự: Bài 1.3 : Xét bàn cờ vua 8 x 8 . Chứng minh rằng nếu xuất phát từ một ô góc, con mã không thể đi qua tất cả các ô của bàn cờ, mỗi ô một lần và kết thúc ở ô góc đối diện với ô góc nó xuất phát. Bài 2.3 : Xác định các số nguyên dương m, n sao cho bảng m x n có thể lát được bởi các quân hình chữ L dưới đây Trường THPT Chuyên Thái Bình 139 HỘI CÁC TRƯỜNG THPT CHUYÊN KHU VỰC DUYÊN HẢI - ĐỒNG BẰNG BẮC BỘ HỘI THẢO KHOA HỌC LẦN THỨ VI Bài 3.3 .(IMO - 2004).Ta định nghĩa viên gạch hình móc câu là hình gồm 6 ô vuông đơn vị như hình vẽ dưới đây, hoặc hình nhận được do lật hình đó (sang trái, sang phải, lên trên, xuống dưới) hoặc hình nhận được do xoay hình đó đi một góc. Hãy xác định tất cả các hình chữ nhật m x n, trong đó m, n là các số nguyên dương sao cho có thể lát hình chữ nhật đó bằng các viên gạch hình móc câu. Bài 4.3(VMO - 1991).Cho bảng 1991 x 1992. Kí hiệu (m, n) là ô vuông nằm ở giao của hàng thứ m và cột thứ n. Tô màu các ô vuông của bảng theo quy tắc sau: lần thứ nhất tô 3 ô (r, s), (r + 1, s + 1), (r + 2, s + 2); 1 ≤ r ≤ 1989, 1 ≤ s ≤ 1990 , từ lần thứ hai, mỗi lần tô đúng ba ô chưa có màu nằm cạnh nhau trong cùng một hàng hoặc cùng một cột. Hỏi bằng cách nào đó có thể tô màu được tất cả các ô của bảng được không? Bài 5.3 (VMO - 2006).Xét bảng ô vuông m x n (m, n là các số nguyên dương lớn hơn 3). Thực hiện trò chơi sau : mỗi lần đặt 4 viên bi vào 4 ô của bảng (mỗi ô một viên bi) mà 4 ô đó tạo thành một trong những hình dưới đây Hỏi sau một số lần ta có thể nhận được bảng mà số bi trong các ô bằng nhau được không nếu a) m = 2004 và n = 2006 ? Trường THPT Chuyên Thái Bình 140 HỘI CÁC TRƯỜNG THPT CHUYÊN KHU VỰC DUYÊN HẢI - ĐỒNG BẰNG BẮC BỘ HỘI THẢO KHOA HỌC LẦN THỨ VI b) m = 2005 và n = 2006 ? Bài 6.3: Cho n ( n ≥ 2) học sinh đứng thành hàng dọc. Sau mỗi lần cô giáo thổi còi, có 2 em đổi chỗ cho nhau. Hỏi sau một số lẻ lần thổi còi, ta có thể thấy tất cả các em học sinh đều đứng ở vị trí ban đầu của mình hay không? 3. SỬ DỤNG ĐẠI LƯỢNG ĐƠN BIẾN ĐỂ GIẢI BÀI TOÁN TỔ HỢP Ví dụ 1.4 Xét bảng ô vuông m x n (m, n ≥ 2 ). Trong mỗi ô của bảng ta điền một số thực. Thực hiện thuật toán như sau: mỗi lần lấy ra một hàng hoặc một cột có tổng các số nhỏ hơn 0 và đổi dấu tất cả các số trong hàng (hoặc cột) đó. Chứng minh rằng sau hữu hạn bước ta nhận được bảng mà tổng các số mỗi hàng và tổng các số trong mỗi cột là số không âm. Lời giải: Gọi S(n) là tổng tất cả các số trong bảng sau bước thứ n. Ta có, S(n+1)> S(n), ∀ n > 0 . Do đó, S(n) là một hàm đơn biến. Mặt khác, số trạng thái có thể nhận được là hữu hạn nên chỉ có thể thực hiện thuật toán hữu hạn lần và ta nhận được bảng thỏa mãn yêu cầu bài toán. Ví dụ 2.4 Cho một dãy phòng dài vô hạn, được đánh số 1, 2, 3, … Có một số hữu hạn người sống trong dãy phòng. Mỗi ngày có hai người sống ở hai phòng cạnh nhau chuyển sang hai phòng khác nhau theo hai hướng ngược nhau nhưng không được tráo đổi vị trí cho nhau. Chứng minh rằng việc chuyển phòng đó dừng lại sau hữu hạn ngày. Lời giải: Ta đưa cho mỗi người một chìa khóa, trên đó có ghi số phòng của mình đang ở. Gọi S(n) là tích các số viết trên các chìa khóa ở ngày thứ n. Ta có: S(n + 1) k(k + 1) = < 1, ∀n ≥ 1 S(n) (k − 1)(k + 2) Trường THPT Chuyên Thái Bình 141 HỘI CÁC TRƯỜNG THPT CHUYÊN KHU VỰC DUYÊN HẢI - ĐỒNG BẰNG BẮC BỘ HỘI THẢO KHOA HỌC LẦN THỨ VI Trong đó k và k + 1 là các phòng có người chuyển. Do đó, S(n) là một đơn biến. Do S(n) giảm và S(n) ∈ N * nên việc chuyển phòng phải dừng lại sau hữu hạn ngày. Ví dụ 3.4 Tô đen 09 ô của bàn cờ 10 x 10. Mỗi lần tô màu đen một ô chưa tô nếu nó kề với ít nhất hai ô đen (kề được hiểu là chung cạnh). Có thể tô màu hết bàn cờ hay không? Nếu là 10 ô thì sao? Nếu là hình vuông n x n thì lúc đầu cần tô đen ít nhất bao nhiêu ô để có thể tô đen cả bàn cờ. Lời giải: Chu vi những hình đen hoặc không giảm , hoặc giảm 2, hoặc giảm 4. Tổng chu vi những hình đen ban đầu nhỏ hơn hoặc bằng 36. Chu vi bàn cờ 10x10 là 40. Như vậy, không thể tô đen cả bàn cờ. Ví dụ 4.4 (VMO 2012) Cho số nguyên dương n. Có n học sinh nam và n học sinh nữ xếp thành một hàng ngang, theo thứ tự tùy ý. Mỗi học sinh (trong số 2n học sinh vừa nêu) được cho một số kẹo bằng đúng số cách chọn ra hai học sinh khác giới với X và đứng ở hai phía của X. Chứng minh rằng tổng số kẹo mà tất cả 2n học sinh nhận được không vượt quá 1 n ( n 2 − 1) . 3 Lời giải: Gọi các bạn nam lần lượt là x1, x2, …, xn và các bạn nữ lần lượt là y1, y2, …, yn. Bằng tính toán trực tiếp ta thấy rằng nếu các bạn nam nữ xếp xen kẽ x1, y1, x2, y2, …, xn, yn thì tổng số kẹo cua 2n bạn đúng bằng 1 n ( n 2 − 1) . 3 Để chứng minh kết luận của bài toán ta sẽ chứng tỏ rằng một cách xếp hàng bất kì đều có thể chuyển dần về cách xếp xen kẽ như trên mà trong quá trình chuyển tổng số kẹo mà các bạn nhận được không giảm đi. Giả sử một cách Trường THPT Chuyên Thái Bình 142 HỘI CÁC TRƯỜNG THPT CHUYÊN KHU VỰC DUYÊN HẢI - ĐỒNG BẰNG BẮC BỘ HỘI THẢO KHOA HỌC LẦN THỨ VI xếp hàng bất kì đã có đúng r bạn nam và đúng r bạn nữ ở cuối được xếp xen kẽ ( 0 ≤ r < n ). Không mất tổng quát, ta có thể giả sử hàng có một trong hai dạng sau: i)..., y r +1 , x r + k ,..., x r +1 , x r , y r , x r −1 , y r −1 ,..., x1 , y1 k ban nam lien tiep r cap nam, nu xem ke ii)..., x r +1 , y r + k ,..., y r +1 , x r , y r , x r −1 , y r −1 ,..., x1 , y1 ,khi k > 1 k ban nu lien tiep r cap nam, nu xem ke Trong trường hợp i, ta chuyển bạn yr+1 đến vị trí ngay trước bạn xr. Khi đó chỉ có số kẹo của các bạn yr+1, xr+k, …., xr+1 thay đổi. Bằng tính toán trực tiếp ta thấy tổng số kẹo tăng một lượng là (k2 - k) Trong trường hợp ii), ta chuyển bạn xr+1 đến vị trí ngay trước bạn yr+1. Khi đó chỉ có số kẹo của các bạn xr+1, yr+k, …, yr+2 thay đổi. Ta tính được tổng số kẹo cũng tăng một lượng là (k2 - k) Như vậy, sau không quá n lần chuyển thì hàng được xếp xen kẽ nam, nữ. Do đó tổng số kẹo trong một cách xếp bất kì luôn nhỏ hơn hoặc bằng 1 n ( n 2 − 1) . 3 C. KẾT LUẬN Trên đây, tôi đã trình bày bốn ứng dụng cơ bản và thường dùng của đại lượng bất biến và đơn biến trong việc giải bài toán tổ hợp, các bài toán được chọn khá đa dạng và phong phú . Qua đó giúp học sinh tiếp cận và hình thành phương pháp giải quyết một lớp các bài toán cùng loại. , đặc biệt là giúp các em nhìn nhận và tìm ra các đại lượng bất biến và đơn biến ẩn trong các bài toán tổ hợp, từ đó có được cách giải bài toán phù hợp, tăng thêm tính say mê và tích cực tìm tòi, sáng tạo ở các em. Tôi viết chuyên đề nhằm mục đích cùng trao đổi với Quý Thầy Cô dạy bộ môn toán về việc “hệ thống” các kiến thức, một vài kỹ năng về đại lượng bất biến và đại lượng đơn biến. Vì kiến thức và thời gian còn nhiều hạn chế nên chắc rằng chuyên đề có thiếu sót, tôi chân thành mong muốn và đón nhận sự trao đổi, góp ý của Quý Thầy Cô để chuyên đề ngày càng hoàn thiện và sâu sắc hơn nữa./. Tôi xin chân thành cảm ơn! Trường THPT Chuyên Thái Bình 143 HỘI CÁC TRƯỜNG THPT CHUYÊN KHU VỰC DUYÊN HẢI - ĐỒNG BẰNG BẮC BỘ HỘI THẢO KHOA HỌC LẦN THỨ VI Chuyên đề xếp loại B PHƯƠNG PHÁP THIẾT LẬP HỆ THỨC TRUY HỒI TRONG CÁC BÀI TOÁN ĐẾM Nguyễn Thành Đô – Trường THPT Chuyên Bắc Ninh I. Cơ sở của phương pháp Trong nhiều trường hợp, việc đếm trực tiếp các đối tượng là khó khăn và phức tạp. Nếu ta thiết lập được mối quan hệ truy hồi giữa số lượng đối tượng cần đếm trong nhóm n đối tượng với số lượng đối tượng cần đếm trong các nhóm ít hơn n đối tượng thì ta có thể đưa về đếm số đối tượng trong nhóm mới với số đối tượng ít hơn. Nói cách khác, thay vì đếm trực tiếp S(n) , ta thiết lập hệ thức liên hệ giữa S(n) với S (n − 1) , S(n − 2) …, từ đó dùng kiến thức về dãy số để tìm được S(n) . II.Các ví dụ Ví dụ 1. Cho số nguyên dương n . Có bao nhiêu số tự nhiên chia hết cho 3, có chữ số và các chữ số đều thuộc {3,4,5,6}? Lời giải: . Gọi xn là số các số có n chữ số lập từ {3, 4,5, 6} và chia hết cho 3, yn là số các số có n chữ số lập từ {3, 4,5, 6} và không chia hết cho 3. . Xét 1 số có n chữ số thoả mãn bài toán là x = a1a2...an TH1: Nếu a1a2...an−1M 3 thì x M3 ⇔ an M3 , do đó có 2 cách chọn an. Như vậy trường hợp này có 2 x n −1 cách chọn x . TH2: thuộc a1a2 ...an −1 không chia hết cho 3. Khi đó ta chỉ chọn được 1 số an {3, 4 , 5, 6} để x = a1a2 ...an M 3 . Như vậy trường hợp này có y n −1 cách chọn x . Như vậy ta có: x n = 2. x n −1 + y n −1 Trường THPT Chuyên Thái Bình 144 n HỘI CÁC TRƯỜNG THPT CHUYÊN KHU VỰC DUYÊN HẢI - ĐỒNG BẰNG BẮC BỘ HỘI THẢO KHOA HỌC LẦN THỨ VI y n = 2. x n −1 + 3. y n −1 Tương tự ta thu được: Biến đổi ta thu được x n +1 − 5 x n + 4 x n −1 = 0. Giải phương trình sai phân này với chú ý rằng x1 = 2; x 2 = 6 ta tìm được xn = 4n + 2 3 Ví dụ 2. Cho số nguyên (1, 2,..., n ) n ≥ 2 . Hãy tìm số các hoán vị ( a1 , a 2 ,..., a n ) của sao cho tồn tại duy nhất một chỉ số i ∈ {1, 2,..., n − 1} thoả mãn a i > a i +1 . Lời giải: Gọi Sn là số các hoán vị thoả mãn điều kiện bài toán. . a n = n ⇒ số các hoán vị có dạng ( a1 , a 2 ,..., a n −1 , n ) là S n −1 . a n −1 = n ⇒ số các hoán vị có dạng ( a1 , a 2 ,..., a n − 2 , n , a n ) là C nn−−12 . a i = n ⇒ số các hoán vị ( a1 , a 2 ,..., a n ) thoả mãn là C ni −−11 với ∀ i = 1; n − 1 . Do vậy ta có S n = S n −1 + n −1 ∑C i =1 i −1 n −1 = S n−1 + 2n−1 − 1 Lại có S 2 = 1 nên S n = 2 − n − 1. n Ví dụ 3. Cho tập S = {1; 2;...; n} với tập S n là số nguyên lớn hơn 2. Tìm số tập con của sao cho trong mỗi tập con đều có ít nhất hai phần tử là hai số nguyên liên tiếp. Lời giải: Gọi Sn là tập hợp các tập con khác ∅ của tập {1; 2;...; n} mà trong mỗi tập con không có hai phần tử nào là hai số nguyên liên tiếp. Chia các phần tử của Sn thành hai nhóm: . Nhóm không chứa phần tử n : Số các tập con như vậy là Sn−1 ; . Nhóm chứa phần tử n : {n} hoặc { a1 ; a 2 ;...; a k ; n } (1 ≤ k ≤ n − 1) Rõ ràng ai ≠ n − 1(i = 1, 2,..., k ) nên số các tập con như vậy là Sn−2 + 1 Do vậy S n = S n −1 + S n − 2 + 1 Với chú ý S 2 = 2, S 3 = 4 , ta có Trường THPT Chuyên Thái Bình 145 HỘI CÁC TRƯỜNG THPT CHUYÊN KHU VỰC DUYÊN HẢI - ĐỒNG BẰNG BẮC BỘ HỘI THẢO KHOA HỌC LẦN THỨ VI 1  1 + 5   Sn =  5  2   Mặt khác, số tập con khác n+2 ∅ 1− 5  −   2  n+2   −1   của tập {1; 2;...; n} là 2n −1. Vậy số tập con thoả mãn đề bài là 1  1 + 5   2 −  5  2   n+2 n 1− 5  −   2  n+2     . Tìm số các bộ số nguyên ( a1 , a 2 ,..., a n ) thoả mãn ai ≤ 1 với ∀i = 1,2,..., n và ai − ai +1 ≤ 1 ∀i = 1,2,..., n − 1. Ví dụ 4. Cho số nguyên n > 1 Lời giải: .Trong tập Sn các bộ số nguyên thoả mãn bài toán, gọi A n , B n , C n lần lượt là tập hợp các bộ có an bằng −1,0,1 tương ứng. Ta có S n = An + B n + C n . .Mặt khác, dễ thấy từ mỗi bộ thuộc A n hoặc B n , ta có thể bổ sung an+1 = −1 để được một bộ thuộc An +1 nên An +1 = An + B n . .Tương tự ta có C n +1 = C n + B n và Bn +1 = An + Bn + C n = S n Từ đó ta có: S n +1 = An +1 + B n +1 + C n +1 = ( An + Bn + C n ) + Bn +1 + Bn = 2 S n + S n −1 Kết hợp với S 2 = 7, S 3 = 17 ta tính được S n (1 + 2 ) = n +1 ( + 1− 2 2 ) n +1 . Ví dụ 5. Cho số nguyên dương n . Có bao nhiêu số tự nhiên có n chữ số, trong mỗi số các chữ số đều lớn hơn 1 và không có hai chữ số khác nhau cùng nhỏ hơn 7 đứng cạnh nhau? Lời giải: Kí hiệu X n là tập tất cả các số tự nhiên có n chữ số thoả mãn đề bài, An , B n là các tập con của X n theo thứ tự gồm các số có tận cùng nhỏ hơn 7; các số có tận cùng lớn hơn 6. Trường THPT Chuyên Thái Bình 146 HỘI CÁC TRƯỜNG THPT CHUYÊN KHU VỰC DUYÊN HẢI - ĐỒNG BẰNG BẮC BỘ HỘI THẢO KHOA HỌC LẦN THỨ VI Ta có X n = An ∪ B n , An ∩ B n = ∅ ⇒ X n = An + B n Lấy một phần tử của X n +1 bỏ đi chữ số tận cùng ta được một phần tử của X n . Nếu chữ số tận cùng nhỏ hơn 7 (thuộc A n ) thì chỉ có 1 cách thêm vào chữ số cuối để được 1 phần tử của An +1 và có đúng 3 cách thêm vào chữ số cuối để được 1 phần tử của B n +1 . Nếu chữ số tận cùng lớn hơn 6 (thuộc B n ) thì có 5 cách thêm vào chữ số cuối để được 1 phần tử của An +1 và có đúng 3 cách thêm vào chữ số cuối để được 1 phần tử của B n +1 . Từ các lập luận trên ta có:  An +1 = An + 5 B n (1)   B n +1 = 3 An + 3 B n (2) Từ (1) và (2) suy ra An +1 + B n +1 = 4 An + 8 B n = 4 ( An + B n ) + 4 B n = 4 ( An + Bn ) + 12 ( An −1 + Bn −1 ) ( n ≥ 2) Kí hiệu xn = X n , ta có x n + 2 = x n +1 + 12 x n , n ∈  x n + 2 − 6 x n + 1 = − 2 ( x n + 1 − 6 x n ) ⇒   x n + 2 + 2 x n + 1 = 6 ( x n + 1 + 2 x n ) * . Từ đó ta có: n  x n + 2 − 6 x n + 1 = ( − 2 ) ( x 2 − 6 x 1 )  n  x n + 2 + 2 x n + 1 = (6 ) ( x 2 + 2 x 1 ) 1 ⇒ xn +1 = [( x2 + 2 x1 ).6 n − ( x2 − 6 x1 ).( − 2) n ] 8 Dễ thấy x1 = 8, ta tìm x2. Xét u ∈ X 2 ⇒ u = ab ; a, b ∈{2,3,4,5,6,7,8,9} . Nếu a∈{2,3,4,5,6} thì có 4 cách chọn . Nếu a ∈ {7,8,9} thì có 8 cách chọn b b Vậy x 2 = 5.4 + 3.8 = 44 . Do đó 1 xn = [15.6n−1 + (−2)n−1 ]. 2 Ví dụ 6.(IMO 2011). Giả sử là một số nguyên. Cho một cái cân đĩa và n > 0 n quả cân có khối lượng lần lượt là 20 , 21 , 2 2 ,..., 2 n−1. Ta muốn đặt lên cái cân mỗi một Trường THPT Chuyên Thái Bình 147 HỘI CÁC TRƯỜNG THPT CHUYÊN KHU VỰC DUYÊN HẢI - ĐỒNG BẰNG BẮC BỘ HỘI THẢO KHOA HỌC LẦN THỨ VI trong n quả cân, lần lượt từng quả một, theo cách để đảm bảo đĩa cân bên phải không bao giờ nặng hơn đĩa cân bên trái. Ở mỗi bước ta chọn một trong các quả cân chưa được đặt lên rồi đặt nó lên đĩa bên phải, hoặc đĩa bên trái, cho đến khi tất cả các quả cân đều được đặt lên đĩa. Hỏi có bao nhiêu cách để thực hiện việc đặt cân theo đúng mục đích đặt ra? Lời giải: Gọi sn là số cách để thực hiện việc đặt cân theo đúng mục đích đặt ra. Xét cách đặt n + 1 quả cân có khối lượng 20 ,21 ,22 ,...,2n. Do 2 0 + 21 + 2 2 + ... + 2 n −1 = 2 n − 1 < 2 n nên trong mọi cách đặt cân thoả mãn n luôn được đặt ở đĩa cân bên trái. bài toán thì quả cân có khối lượng 2 n n Nếu quả cân 2 được chọn cuối cùng: chỉ có một cách đặt ( vì quả 2 chỉ đặt lên đĩa bên trái ) và số cách đặt n quả cân còn lại là sn. n được đặt ở bước thứ i (i = 1, 2,..., n ) . Khi đó có Nếu quả cân 2 n cách chọn i và trong trường hợp này quả cân 2n−1 có 2 cách đặt ( đĩa bên phải hay bên trái đều thoả mãn ), do đó số cách đặt n + 1 quả cân trong trường hợp này là 2ns . n . Vậy ta có hệ thức truy hồi: s n +1 = 2 n.s n + s n = ( 2 n + 1) s n Ta có s1 =1 nên s n = ( 2 n − 1)( 2 n − 3 ) ...3.1. Ví dụ 7.(VMO 2009). Cho số nguyên dương n . Kí hiệu T là tập hợp gồm số nguyên dương đầu tiên. Hỏi có tất cả bao nhiêu tập con S 2 n của T có tính chất: trong không tồn tại các số a, b mà a − b ∈ {1; n}? S Lời giải: Với mỗi n ∈ * , kí hiệu dn là số cần tìm theo yêu cầu đề bài. Xét bảng ô vuông kích thước 2 × n . Điền vào các ô vuông con của bảng, lần lượt từ trái qua phải, từ trên xuống dưới, các số từ 1 đến 1 2 n + 1 n + 2 … … n − 1 n 2 n − 1 2 n Trường THPT Chuyên Thái Bình 148 2 n . HỘI CÁC TRƯỜNG THPT CHUYÊN KHU VỰC DUYÊN HẢI - ĐỒNG BẰNG BẮC BỘ HỘI THẢO KHOA HỌC LẦN THỨ VI Gọi ô thứ n của hàng 1 và ô thứ 1 của hàng 2 là hai ô đặc biệt. Khi đó hai số ab∈ , T thoả a − b ∈ {1; n} khi và chỉ khi chúng nằm ở 2 ô kề nhau hoặc ở 2 ô đặc biệt. Vì thế dn chính bằng số cách chọn 1 số ô của bảng (kể cả số ô được chọn bằng 0) mà ở mỗi cách không có 2 ô kề nhau hoặc 2 ô đặc biệt được chọn. Với mỗi n ∈ * , kí hiệu +/ kn là số cách chọn mà ở mỗi cách không có 2 ô kề nhau được chọn (*) +/ sn là số cách chọn mà trong các ô được chọn ở mỗi cách có 2 ô đặc biệt và không có 2 ô kề nhau. Ta có: d n = k n − s n • Tính kn Dễ thấy, tất cả các cách chọn ô thoả mãn điều kiện (*) bao gồm : +/ k n −1 cách chọn mà ở mỗi cách không có ô nào thuộc cột 1 của bảng được chọn +/ 2 t n −1 cách chọn mà ở mỗi cách đều có ô thuộc cột 1 của bảng được chọn; trong đó tn là số cách chọn ô thoả mãn điều kiện (*) của bảng khuyết đơn 2 × n (h.2) … … (h.2) x Do đó k n = k n −1 + 2 t n −1 (1) Lại có, tất cả các cách chọn ô thoả mãn điều kiện (*) từ bảng khuyết đơn 2 × n bao gồm: +/ k n −1 cách chọn mà ở mỗi cách ô đánh dấu “x” không được chọn; +/ tn−1 cách chọn mà ở mỗi cách ô đánh dấu “x” đều được chọn. Vì thế t n = k n −1 + t n −1 Từ đó và (1) suy ra k n = k n −1 + 2( k n − 2 + t n − 2 ) = 2 k n −1 + k n − 2 (2) Bằng cách đếm trực tiếp, ta có k 1 = 3, k 2 = 7 . Do đó ta tìm được kn (1 + 2 ) = n +1 ( + 1− 2 Trường THPT Chuyên Thái Bình ) n +1 2 149 (3) HỘI CÁC TRƯỜNG THPT CHUYÊN KHU VỰC DUYÊN HẢI - ĐỒNG BẰNG BẮC BỘ HỘI THẢO KHOA HỌC LẦN THỨ VI • Tính sn Dễ thấy s1 = 0, s 2 = s 3 = 1 và với n ≥ 4 ta có: sn = hn−2 , trong đó hn là số cách chọn ô thoả mãn ðiều kiện (*) từ bảng khuyết kép … … A 2 × n (h.3) B (h.3) Do s3 =1, đặt h1 =1. Bằng cách đếm trực tiếp, ta có h2 = 4 . Xét n ≥ 3 . Dễ thấy, tất cả các cách chọn ô thoả mãn điều kiện (*) từ bảng khuyết kép 2 × n bao gồm: +/ k n − 2 cách chọn mà ở mỗi cách cả 2 ô A và B đều không được chọn; +/ 2tn − 2 cách chọn mà ở mỗi cách có đúng 1 trong 2 ô A, B được chọn; +/ th − 2 cách chọn mà ở mỗi cách cả 2 ô A, B cùng được chọn. Do đó h n = k n − 2 + 2 t n − 2 + h n − 2 = k n −1 + h n − 2 (4) Từ (2) và (4) suy ra 2 h n − k n = 2 h n − 2 − k n − 2 , ∀ n ≥ 3 Dẫn tới 2 hn − k n = ( −1) , ∀n ≥ 1 . n Vì thế sn = hn−2 • kn−2 + (−1) n−2 = , ∀n ≥ 3. 2 Vậy d 1 = 3, d 2 = 6 2kn − kn−2 + (−1)n−3 dn = , ∀n ≥ 3( kn theo (3) ) 2 Ví dụ 8. Có n quả bóng b1 , b2 ,..., bn và 2 n hộp h1 , h2 ,..., h2 n . Biết rằng quả bóng bi ( i = 1, 2,..., n ) chỉ bỏ được vào các hộp h1 , h 2 ,..., h 2 i . Hỏi có bao nhiêu cách bỏ k (1 ≤ k ≤ n ) quả bóng vào các hộp, biết rằng mỗi hộp chứa nhiều nhất một quả bóng? (Hai cách bỏ bóng được gọi là khác nhau khi có ít nhất một quả bóng được bỏ vào hai hộp khác nhau trong hai cách đó) Lời giải: Trường THPT Chuyên Thái Bình 150 HỘI CÁC TRƯỜNG THPT CHUYÊN KHU VỰC DUYÊN HẢI - ĐỒNG BẰNG BẮC BỘ HỘI THẢO KHOA HỌC LẦN THỨ VI Đặt Sn,k là số cách bỏ trong k quả bóng vào các hộp. Giả sử k quả bóng được chọn là bn thì 2 ≤ k ≤ n . Nếu một quả bóng còn lại có thể bỏ vào các hộp k − 1 bằng Sn−1,k −1 cách. Đồng thời, bncó 2 n − ( k − 1) = 2 n − k + 1 cách chọn một hộp trong các hộp còn lại để bỏ. Do đó số cách bỏ bóng trong trường hợp này là: ( 2 n − k + 1) .S n −1,k −1 Trường hợp quả bóng bn không được chọn, lưu ý rằng k ≤ n − 1 . Mọi quả bóng trong các quả bóng b1 , b2 ,..., bn −1 đều có thể bỏ vào các hộp bằng Sn−1,k cách, suy ra S n , k = S n −1, k + ( 2 n − k + 1 ) S n −1, k −1 ( n ≥ 3, 2 ≤ k ≤ n ) Nhận thấy S n , n = ( n + 1 ) S n −1, n −1 ; S n ,1 = n ( n + 1 ) ; S 1,1 = 2 Từ đó bằng quy nạp ta chứng minh được Sn,k = Ví dụ 9. Xét đa giác đều OAi Ai +1 (1 ≤ i ≤ n ) ( An +1 ≡ A1 ) n ( n + 1) k !(Cnk ) 2 . n − k +1 đỉnh với tâm O . Người ta tô màu các miền tam giác bằng k ( k ≥ 3) màu sao cho hai miền kề nhau được tô bởi hai màu khác nhau. Hỏi có bao nhiêu cách tô màu như vậy? Lời giải: Gọi S ( n , k ) là số cách tô màu thoả mãn bài toán. Ta có k cách tô màu miền O A1 A2 , k − 1 cách tô màu miền OA2 A3 ,…, cách tô màu miền O An A1 . Do đó có tất cả k ( k − 1) n−1 k − 1 cách tô. Tuy nhiên, ta phải trừ đi các cách tô sai, chẳng hạn khi các miền O An A1 và O A1 A2 cùng màu, khi đó ta coi ), số cách tô như vậy là S ( n − 1, k ) . Do OAn A2 như một miền tam giác (bỏ qua đỉnh A 1 đó ta có hệ thức: S ( n, k ) = k ( k − 1) n −1 = k ( k − 1) − S ( n − 1, k ) n −1 − [ k ( k − 1) n −1 − k ( k − 1) n−2 − S ( n − 2, k )] = ... = k ( k − 1) Trường THPT Chuyên Thái Bình n−2 151 + ... + ( −1) n−4 [k ( k − 1) − S ( 3, k )] 3 HỘI CÁC TRƯỜNG THPT CHUYÊN KHU VỰC DUYÊN HẢI - ĐỒNG BẰNG BẮC BỘ HỘI THẢO KHOA HỌC LẦN THỨ VI Suy ra S ( n, k ) = ( k − 1) + ( −1) ( k − 1) . n n Ví dụ 10. Kí hiệu f (n) là số hoán vị ( a1 , a 2 ,..., a n ) của (1, 2,..., n ) thoã mãn đồng thời các điều kiện: 1) a1 = 1 2) a i − a i +1 ≤ 2, ∀ i = 1, 2,..., n − 1 Hỏi f ( 2013 ) có chia hết cho 3 không? Lời giải: Ta xét với n ≥ 5 . Do a1 = 1 và a1 − a 2 ≤ 2 nên a 2 = 2 hoặc a 2 = 3 . +) Nếu a 2 = 2 thì ( a 2 , a 3 ,..., a n ) là hoán vị của ( 2,3,..., n ) thoả mãn i. a 2 = 2 ii. a i − a i +1 ≤ 2, ∀ i = 2, 3,..., n − 1 Số các hoán vị như vậy chính là f ( n − 1) +) Nếu a 2 = 3 thì a3 ∈ {2, 4,5} Giả sử có a k = 2 (3 < k < n ) thì do a k −1 − a k ≤ 2 , a k − a k +1 ≤ 2 và a k −1 , a k khác 1, 2, 3 nên a k −1 = a k +1 = 4 ⇒ vô lí. Vậy a 3 = 2 hoặc a n = 2 . Nếu a 3 = 2 thì a 4 = 4 , do đó ( a 4 , a 5 ,..., a n ) là hoán vị của ( 4, 5,..., n ) thoả mãn i. a 4 = 4 ii. a i − a i +1 ≤ 2, ∀ i = 4, 5,..., n − 1 Số các hoán vị như vậy chính là f ( n − 3 ) Nếu a n = 2 thì a n −1 = 4 nên a 3 = 5 , kết hợp với giả thiết suy ra a n − 2 = 6, a 4 = 7, a n −3 = 8,... Cứ như thế chỉ có một hoán vị thoả mãn. Dễ dàng tính được f ( 2 ) = 1, f ( 3 ) = 2, f ( 4 ) = 4 Tóm lại, ta có hệ thức truy hồi: f ( 2 ) = 1, f ( 3 ) = 2, f ( 4 ) = 4 f ( n ) = f ( n − 1) + f ( n − 3 ) , ∀ n ≥ 5 Trường THPT Chuyên Thái Bình 152 HỘI CÁC TRƯỜNG THPT CHUYÊN KHU VỰC DUYÊN HẢI - ĐỒNG BẰNG BẮC BỘ HỘI THẢO KHOA HỌC LẦN THỨ VI Khi đó ta chứng minh được dãy { f ( n ) (mod 3)}∞n = 2 là dãy tuần hoàn chu kì 2, do đó: f ( 2013 ) ≡ f ( 3 ) ≡ 2 (mod 3) Vậy f ( 2013 ) không chia hết cho 3. III. Luyện tập Bài1.Cho là n một số nguyên dương. Từ các E = {1; 2;3; 4;5;6;7;8;9} có thể lập được bao nhiêu số tự nhiên có số n thuộc tập chữ số mà trong mỗi số đều chứa một số lẻ chữ số 1 và một số chẵn chữ số 2? 9n − 5n (Đáp số: ) 4 Bài 2. Cho số nguyên dương B của A n ≥ 2 . Xét tập A = {1; 2;3;...; 2 n } . Tìm số tập con mà mỗi tập con đều có tính chất: Nếu x , y là hai phần tử khác nhau của có tổng là một luỹ thừa của 2 thì đúng một trong hai phần tử x , y A và này thuộc B. (Đáp số: 2n+1 ) Bài 3. Có n người ngồi thành một hàng ngang vào n ghế. Hỏi có bao nhiêu cách lập hàng mới mà trong mỗi cách lập hàng mới; mỗi người hoặc giữ nguyên vị trí của mình, hoặc đổi chỗ cho người liền bên phải, hoặc đổi chỗ cho người liền bên trái? Đ/s: 1  1 + 5    5  2   n +1 n +1 1− 5   −    2   Bài 4. Cho tập S = {1; 2;...; n} với tập S n là số nguyên dương. Tìm số tập con A của mà A chứa đúng hai số nguyên dương liên tiếp. Đ/s: a n + 2 = 2( n + 2) Fn + 2 − ( n + 3) Fn + 2 , ở đó Fn là shtq của dãy Fibonaci 5 Bài 5 Có n (n > 1) thí sinh ngồi xung quanh một bàn tròn. Hỏi có bao nhiêu cách phát đề sao cho hai thí sinh ngồi cạnh nhau luôn có đề khác nhau, biết rằng trong ngân hàng đề có đúng m ( m > 1) đề và hiển nhiên mỗi đề có nhiều bản? Đ/s: Pn = ( m − 1) + ( m − 1).( −1) n Trường THPT Chuyên Thái Bình n 153 HỘI CÁC TRƯỜNG THPT CHUYÊN KHU VỰC DUYÊN HẢI - ĐỒNG BẰNG BẮC BỘ HỘI THẢO KHOA HỌC LẦN THỨ VI Bài 6 Có bao nhiêu số tự nhiên a) n n thoả mãn đồng thời các điều kiện sau: có 1000 chữ số b) Tất cả các chữ số của n là lẻ c) Hiệu của hai chữ số liên tiếp bất kì của n luôn bằng 2. Đáp số: 8.3499 Bài 7 Cho bảng ô vuông n × n (n > 1) . Hỏi có bao nhiêu cách đánh dấu các ô vuông trong bảng sao cho mỗi hình vuông 2 × 2 có đúng hai ô vuông được đánh dấu? ( Hai cách đánh dấu được gọi là khác nhau nếu có một ô vuông nào đó mà trong cách này thì được đánh dấu còn trong cách kia thì không ). Đ/s: Sn = 8n − 10 Bài 8. Cho tập S = {1; 2;...; n} với của tập S n là số nguyên lớn hơn 2. Tìm số tập con sao cho trong mỗi tập con không chứa hai số nguyên liên tiếp nào. n n     Đ/s: 5 − 3  1 − 5  + 5 + 3  1 + 5  − 1 2 5  2  2 5  2  Bài 9. (IMO 1979). Giả sử A và E là hai đỉnh đối tâm của một bát giác đều. Một con ếch bắt đầu nhảy từ đỉnh A. Tại mỗi đỉnh của bát giác (trừ đỉnh E), mỗi cú nhảy con ếch chỉ có thể nhảy tới hai đỉnh kề nó. Khi con ếch nhảy vào đỉnh E thì nó bị mắc kẹt ở đó. Cho trước số nguyên dương n . Hỏi với n cú nhảy, có bao nhiêu cách để con ếch nhảy vào đỉnh E? Đ/s: a2 n −1 = 0; a2 n = ( 1  2+ 2 2  ) n −1 ( − 2− 2 ) n −1   Bài 10. Giả sử P1 , P2 ,..., Pn theo thứ tự là các điểm trên cùng một đường thẳng. Người ta tô các điểm đó bằng 5 màu khác nhau, mỗi điểm tô 1 màu sao cho 2 điểm Pi , Pi +1 ( i = 1, 2,..., n − 1) luôn hoặc là cùng màu hoặc là 1 trong 2 điểm được tô màu xanh. Hỏi có bao nhiêu cách tô như vậy? 3n+1 + ( −1) Đ/s: 2 n +1 . Trường THPT Chuyên Thái Bình 154 HỘI CÁC TRƯỜNG THPT CHUYÊN KHU VỰC DUYÊN HẢI - ĐỒNG BẰNG BẮC BỘ HỘI THẢO KHOA HỌC LẦN THỨ VI Chuyên đề xếp loại B MỘT SỐ BÀI TOÁN TỔ HỢP LIÊN QUAN ĐẾN PHỦ HÌNH CHỮ NHẬT Trần Mạnh Sang – THPT chuyên Lê Hồng Phong, Nam Định Một hình chữ nhật m × n (có m dòng và n cột) được phân chia thành m.n ô vuông đơn vị. Ta sẽ nghiên cứu bài toán phủ hình chữ nhật này bằng các hình được ghép từ các ô vuông đơn vị. Với các bài toán dạng này, thường ta đánh số hoặc tô màu các ô vuông đơn vị của hình chữ nhật, có hai cách tô màu cơ bản: Cách 1: Tô màu đen và trắng xem kẽ trên mỗi hàng và mỗi cột Nếu có một trong hai số m, n chẵn thì số ô đen và trắng là bằng nhau Nếu cả hai số đều lẻ thì màu nào được tô vào ô đầu tiên sẽ nhiều hơn màu kia 1 đơn vị. Cách 2: Tô màu giống nhau trên 1 hàng (hoặc cột) và xen kẽ trên 1 cột (hoặc hàng) Nếu có một trong hai số m, n chẵn thì số ô đen và trắng là bằng nhau Nếu cả hai số đều lẻ thì màu nào được tô vào ô đầu tiên sẽ nhiều hơn màu kia m ô (nếu tô màu giống nhau theo mỗi cột). Các dạng hình sử dụng để phủ hình chữ nhật: Dạng 1: Monomino là một ô vuông đơn vị 1 × 1 . Dạng này không xét riêng, bởi đơn giản. Ta sẽ tìm hiểu chúng khi kết hợp với các loại khác. Dạng 2: Domino là hình được tạo từ 2 ô vuông đơn vị, ta sẽ gặp khi xét chung cho các loại thẳng. Dạng 3: Trimino được tạo từ 3 ô vuông đơn vị, có 2 dạng: Trường THPT Chuyên Thái Bình 155 HỘI CÁC TRƯỜNG THPT CHUYÊN KHU VỰC DUYÊN HẢI - ĐỒNG BẰNG BẮC BỘ HỘI THẢO KHOA HỌC LẦN THỨ VI Trimino thẳng (sẽ được xét trong phần monomino thẳng) L - Trimino Ta xây dựng điều kiện cần và đủ để hình chữ nhật m × n bởi các L – trimino bằng các bài toán: 1. Một hình chữ nhật 4 × 5 không phủ được bởi các L – trinimo bởi 20 không chia hết cho 3. Qua đây cho ta điều kiện cần để hình chữ nhật m × n có thể được phủ bởi một hình được tạo từ k ô vuông đơn vị: m n k . 2. Hai L – trimino phủ được một hình chữ nhật 2 × 3 . Vậy một hình chữ nhật được phân chia thành các hình chữ nhật 2 × 3 thì có thể được phủ bởi các L – trimino. 3. Tìm hình vuông nhỏ nhất có thể được phủ bởi các L – trinimo? Hình vuông n × n được phủ bởi L – trimino thì n2 3 Xét với n = 3 thấy không thỏa mãn. Xét với n = 6 , ta phủ bằng cách chia thành các hình chữ nhật 2 × 3 . 4. Tìm tất cả các số nguyên dương b sao cho hình chữ nhật 2 × b có thể được phủ bởi các L – trimino. Nhận thấy: Nếu b 3 ⇒ 2b 3 ⇒ không phủ được. Nếu b 3 ⇒ hình chữ nhật 2 × b được phân thành các hình chữ nhật 2 × 3 , suy ra phủ được bởi các L – trimino, vậy b 3 là tất cả các số b thỏa mãn. 5. Tìm tất cả các số nguyên dương b sao cho hình chữ nhật 3 × b có thể được phủ bởi các L – trimino. Muốn phủ hình chữ nhật thì cần phủ ô vuông đầu tiên, nhận thấy có 3 cách như sau: Trường THPT Chuyên Thái Bình 156 HỘI CÁC TRƯỜNG THPT CHUYÊN KHU VỰC DUYÊN HẢI - ĐỒNG BẰNG BẮC BỘ HỘI THẢO KHOA HỌC LẦN THỨ VI Với trường hợp c thấy không thể phủ được dòng cuối. Với hai trường hợp còn lại muốn phủ được dòng cuối cần tạo ra các hình chữ nhật 3 × 2 . Vậy muốn phủ được thì b 2 . Từ 5 bài toán trên, ta có định lý 1: Cho các số nguyên dương a, b với 2 ≤ a ≤ 3 và a ≤ b . Hình chữ nhật a × b có thể được phủ bởi các L-trimino khi và chỉ khi ab 6 . Tuy nhiên định lý chỉ đưa ra điều kiện cho hình chữ nhật có 1 trong hai cạnh là 2 hoặc 3, ta sẽ xét các hình chữ nhật mà cả hai cạnh đều lớn hơn 3. Cùng đến với một số bài toán sau: 6. Chứng minh rằng hình chữ nhật 5 × 6 có thể được phủ bởi các L – trimino. Nhận thấy có thể chia thành các hình chữ nhật 2 × 3 7. Chứng minh rằng hình chữ nhật 5 × 9 có thể được phủ bởi các L – trimino. Chỉ rõ một cách phủ như sau: 8. Từ bài 7 hãy chứng minh rằng hình chữ nhật 9 × 9 có thể được phủ bởi các L – trimino. Trường THPT Chuyên Thái Bình 157 HỘI CÁC TRƯỜNG THPT CHUYÊN KHU VỰC DUYÊN HẢI - ĐỒNG BẰNG BẮC BỘ HỘI THẢO KHOA HỌC LẦN THỨ VI Gộp thêm các hình chữ nhật 2 × 3 vào hình chữ nhật 5 × 9 để được hình chữ nhật 9×9. 9. Từ đây nhận thấy: Nếu hình chữ nhật a × b với b 3 phủ được bởi các L – trimino thì hình chữ nhật ( a + 2 ) × b cũng phủ được. (Dễ dàng chứng minh như ý 8) 10. Chứng minh rằng hình chữ nhật 5 × b với b 3, b > 5 có thể được phủ bởi các L – trimino. Hình chữ nhật như trên được ghép bởi các hình 5 × 6 và 5 × 9 . 11. Chứng minh rằng hình chữ nhật a × b với a ≥ 4, b ≥ 5, b 3 có thể được phủ bởi các L – trimino. Nếu a 2 thì phân chia thành các hình chữ nhật 2 × 3 . Nếu a 2 thì chia thành hình chữ nhật 5 × b và ( a − 5 ) × b . 12. Ta có định lý 2: Hình chữ nhật a × b với a ≥ 4, b ≥ 4 có thể được phủ bởi các L – trimino khi và chỉ khi ab 3 (nghĩa là cần 1 trong hai số chia hết cho 3). Nếu ab 3 thì hình chữ nhật a × b có thể được phủ bởi các L – trimino đã được chứng minh bởi các bài toán trên, cụ thể như sau: Giả sử có b 3 và a chẵn thì có thể phân chia thành các hình chữ nhật 2 × 3 . Giả sử có b 3 và a lẻ thì do 5 × b phủ được nên ( a + 2) × b cũng phủ được, lúc này đã qua hết tất cả các hình chữ nhật có cạnh a lẻ. Gộp cả hai định lý 1 và định lý 2 ta có định lý: Cho a, b là các số nguyên thỏa mãn: 2 ≤ a ≤ b . Hình chữ nhật a × b có thể được phủ bởi các L – trimino khi và chỉ khi một trong các điều sau: i. a = 3 và b chẵn. ii. a ≠ 3 và ab 3 . Dạng 4: Tetramino Là dạng được tạo từ 4 ô vuông đơn vị, có các dạng sau: Trường THPT Chuyên Thái Bình 158 HỘI CÁC TRƯỜNG THPT CHUYÊN KHU VỰC DUYÊN HẢI - ĐỒNG BẰNG BẮC BỘ HỘI THẢO KHOA HỌC LẦN THỨ VI O – tetramino Z – tetramino S – tetramino L – tetramino J – tetramino T – tetramino Tetramino thẳng 1. Với dạng O – tetramino Hình chữ nhật m × n phủ được bởi các O – tetramino khi và chỉ khi m , n đều chẵn. 2. Với S – tetramino và Z – tetramino Một hình chữ nhật không thể được phủ nếu chỉ dùng S – tetramino và Z – tetramino. muốn phủ được thì phải phủ ô vuông đầu tiên, với mỗi loại cho ta một cách phủ ô đầu. khi đó tiếp tục sẽ chỉ có một cách để phủ ô tiếp theo trên hàng 1, cứ như vậy sẽ không phủ được ô cuối cùng trên hàng 1. 3. Với J – tetramino và L – tetramino. Hình chữ nhật m × n được phủ chỉ bởi các J – tetramino hoặc L – tetramino nếu và chỉ nếu m .n 8 . Một số bài toán gợi ý để giải quyết ý trên (Xét với L – tetramino, với J – tetramino hoàn toàn tương tự) a. Tổng số lượng L – tetramino là số chẵn. Trường THPT Chuyên Thái Bình 159 HỘI CÁC TRƯỜNG THPT CHUYÊN KHU VỰC DUYÊN HẢI - ĐỒNG BẰNG BẮC BỘ HỘI THẢO KHOA HỌC LẦN THỨ VI Chứng minh Giả sử hình chữ nhật m × n được tô bởi k , suy ra m .n = 4 k , cần chứng minh k chẵn. Nhận thấy trong hai số m, n có ít nhất 1 số chẵn, giả sử là n. Tô mầu như hình bên, có nhận xét rằng: Số ô đen và trắng bằng nhau. Mỗi L – tetramino khi đặt vào hình chữ nhật sẽ phủ hoặc 3 ô trắng 1 ô đen, hoặc 3 ô đen 1 ô trắng. Gọi a là số L – tetramino khi đặt vào hình chữ nhật sẽ phủ 3 ô trắng 1 ô đen và b là số L – tetramino khi đặt vào hình chữ nhật sẽ phủ 3 ô đen 1 ô trắng. Khi đó số ô trắng trên hình chữ nhật là 3 a + b và số ô đen là 3b + a , suy ra 3 b + a = 3 a + b ⇒ a = b , ta có điều phải chứng minh. b. Hình chữ nhật m × n với m chẵn và n 4 sẽ được phủ bởi các L – tetramino. Do hình chữ nhật 2 × 4 được phủ bởi 2 L – tetramino nên chia nhỏ hình m × n thành các hình 2 × 4 . c. Hình chữ nhật 3 × 8 phủ được bởi các J – tetramino và L – tetramino. 4. Với T – tetramino. • Chứng minh rằng hình chữ nhật m × n được phủ bởi các T – tetramino thì số lượng T – tetramino phải chẵn (chứng minh tương tự như với L – tetramino). • Tìm số hình vuông nhỏ nhất có thể được phủ bởi các T – tetramino. xét hình vuông m × m được phủ bởi k T – tetramino suy ra m2 = k.4 với k chẵn, hay m2 8 xét với m = 4 thấy thỏa mãn. Trường THPT Chuyên Thái Bình 160 HỘI CÁC TRƯỜNG THPT CHUYÊN KHU VỰC DUYÊN HẢI - ĐỒNG BẰNG BẮC BỘ HỘI THẢO KHOA HỌC LẦN THỨ VI Định lý (DW.Walkup, tạp chí Amer Math Monthly – T11, 1965): Hình chữ nhật m × n phủ được bởi các T – tetramino nếu và chỉ nếu m và n cùng chia hết cho 4. Dạng 5: Polimino thẳng Vấn đề 1: Tìm tất cả các hình chữ nhật m × n có thể được phủ bởi k – omino thẳng (hình chữ nhật 1 × k ) 1.1 . Chứng minh rằng nếu m hoặc n chia hết cho k thì hình chữ nhật m × n có thể được phủ bởi k – omino thẳng. 1.2 . Chứng minh rằng nếu m hoặc n cùng không chia hết cho k thì hình chữ nhật m × n không thể được phủ bởi k – omino thẳng. Chứng minh: Thực hiện phép chia có dư: m = k .q1 + r1 ; n = k .q2 + r2 với 0 < r1 ; r2 < k Đánh số hình chữ nhật theo quy tắc sau: Trên mỗi cột các số được đánh giống nhau, trên mỗi hàng các số được xếp liên tiếp theo thứ tự 1, 2, …, k, 1, 2, … đến hết, trên mỗi hàng sẽ kết thúc tại r2 1 2 3 … r2 1 2 3 … r2 1 2 3 … r2 2 2 3 … r2 Nhận xét mỗi k – omino thẳng khi đặt vào hình chữ nhật sẽ phủ các ô cùng đơn vị hoặc phủ đủ các ô từ 1 đến k Đặt Si là số ô được đánh số i, suy ra S1 ≡ S 2 ≡ ≡ S k ( mod k ) Do cách đánh số suy ra số r2 được đánh nhiều hơn số r2 + 1 đến m đơn vị, suy ra S r2 +1 − S r2 = m ⇒ m k , điều này vô lí. 2. Một hình chữ nhật có thể được phủ bởi 1 monomino và một số k – omino thẳng khi nào? Nếu được thì đặt monomino ở vị trí nào? Bài toán: Nếu hình chữ nhật m × n được phủ bởi 1 monomino và một số k – omino thẳng khi và chỉ khi ( mn − 1) k . Trường THPT Chuyên Thái Bình 161 HỘI CÁC TRƯỜNG THPT CHUYÊN KHU VỰC DUYÊN HẢI - ĐỒNG BẰNG BẮC BỘ HỘI THẢO KHOA HỌC LẦN THỨ VI Nếu hình chữ nhật m × n được phủ bởi 1 monomino và một số k – omino thẳng thì hiển nhiên ta có ( mn − 1) k . Ta chứng minh điều ngược lại Xét trường hợp với k = 2 , suy ra m , n đều lẻ. Ta xếp monomino ở một ô thuộc cột hoặc dòng có thứ tự lẻ (tính từ dòng đầu hoặc cột đầu) Với k > 2 . Vị trí của ô vuông đơn vị là giao của dòng x và cột y được kí hiệu ( x, y ) . m ≡ n ≡ 1( mod k )  m ≡ n ≡ −1( mod k ) Bài toán được giải quyết khi  hoặc   x ≡ y ≡ 1( mod k )  x ≡ y ≡ 0 ( mod k ) Một số bài toán Bài 1: Cho hình chữ L như hình vẽ . Chứng minh rằng không thể phủ hình này bởi các trimino thẳng. Ta vẫn giải quyết bài toán theo cách như đã dùng với monomino thẳng Tô màu như hình dưới Một trimino thẳng khi đặt vào hình sẽ phủ hoặc 3 ô cùng màu, hoặc 3 ô có đủ cả 3 màu. Gọi S1 ; S 2 ; S 3 tương ứng là số ô bị phủ mang màu đen, gạch và trắng. Muốn phủ được thì phải có điều kiện S1 ≡ S 2 ≡ S 3 ( mod 3 ) ( * ) Ta tính cụ thể các giá trị từ hình vẽ S1 = 30; S 2 = 20; S 3 = 25 Vậy điều kiện (*) không thỏa mãn, nghĩa là không phủ được. Trường THPT Chuyên Thái Bình 162 HỘI CÁC TRƯỜNG THPT CHUYÊN KHU VỰC DUYÊN HẢI - ĐỒNG BẰNG BẮC BỘ HỘI THẢO KHOA HỌC LẦN THỨ VI Bài 2: Cho hình chữ nhật 7 × 7 . Có thể bỏ đi một ô nào để phần còn lại có thể được phủ bởi các quân trimino thẳng (1× 3) . Giải Khi làm bài toán, chúng ta sẽ dùng tư tưởng cách làm của dạng 5 phần 2 để giải quyết. Nhận thấy 7 ≡ 7 ≡ 1 ( mod 3 ) Vậy ta có thể bỏ đi những ô là giao của dòng x, cột cột y thỏa mãn x ≡ y ≡ 1( mod 3 ) . Cần chỉ ra hai ý: Ý 1: Tồn tại cách phủ với mọi cách bỏ đi 1 trong các ô thỏa mãn điều trên. Đánh số các ô như hình vẽ, ta sẽ bỏ đi các ô bị bôi đen. 1 2 3 1 2 3 1 Dễ thấy được cách phủ với các quân (1× 3) . 1 2 3 1 2 3 1 Ý 2: Chứng minh nếu bỏ đi một ô nào khác thì không 1 2 3 1 2 3 1 1 2 3 1 2 3 1 1 2 3 1 2 3 1 1 2 3 1 2 3 1 1 2 3 1 2 3 1 thể phủ được Gọi Si là số các ô được đánh số i sau khi bỏ đi 1 ô Suy ra điều kiện để phủ được là S1 ≡ S 2 ≡ S3 ( mod 3 ) (điều này đã được nêu rõ trong chứng minh định lý: Mỗi quân (1× 3) khi đặt vào hình chữ nhật thì phủ được 3 ô cùng số hoặc 3 ô có đầy đủ 3 số) Mà số các ô 2 và 3 là bằng nhau nên không thể bỏ đi ô được đánh một trong hai số này. Điều này có nghĩa ta sẽ phải bỏ đi ô được đánh số 1. Quay ô vuông một góc 90o và tìm giao của các dòng, cột mang số 1 ta sẽ được các ô cần tìm. Bài 3 (Russia 1996): Cho hình chữ nhật 5 × 7 . Một số người phủ các ô vuông đơn vị của hình chữ nhật bởi các L – trimino. Hỏi có thể xảy ra trường hợp mỗi ô vuông đơn vị của hình chữ nhật được phủ bởi cùng số lượng L – trimino (ở đây mỗi người thực hiện việc phủ một số ô vuông đơn vị của mình, để nguyên như thế đến lượt người tiếp theo lại phủ các ô vuông đơn vị, khi đó mỗi ô có thể được phủ nhiều lần hoặc không được phủ lần nào). Giải x x x Đánh dấu các ô như hình vẽ Trường THPT Chuyên Thái Bình 163 x HỘI CÁC TRƯỜNG THPT CHUYÊN KHU VỰC DUYÊN HẢI - ĐỒNG BẰNG BẮC BỘ HỘI THẢO KHOA HỌC LẦN THỨ VI Nhận thấy mỗi L – trimino khi đặt vào hình chữ nhật sẽ chỉ phủ được tối đa 1 ô được đánh dấu. Giả sử mỗi ô của hình chữ nhật được phủ bởi đúng k quân L – trimino, suy ra số lượng L – trimino tối thiểu là 1 2 k . x x x x x x x x Nghĩa là sẽ phủ được không ít hơn 36 k ô đơn vị của hình chữ nhật (các ô có thể được đếm nhiều lần) Bên cạnh đó chỉ có đúng 35k ô đơn vị được phủ, số lượng này nhỏ hơn 36 k m suy ra vô lí. Bài 4 (Korea 2004): Có thể hay không để phủ một hình chữ nhật 4 × 1 1 bởi các L – tetramino? Giải Theo dạng 4: Hình chữ nhật m × n được phủ chỉ bởi các L – tetramino nếu và chỉ nếu m .n 8 . Do 4.11 = 44 8 suy ra không phủ được. Bài 5: Ở hai góc trên cùng của một mảnh đất hình vuông có kích thước m × m ( 2 < m ∈ ) (được chia thành m2 ô vuông đơn vị), người ta trồng hai cột trụ, mỗi cột lấy đi 1 phần đất hình chữ nhật có kích thước 2 × 1 . Cần lát gạch toàn bộ phần đất còn lại của nền đất, được phép sử dụng các viên gạch hình L – tetramino (có thể xoay nhưng không được lật). Với các giá trị nào của m thì có thể thực hiện được việc lát. Giải Ta cần chỉ ra hai điều: Điều kiện cần của m và với điều kiện đó ta có thể phủ được Nhận thấy m2 − 4 4 ⇒ m chẵn Với cách tô màu đen và trắng như hình vẽ. Bằng cách chứng minh như trên, ta được số lượng L – Tetramino cần dủng phải là số chẵn. Trường THPT Chuyên Thái Bình 164 HỘI CÁC TRƯỜNG THPT CHUYÊN KHU VỰC DUYÊN HẢI - ĐỒNG BẰNG BẮC BỘ HỘI THẢO KHOA HỌC LẦN THỨ VI Suy ra m 2 − 4 8 ⇒ m = 4k + 2, k ∈ Với m = 4 k + 2, k ∈ * * ta chỉ ra cách phủ Xét hình chữ nhật gồm 2 dòng đầu, có dạng 2 × ( 4k ) có tích hai cạnh chia hết cho 8, nên phủ được. Phần còn lại là hình chữ nhật ( 4 k ) × ( 4 k + 2 ) có tích hai cạnh chia hết cho 8, nên phủ được. Vậy điều kiện cần và đủ là m = 4 k + 2, k ∈ * Bài 6 (IMO Shortlist 2002): Cho n là số nguyên dương lẻ. Tô màu hình vuông n × n bởi màu đen và trắng xen kẽ (như hình ảnh của bàn cờ vua) sao cho 4 góc là 4 ô được tô đen. Tìm n để có thể phủ tất cả các ô đen bởi L – trimino. Trong trường hợp phủ được hãy tìm số lượng L – trimino tối thiểu phải dùng. Giải Nhận xét với n = 1,3 không thỏa mãn. Với n = 5 . Đánh dấu các ô như hình vẽ x x x Suy ra phải có tối thiếu 9 L – trimino , suy ra số ô vuông đơn vị sinh ra là không ít hơn 27 ô, số lượng này lớn hơn thực tế là 25 ô. x x x Với n = 7 , ta chỉ ra được một cách phủ thỏa mãn, cần dùng 16 quân L – trimino. x x x Chứng minh với mọi n ≥ 7 ta có thể phủ được, số lượng L –trimino tối thiểu là ( n + 1) 4 2 . Giả sử hình vuông k × k được phủ bởi tối thiểu ( k + 1) 4 2 . Với hình vuông ( k + 2 ) × ( k + 2 ) được sinh ra từ hình vuông k × k bằng cách gắn thêm vào các hình chữ nhật 2 × k và (k + 2) × 2 , k + 2 quân L – trimino. Trường THPT Chuyên Thái Bình 165 hai hình chữ nhật này cần tối thiểu HỘI CÁC TRƯỜNG THPT CHUYÊN KHU VỰC DUYÊN HẢI - ĐỒNG BẰNG BẮC BỘ HỘI THẢO KHOA HỌC LẦN THỨ VI Vậy số lượng L – trimino tối thiểu cần dùng là ( k + 1) 4 2 ( k + 3) +k+2= 4 2 , ta có điều phải chứng minh. Bài toán 7 (VMO – 2006). Xét bảng ô vuông m × n ( m , n ≥ 3 ) . Thực hiện trò chơi sau: mỗi lần đặt 4 viên bi vào 4 ô của bảng, mỗi ô một viên bi, sao cho 4 ô đó tạo thành một trong các hình dưới đây: Hỏi sau một số lần ta có thể nhận được bảng mà số bi trong các ô bằng nhau được không nếu: a) m = 2004, n = 2006? b) m = 2005, n = 2006 ? Giải a) Bảng đã cho có thể chia thành các hình chữ nhật 4 × 2 nên có thể nhận được trạng thái mà số bi trong các ô bằng nhau. b) Tô màu các ô như hình vẽ Dễ thấy, mỗi lần đặt bi có 2 viên được đặt vào các ô màu đen và 2 viên được đặt vào ô màu trắng. Do đó, nếu gọi S ( n ) là số bi trong các ô màu đen và T ( n ) là số bi trong các ô màu trắng sau lần đặt bi thứ n thì S ( n ) − T ( n ) là đại lượng bất biến. Ta có S ( n ) − T ( n ) = S ( 0 ) − T ( 0 ) = 0, ∀ n ≥ 0. Vì m = 2 0 0 5 là số lẻ nên nếu nhận được trạng thái mà số bi trong các ô bằng nhau thì S ( n ) − T ( n ) = n = 2006 vô lý. Trường THPT Chuyên Thái Bình 166 HỘI CÁC TRƯỜNG THPT CHUYÊN KHU VỰC DUYÊN HẢI - ĐỒNG BẰNG BẮC BỘ HỘI THẢO KHOA HỌC LẦN THỨ VI Nhận thấy câu hỏi ý 2 tương tự bài Russia 1996, ta suy nghĩ hướng giải dựa trên tư tưởng đó, và có lời giải thứ 2. Lời giải 2 Đánh dấu các ô như hình vẽ, khi đó mỗi hàng và mỗi cột đều có 1003 viên bi x x x x x x x x x Nhận thấy mỗi lần đặt bi có nhiều nhất 1 ô trong số các ô được đánh dấu sẽ có bi. Giả sử mỗi ô của hình chữ nhật được đặt đúng k viên bi, suy ra số lượng bi tối thiểu là 1 0 0 3 .1 0 0 3 .4 k . Bên cạnh đó chỉ có đúng 2 0 0 5 .2 0 0 6 k 1 0 0 3 .1 0 0 3 .4 k suy ra vô lí. viên bi được đặt, số lượng này nhỏ hơn Bài 8 (IMO – 2004): Ta định nghĩa viên gạch hình móc câu là hình gồm 6 ô vuông đơn vị như hình vẽ dưới đây, hoặc hình nhận được do lật hình đó (sang trái, sang phải, lên trên, xuống dưới) hoặc hình nhận được do xoay hình đó đi một góc. Hãy xác định tất cả các hình chữ nhật m× n , trong đó m, n là các số nguyên dương sao cho có thể lát hình chữ nhật đó bằng các viên gạch hình móc câu? Lời giải. Dễ thấy m , n ∉ {1; 2; 5} . Chi hình chữ nhật đã cho thành các m × n ô vuông và đánh số các hàng, các cột từ dưới lên trên, từ trái sang phải. Ta gọi ô ( p ; q ) là ô nằm ở giao của hàng thứ p và cột thứ q. Hai viên gạch hình móc câu chỉ có thể ghép lại để được một trong hai hình dưới đây: (H2) (H1) Trường THPT Chuyên Thái Bình 167 HỘI CÁC TRƯỜNG THPT CHUYÊN KHU VỰC DUYÊN HẢI - ĐỒNG BẰNG BẮC BỘ HỘI THẢO KHOA HỌC LẦN THỨ VI Do đó, để lát được hình chữ nhật m × n thì m.n phải chia hết cho 12. Nếu ít nhất một trong hai số m, n chia hết cho 4 thì có thể lát được. Thật vậy, giả sử được m chia hết cho 4. Ta có thể viết n dưới dạng: n = 3 a + 4 b , do đó có thể lát được. Xét trường hợp m, n đều không chia hết cho 4. Ta chứng minh trường hợp này không thể lát được. Giả sử ngược lại, khi đó m, n đều chia hết cho 2 nhưng không chia hết cho 4. Ta tạo bất biến như sau: Xét ô ( p ; q ) . Nếu chỉ một trong hai toạ độ p, q chia hết cho 4 thì điền số 1 vào ô đó. Nếu cả hai toạ độ p, q chia hết cho 4 thì điền số 2. Các ô còn lại điền số 0. Với cách điền số như vậy ta thu được bất biến là tổng các số trong hình (H1) và tổng các số trong hình (H2) đều là số lẻ. Do m, n chắn nên tổng các số trong toàn bộ hình chữ nhật m × n là số chẵn. Để lát được thì tổng số hình (H1) và (H2) được sử dụng phải là số chẵn. Khi đó, m.n chia hết cho 24, vô lý. Bài 9 (Belarus 1999): Có 1 nền đất hình vuông 7 × 7 được chia thành 49 ô vuông đơn vị. Thực hiện việc lát nền đất bởi 3 loại gạch lát nền: Loại 1: Monomino ( 1 × 1 ) Loại 2: Trimino thẳng ( 1 × 3 ) Loại 3: L – trimino (có 3 ô đơn vị, hình chữ L) Giả sử A có 1 viên loại 3 và rất nhiều viên loại 2, B có 1 viên loại 1. a. Chứng minh rằng B có thể lát viên gạch của mình lên một ô vuông đơn vị nào đó trên nền đất mà A không thể lát kín phần còn lại. b. Giả sử A có thêm 1 viên loại 3 nữa. Chứng minh rằng dù B lát viên gạch của mình ở đâu thì A cũng lát kín được phần còn lại. Giải Đánh số các ô vuông đơn vị của hình chữ nhật như hình vẽ, ô màu đen là ô B lát viên gạch của mình Số các số 1 là 17, số 2 là 15, số 3 là 16 Nhận xét về các quân của A khi đặt nên hình chữ nhật này: Trường THPT Chuyên Thái Bình 168 HỘI CÁC TRƯỜNG THPT CHUYÊN KHU VỰC DUYÊN HẢI - ĐỒNG BẰNG BẮC BỘ HỘI THẢO KHOA HỌC LẦN THỨ VI Với Trimino thẳng, sẽ phủ được 3 ô có đủ 3 số lượng 1, 2 và 3. L – trimino sẽ phủ được hoặc 3 số 1, 2, 3 hoặc 3 ô có 2 số giống nhau (ví dụ 2 ô 1 và 1 ô 3…) Nếu A chỉ dùng Trimino thẳng thì hiển nhiên không phủ được. Nếu A dùng thêm L – trimino thì lúc đó có hai trường hợp • Phủ được 3 ô 1, 2, 3 còn lại không phủ được bởi trimino thẳng, hoặc phủ được 2 ô giống nhau và 1 ô khác thì • Nếu dùng L – trimino mà phủ 2 ô giống nhau và 1 ô khác, giữ nguyên mô hình đó nhưng ta đánh số lại như hình sau, số lượng số 1, 2, 3 không thay đổi Khi đó với cách đặt quân L – trimino như trường hợp này lại chuyển về việc phủ được 3 ô có đủ cả 1, 2, 3 Như thế ta đưa về trường hợp trên Tóm lại, A không thể phủ được hình chữ nhật bằng các quân mình có. b. Phân chia theo các khối hình sau Nhận thấy dù ô vuông đơn vị của B đặt vào đâu thì cũng có thể phủ được bằng 3 khối hình này. Chú ý ta có thể xoay các hình này để phủ hợp với việc phủ. Bài 10 (Canada 2007): Cho hình chữ nhật 8 × 9 bỏ đi các ô được đánh dấu x. Hỏi phải dùng tối đa bao nhiêu domino để phủ phần còn lại. Giải Trường THPT Chuyên Thái Bình 169 HỘI CÁC TRƯỜNG THPT CHUYÊN KHU VỰC DUYÊN HẢI - ĐỒNG BẰNG BẮC BỘ HỘI THẢO KHOA HỌC LẦN THỨ VI Đánh số các ô vuông đơn vị như hình vẽ. 1 2 1 2 1 2 1 2 1 Nhận thấy mỗi domino khi đặt vào hình chữ nhật sẽ phủ được đúng hai ô mang số 1 và 2. 2 1 2 1 2 1 x x 2 1 2 1 2 1 x x 2 1 Ta đếm số lượng số 1 và 2 sẽ suy được số lượng tối đa các quân domino cần dùng. 2 1 2 1 x x 2 1 2 1 2 1 x x 2 1 2 1 2 1 x x 2 1 2 1 2 1 x x 2 1 2 1 2 1 2 1 2 1 2 1 2 1 2 Nhận thấy số lượng số 1 và 2 bằng nhau và bằng 28 Vậy số lượng tối đa các quân domino cần dùng là 28. Một số bài tập tham khảo Bài 11: Tìm n để hình vuông n × n có thể được phủ bởi các hình chữ nhật 2 × 2 và 3×3 . Bài 12: Tìm n ≥ 3 để hình vuông n × n cắt bỏ đi 4 ô vuông đơn vị ở 4 góc có thể được phủ bởi các quân L – tetramino. Bài 13: Có thể phủ được hình vuông 1 3 × 1 3 bỏ đi ô chính giữa bởi các hình chữ nhật 1 × 4 hay không? Bài 14 (Đề đề nghị DHĐBBB 2012 của Hải Phòng): Có bao nhiêu cách lát kín hình “hàm răng 2012 chiếc” (hình vẽ) bằng các quân đôminô 1 × 2 sao cho các quân đôminô không đè lên nhau và cũng không đè lên các đường tô đậm? 4 2 2012 Bài 15 (Tournament of the Towns 1984): Chứng minh rằng có thể đặt hình vuông 1 × 1 vào một ô vuông đơn vị của hình chữ nhật 7 × 7 sao cho phần còn lại có thể được phủ bởi 16 hình chữ nhật 1 × 3 Trường THPT Chuyên Thái Bình 170 HỘI CÁC TRƯỜNG THPT CHUYÊN KHU VỰC DUYÊN HẢI - ĐỒNG BẰNG BẮC BỘ HỘI THẢO KHOA HỌC LẦN THỨ VI Tài liệu tham khảo [1]. Alexander Soifer, Geometric Etudes in Combinatorial Mathematics,Springer 2010 [2].Michael Korn and Igor Pak, Combinatorial evaluations of the Tutte polynomial, Department of Mathematics Massachusetts Institute of Technology Cambridge, MA, 02139, August 18, 2003 [3]. Titu Andresscu, Zuming Feng, A path to combinatorics for undergranduates, Birkhauser, 2007. [4]. Titu Andresscu, Zuming Feng, 102 combinatorial Problems from the Training of the USA IMO Tearm, Birkhauser, 2002. [5]. Titu Andresscu, Zuming Feng, Mathematical Olympiads Ptoblems and Solution from around the World, to 1995 from 2002. [6]. Arthur Engel, Problem – Solving Strategies, Springer, 1999. [7] www.mathlinks.ro Trường THPT Chuyên Thái Bình 171 HỘI CÁC TRƯỜNG THPT CHUYÊN KHU VỰC DUYÊN HẢI - ĐỒNG BẰNG BẮC BỘ HỘI THẢO KHOA HỌC LẦN THỨ VI CHUYÊN ĐỀ MỘT SỐ BÀI TOÁN HÌNH HỌC GIẢI BẰNG PHƯƠNG PHÁP TỌA ĐỘ Người viết: Trịnh Thị Thanh Bình Trường THPT Chuyên Biên Hòa Hình học phẳng là một lĩnh vực quan trọng trong toán sơ cấp ở bậc trung học phổ thông. Chúng ta gặp các bài toán về hình học phẳng trong rất nhiều kì thi quan trọng: thi học sinh giỏi tỉnh, thi học sinh giỏi quốc gia, quốc tế... Tuy nhiên để làm được những bài toán đó thì không hề đơn giản, đòi hỏi phải có một vốn kiến thức phong phú và một tư duy linh hoạt. Bài viết này giới thiệu một trong những phương pháp giải toán hình học phẳng đó là phương pháp tọa độ. Trong nhiều bài toán hình học nếu đưa về phương pháp tọa độ thì bài làm sẽ sáng sủa và rõ ràng hơn cách mà chúng ta dùng tính chất hình học thuần túy. Ta sẽ thấy phương pháp tọa độ không chỉ đơn thuần áp dụng cho các bài toán liên quan đến hình vuông, hình chữ nhật mà còn có thể áp dụng cho các bài toán liên quan đến tam giác, đến đường tròn… Phương pháp - Chọn hệ trục tọa độ thích hợp tùy theo bài toán sao cho việc tính toán đơn giản. Tìm tọa độ các đối tượng đã cho và các đối tượng liên quan theo hệ trục đã chọn. Chuyển các tính chất hình học trong giả thiết và điều cần chứng minh theo các công thức tọa độ. Chứng minh bài toán theo phương pháp tọa độ. Trước hết ta xét một số bài liên quan đến hình vuông. Hệ trục tọa độ có thể lấy gốc là một trong bốn đỉnh của hình vuông hoặc là tâm của hình vuông. Ta xét hai bài toán sau. Trường THPT Chuyên Thái Bình 172 HỘI CÁC TRƯỜNG THPT CHUYÊN KHU VỰC DUYÊN HẢI - ĐỒNG BẰNG BẮC BỘ HỘI THẢO KHOA HỌC LẦN THỨ VI Bài 1 (Iran, 2001). Về phía bên trong hình vuông ABCD, ta dựng các tam giác đều ABK, BCL, CDM, DAN. Chứng minh rằng các trung điểm của KL, LM, MN, NK và các trung điểm của AK, BK, BL, CL, CM, DM, DN, AN tạo thành một đa giác đều 12 cạnh. Lời giải. Gọi O là tâm hình vuông, lập hệ trục xOy sao cho các điểm A, B, C, D lần lượt có tọa độ là (1; 1), (-1; 1), (-1; -1), (1; -1). Khi đó, dễ dàng tính được tọa độ các điểm K, L, M, N lần lượt là (0; -2k), (2k; 0), (0; 2k), (-2k; 0) với k = 3 −1 . Từ đó ta có tọa độ 2 các trung điểm E, F, G, H tương ứng của KL, y B A M N O x L K C D LM, MN, NK là (k; -k), (k; k), (-k; k), (-k; -k). Suy ra rằng các khoảng cách từ E, F, G, H đến O bằng nhau và bằng k 2 , đồng thời các vectơ gốc O, điểm mút tương ứng là E, F, G, H hợp với trục hoành các góc lần lượt là 315o , 45o , 135o , 225o. Tiếp đến, từ tọa độ đã xác định được của các điểm trên, ta dễ dàng tình được tọa độ tương ứng của các trung điểm P, Q, R, S, R, U, V, X của các cạnh AK, BK, BL, CL, CM, DM, DN, AN lần lượt là (h; j), (-h; j), (-j; h), (-j; -h), (-h; -j), (h; -j), (j; -h), (j; h), 1 2 ở đây h = , j = 1 − 3 . 2 Suy ra các điểm P, Q, R, S, R, U, V, X cách O một đoạn bằng Trường THPT Chuyên Thái Bình 173 h2 + j2 = 3 −1 = k 2. 2 HỘI CÁC TRƯỜNG THPT CHUYÊN KHU VỰC DUYÊN HẢI - ĐỒNG BẰNG BẮC BỘ HỘI THẢO KHOA HỌC LẦN THỨ VI Các điểm này cũng là đầu mút của những vectơ gốc O lần lượt hợp với trục hoành các góc tương ứng 15o , 165o , 105o , 255o , 195o , 345o , 285o , 75o. Tiếp đến ta cần xét các góc của tam giác vuông có 3 cạnh là k, h, j. Góc x giữa h và k j k h k có s inx = , cos x = . Do đó sin 2 x = 2 sin x cos x = 2hj 1 = . Suy ra x = 15o . Như vậy 12 k2 2 điểm nói trên cách đều gốc tọa độ O và là đầu mút của những vectơ gốc O hợp với trục hoành các góc 15o + 30n với n=0, 1, 2,…, 11. Từ đó suy ra rằng chúng lập thành một đa giác đều 12 cạnh. Bài 2. Cho hình vuông ABCD, E là trung điểm của BC. Điểm M tùy ý thuộc AB, P là giao điểm của AE và CM, N là giao điểm của MD và AE, H là giao điểm của DP và CN, I là giao điểm của đường trung trực của PH và đường vuông góc với AH tại H. Chứng minh rằng I thuộc một đường cố định. Lời giải. Chọn hệ trục tọa độ Oxy có O trùng với A, trục Ox đi qua B, Oy đi qua D. Giả sử cạnh của hình vuông là 1. Suy ra A(0; 0), B(1; 0), C(1; 1), D(0; 1), E(1; ½). Phương trình của đường thẳng AE: x - 2y = 0. Phương trình của đường thẳng DM: x + my – m = 0. Phương trình của đường thẳng CM: x + (m-1)y – m = 0. y D C I E H N A Trường THPT Chuyên Thái Bình P B M 174 x HỘI CÁC TRƯỜNG THPT CHUYÊN KHU VỰC DUYÊN HẢI - ĐỒNG BẰNG BẮC BỘ HỘI THẢO KHOA HỌC LẦN THỨ VI  Vì N là giao điểm của MD và AE suy ra N  ;  . P là giao điểm của AE và m 2 m 2 + +   2m m CM suy ra P  2m m  ; .  m +1 m +1 Đường thẳng DP có phương trình: x + 2my - 2m = 0. Đường thẳng NC có phương trình: 2x + (m-2)y - m(m-2) = 0.  H là giao của DP và CN suy ra H  ; .  3m + 2 m + 1  4m m AH có phương trình 3x - 4y = 0. Ta thấy ID = IH hay ID = d(I, AD). Từ đó suy ra I thuộc đường parabol với tiêu điểm D, AH là đường chuẩn. Phương pháp tọa độ cũng có thể áp dụng với các bài toán về tam giác. Với các tam giác vuông ta thường chọn hệ trục tọa độ có hai trục là hai cạnh góc vuông, với các tam giác thường nếu có sẵn hai đường vuông góc tại cùng một điểm trong giả thiết rồi thì chọn hai đường đó là hai trục tọa độ, nếu chưa có thì ta nên kẻ thêm đường cao và chọn hệ trục có một trục là đường cao, một trục là cạnh tương ứng của tam giác. Ta xét các bài toán sau. Bài 3 ( VMO 2007-2008). Cho tam giác ABC, trung tuyến AD. Một đường thẳng d vuông góc với AD. Xét M thuộc d. Gọi E, F là trung điểm của MB, MC. Đường thẳng qua E, vuông góc với d cắt AB tại P, đường thẳng qua F vuông góc với d cắt AC tại Q. Chứng minh rằng đường thẳng qua M, vuông góc với PQ luôn đi qua một điểm cố định khi M thay đổi trên d. Lời giải. Trường THPT Chuyên Thái Bình 175 HỘI CÁC TRƯỜNG THPT CHUYÊN KHU VỰC DUYÊN HẢI - ĐỒNG BẰNG BẮC BỘ HỘI THẢO KHOA HỌC LẦN THỨ VI y P A x M E F B C D Q Rõ ràng chỉ cần xét d ⊥ AD tại D là đủ. Chọn hệ trục Dxy như hình vẽ sao cho A(0; a), C(2m; 2n), M(2xo, 0). Do B, C đối xứng với nhau qua D nên B(-2m; -2n). Phương trình của AB: (2n+a)x – 2my + 2ma = 0. Phương trình của AC: (2n - a)x – 2my + 2ma = 0. Từ đó suy ra (2n − a )( xo − m ) (2n − a )( xo + m )     + a  , Q  xo + m; + a P  xo − m; 2m 2m     ax ⇒ PQ (2m;2n − o ) m Đường thẳng đi qua M, vuông góc với PQ có phương trình ax   2m( x − 2 xo ) +  2n − o  y = 0 . m    4mn 4m 2  ;− với mọi xo. Ta có điều phải a   a Dễ thấy đường thẳng này luôn đi qua S  chứng minh. Bài 4 (VMO 2006 - 2007). Cho tam giác ABC có hai đỉnh B, C cố định và đỉnh A thay đổi. Gọi H, G lần lượt là trực tâm và trọng tâm của tam giác ABC. Tìm quỹ tích của điểm A biết rằng trung điểm K của HG thuộc đường thẳng BC. Lời giải. Trường THPT Chuyên Thái Bình 176 HỘI CÁC TRƯỜNG THPT CHUYÊN KHU VỰC DUYÊN HẢI - ĐỒNG BẰNG BẮC BỘ HỘI THẢO KHOA HỌC LẦN THỨ VI Chọn hệ trục Oxy với O là trung điểm của BC, trục Ox là đường thẳng BC. Đặt BC = 2a > 0. Khi đó B(-a; 0), C(a; 0). Giả sử A(xo; yo) yo ≠ 0 . y A G B K O x C H Khi đó trực tâm H là nghiệm của hệ phương trình  x = xo  a 2 − xo 2  ⇒ H  xo ;  . yo  ( x + a )( a − xo ) − yo y = 0   2 x 3a 2 − 3xo2 + yo2  . 6 yo  Trọng tâm G  o ; o  , suy ra trung điểm K  o ;  3 3  3 x y K thuộc đường thẳng BC khi và chỉ khi 3a 2 − 3xo2 + yo2 = 0 ⇔ Vậy quỹ tích điểm A là Hyperbol xo2 yo2 − = 1( yo ≠ 0). a 2 3a 2 x2 y2 − = 1 bỏ đi hai điểm B, C. a 2 3a 2 Bài 5. Cho tam giác ABC nhọn , d là đường thẳng thay đổi. Gọi D, E, F là hình chiếu vuông góc của A, B, C trên d. Biết rằng AD 2 tan A+BE 2 tan B + CF 2 tan C = 2 S ∆ABC . Tìm vị trí của d để AD đạt giá trị lớn nhất. Lời giải. Gọi AO là đường cao của tam giác ABC. Chọn hệ trục Oxy có Ox đi qua C, Oy đi qua A. Ta sẽ chứng minh đường thẳng d đi qua trực tâm H của tam giác ABC. Giả sử A(0; a), B(-b; 0), C(c; 0) Trường THPT Chuyên Thái Bình 177 HỘI CÁC TRƯỜNG THPT CHUYÊN KHU VỰC DUYÊN HẢI - ĐỒNG BẰNG BẮC BỘ HỘI THẢO KHOA HỌC LẦN THỨ VI tan B = a a , tan C = b c t anA = − tan( B + C ) = S ∆ABC = ac + ab a 2 − bc 1 a (b + c ) 2 Gọi phương trình đường thẳng d là x + y + = 0( 2 + 2 ≠ 0). y E A D H F O B C x Ta có AD = a+ 2 + 2 , BE = b+ 2 + 2 , AD = Đẳng thức đề bài cho tương đương với c+ 2 + bc a 2 . + = 0. Từ đó suy ra d luôn đi qua điểm  bc  H  0;  . Dễ chứng minh được H là trực tâm của tam giác ABC. Ta có AD ≤ AH .  a  Vậy ADmax = AH khi và chỉ khi d là đường thẳng đi qua H và song song với BC. Bài 6. Cho tam giác ABC vuông cân tại A, gọi M là trung điểm của BC, G là điểm trên cạnh AB sao cho GB=2GA. Các đường thẳng GM và CA cắt nhau tại D. Đường thẳng qua M vuông góc với CG tại E và cắt AC tại K. Gọi P là giao điểm của DE và GK. Chứng minh rằng: Trường THPT Chuyên Thái Bình 178 HỘI CÁC TRƯỜNG THPT CHUYÊN KHU VỰC DUYÊN HẢI - ĐỒNG BẰNG BẮC BỘ HỘI THẢO KHOA HỌC LẦN THỨ VI a) DE = BC. b) PG = PE. Lời giải. Chọn hệ trục Oxy như hình vẽ, giả sử AB =AC=1. Ta có A(0; 0), B(0; 1), C(1; 0),  1 1 1 G  0;  , M  ;  , D ( −1;0) . Gọi (d) là đường thẳng đi qua M và vuông góc với GC.  3 2 2 Đường thẳng (d) có phương trình: 3x – y – 1 = 0. Đường thẳng GC có phương trình: x + 3y -1 = 0.  2 1 E = ( d ) GC ⇒ E  ;   5 5 Suy ra BC = DE = 2 . y B M G E P D K A b) Vì K  ;0  ⇒ GK BC. 1 3  Phương trình của đường thẳng GK: 3 x − 3 y + 1 = 0 . Phương trình của đường thẳng DE: − x + 7 y − 1 = 0 . Vì P là giao điểm của DE và GK nên suy ra D  ;  . 6 6 1 1 Trường THPT Chuyên Thái Bình 179 C x HỘI CÁC TRƯỜNG THPT CHUYÊN KHU VỰC DUYÊN HẢI - ĐỒNG BẰNG BẮC BỘ HỘI THẢO KHOA HỌC LẦN THỨ VI Do đó PG = PE = 1 3 2 . Bài 7. Cho tam giác ABC có hai đỉnh B, C cố định và đỉnh A thay đổi. Qua B dựng đường thẳng d vuông góc với BC, d cắt đường trung tuyến AI của tam giác ABC tại K. Gọi H là trực tâm của tam giác ABC. Tìm quỹ tích của A biết rằng IH song song với KC. Lời giải. y A H B I C x K Chọn hệ trục Oxy với O trùng I, trục Ox là đường thẳng BC. Đặt BC=2a > 0. Khi đó B(-a; 0), C(a; 0). Giả sử A (xo; yo), yo ≠ 0 . Khi đó trực tâm H là nghiệm của hệ  x = xo  a 2 − xo2  ⇒ H  xo ; . yo  ( x + a )( a − xo ) − yo y = 0  phương trình   y  K là giao của d và AI nên K  −a; −a o  , xo ≠ 0 . xo   Theo giả thiết ta có IH cùng phương với KC . Điều này tương đương với a x2 y2 yo a 2 − xo2 x2 y2 = 0 ⇔ o2 + o2 = 1. Vậy quỹ tích điểm A là Elip 2 + 2 = 1 bỏ đi . xo − 2a a 2a xo yo a 2a bốn điểm B, C, A1 (0; −a 2), A2 (0; a 2) . Trường THPT Chuyên Thái Bình 180 HỘI CÁC TRƯỜNG THPT CHUYÊN KHU VỰC DUYÊN HẢI - ĐỒNG BẰNG BẮC BỘ HỘI THẢO KHOA HỌC LẦN THỨ VI Ngoài ra, phương pháp tọa độ cũng có thể áp dụng với các bài toán về hình tròn. Gốc tọa độ thường được chọn là tâm của hình tròn. Bài 8 (IMO, 1999). Cho hai đường tròn (C1) và (C2) nằm bên trong và tiếp xúc với đường tròn (C) theo thứ tự tại M và N. Giả sử (C1) đi qua tâm của (C2). Đường nối hai điểm chung của (C1), (C2) cắt (C) tại A và B. Các đường thẳng MA, MB cắt (C1) tương ứng tại E và F. Chứng minh rằng đường thẳng EF là tiếp tuyến của (C2). N A I Y X V W E M K B F O Gọi O, K, I, r1, r2, r3 lần lượt là tâm và bán kính của các đường tròn (C), (C1), (C2) tương ứng. Giả sử EF cắt KI tại W, đặt IW = x. Ta cần chỉ ra x = r 2. Chọn hệ trực chuẩn có gốc là I, IK là trục hoành, giả sử O(a; b). Giả sử AB cắt IK tại V. Gọi X là một giao điểm của (C1) và (C2), Y là trung điểm của IX. Dễ thấy IV = KYI và XVI đồng dạng suy ra r2 2 . Tam giác 2 r1 IV IY r . Phép vị tự tâm M, tỉ số biến K thành O, = IX IO r1 biến EF thành AB. Do đó EF ⊥ IK . Cũng vậy khoảng cách từ K đến EF bằng r r1 lần r r2  khoảng cách từ O đến AB, suy ra r1 − x = 1  a − 2  (*) . Bây giờ ta cần xác định a. r 2r  1  Bằng cách tính khoảng cách từ O đến I và K ta nhận được hai phương trình chứa a và b sau đây: Trường THPT Chuyên Thái Bình 181 HỘI CÁC TRƯỜNG THPT CHUYÊN KHU VỰC DUYÊN HẢI - ĐỒNG BẰNG BẮC BỘ HỘI THẢO KHOA HỌC LẦN THỨ VI ( r − r1 ) 2 = ( r − a ) 2 + b 2 ( r − r2 ) 2 = a 2 + b 2 Khử b từ hai phương trình trên ta có a = r22 r + r − r 2 . Thay vào (*) ta nhận được x = 2r1 r1 r2. Ta có điều phải chứng minh. Bài 9 (Olympic 30-04, 2001, đề đề nghị). Cho tứ giác IAJB có các góc A, B vuông, IA>IB. Chứng minh rằng với mọi M trên đường thẳng IJ ta luôn có JA MA IA . ≤ ≤ JB MB IB Lời giải. y A O I M J x B Vì góc A, B vuông nên IAJB là tứ giác nội tiếp được. Giả sử IAJB nội tiếp đường tròn có tâm O là trung điểm của IJ và bán kính R=1. Ta chọn hệ trục tọa độ vuông góc như hình vẽ. Khi đó, ta có I(-1; 0), J(1; 0). Do A, B thuộc đường tròn nên A(cosa, sina), B(cosb, sinb), a, b ∈ (0;2 ) . Ta có IA > IB ⇔ IA2 > IB 2 ⇔ ( −1 − cos a ) 2 + sin 2 a > ( −1 − cos b) 2 + sin 2 b ⇔ cos a > cos b. Trường THPT Chuyên Thái Bình 182 HỘI CÁC TRƯỜNG THPT CHUYÊN KHU VỰC DUYÊN HẢI - ĐỒNG BẰNG BẮC BỘ HỘI THẢO KHOA HỌC LẦN THỨ VI Giả sử M(x; 0), ta có Xét hàm số y = MA2 x 2 − 2 x cos a + 1 . = MB 2 x 2 − 2 x cos b + 1 x 2 − 2 x cos a + 1 x 2 − 2 x cos b + 1 Ta có 2( x 2 − 1)(cos a − cos b) ( x 2 − 2 x cos b + 1) 2 y ' = 0 ⇔ x = ±1 y'= Bảng biến thiên: −∞ x y’ -1 + 0 +∞ 1 - 0 + 1 y y(-1) 1 y(1) Ta có y ( −1) = 1 + cos a 1 − cos a > 1, y (1) = b. Xét tất cả các điểm P, Q sao cho AP=a, AQ=b và đường thẳng d là phân giác của góc PAQ. Ứng với mỗi cặp điểm P, Q xét điểm M sao cho AM = AP + AQ . Tìm quỹ tích các điểm M. Bài 7. Cho tam giác ABC vuông cân tại C. Trên các cạnh BC, CA, AB lần lượt lấy các điểm M, N, P sao cho MB NC PA . Chứng minh rằng CP ⊥ MN , CP = MN . = = MC NA PB Bài 8. Cho tam giác ABC nội tiếp đường tròn tâm I. Gọi D là trung điểm của AB, E là trọng tâm của tam giác ADC. Chứng minh rằng nếu AB=AC thì IE vuông góc với CD. Bài 9. Cho hình bình hành ABCD thay đổi trong đó A, D cố định thỏa mãn AC BD . Tìm tập hợp điểm B, C. = AD BA Trường THPT Chuyên Thái Bình 185 HỘI CÁC TRƯỜNG THPT CHUYÊN KHU VỰC DUYÊN HẢI - ĐỒNG BẰNG BẮC BỘ HỘI THẢO KHOA HỌC LẦN THỨ VI Bài 10. Cho đường tròn ( C ) tâm O và tiếp tuyến d tiếp xúc với (C) tại một điểm A cố định trên (C). M là một điểm trên mặt phẳng, kẻ tiếp tuyến MT với (C) ( T là tiếp điểm), gọi H là hình chiếu vuông góc của M trên d. a) Tìm quỹ tích các điểm M thỏa mãn MT=MH. b) Chứng minh rằng các đường tròn tâm M bán kính MT luôn tiếp xúc với một đường tròn cố định. Trường THPT Chuyên Thái Bình 186 HỘI CÁC TRƯỜNG THPT CHUYÊN KHU VỰC DUYÊN HẢI - ĐỒNG BẰNG BẮC BỘ HỘI THẢO KHOA HỌC LẦN THỨ VI CHUYÊN ĐỀ : MỘT SỐ BÀI TOÁN VỀ ỨNG DỤNG CỦA GRAPH KHI GIẢI TOÁN TỔ HỢP Giáo viên: Nguyễn Ngọc Xuân Trường: THPT chuyên Hoàng Văn Thụ Trường THPT Chuyên Thái Bình 187 HỘI CÁC TRƯỜNG THPT CHUYÊN KHU VỰC DUYÊN HẢI - ĐỒNG BẰNG BẮC BỘ HỘI THẢO KHOA HỌC LẦN THỨ VI Tỉnh: Hòa Bình. MỘT SỐ BÀI TOÁN VỀ ỨNG DỤNG CỦA GRAPH KHI GIẢI TOÁN TỔ HỢP Trường THPT Chuyên Thái Bình 188 HỘI CÁC TRƯỜNG THPT CHUYÊN KHU VỰC DUYÊN HẢI - ĐỒNG BẰNG BẮC BỘ HỘI THẢO KHOA HỌC LẦN THỨ VI I. Những khái niệm cơ bản: Graph là một mô hình toán học có thể dùng để giải quyết khá nhiều bài toán và vấn đề toán học. Một graph là hệ thống các đỉnh và các cạnh nối các đỉnh này với nhau. 1. Định nghĩa Graph và các ký hiệu cơ bản Định nghĩa: Một Graph được hiểu là một bộ phận tập hợp hữu hạn: Tập hợp các đỉnh và tập hợp cạnh nối các đỉnh này với nhau Thông thường ta hay ký hiệu một Graph bởi chữ G. Còn tập đỉnh được ký hiệu bởi chữ V, tập cạnh ký hiệu bởi chữ E Graph không có cạnh có hướng được ký hiệu là G = (V, E) còn graph chỉ có cạnh có hướng được ký hiệu là G = [V, E] Với hai đỉnh a và b của graph, ta ký hiệu cạnh không có hướng nối a với b bởi (a,b) và cạnh có hướng nối chúng bởi [a,b]. Trong trường hợp có nhiều cạnh nối a với b ta ký hiệu cạnh thứ n nối chúng bởi (a,b,n) hoặc [a,b,n] Hai đỉnh khác nhau của Graph được gọi là kề nhau hoặc láng giềng nếu chúng được nối với nhau bởi một cạnh Một Graph được gọi là vô hướng nếu các cạnh của chúng đều là cạnh vô hướng. Một Graph được gọi là có hướng nếu các cạnh của chúng đều là cạnh có hướng. Một Graph vừa có cạnh vô hướng vừa có cạnh có hướng được gọi là Graph hỗn độn. Một Graph được gọi graph đơn nếu nó không có khuyên và không cố cạnh kép. Graph điểm là graph chỉ có đúng một đỉnh và không có cạnh. Graph rỗng là graph không có đỉnh và không có cạnh nào cả 2. Bậc của đỉnh Trong phần này chúng ta chỉ xét các Graph vô hướng, tức là những graph mà các cạnh của chúng không có hướng. Ta gọi bậc của một đỉnh là số cạnh xuất phát từ đỉnh đó(các khuyên được tính gấp đôi). Đương nhiên bậc của một đỉnh là một số nguyên không âm. Một đỉnh được gọi là cô lập nếu nó không có cạnh nào cả, tức là khi đó nó có bậc là 0. Đỉnh có bậc 1 gọi là đỉnh treo. Định lý 2.1: Trong một Graph vô hướng G tùy ý tổng bậc của tất cả các đỉnh gấp đôi số cạnh của Graph Trường THPT Chuyên Thái Bình 189 HỘI CÁC TRƯỜNG THPT CHUYÊN KHU VỰC DUYÊN HẢI - ĐỒNG BẰNG BẮC BỘ HỘI THẢO KHOA HỌC LẦN THỨ VI Chứng minh: Trong mỗi graph tổng bậc các đỉnh của một graph thì mỗi cạnh được tính đúng hai lần bởi hai đỉnh của nó. Do đó tổng này gấp đôi số cạnh của graph Hệ quả 1:Trong một Graph vô hướng G tùy ý số đỉnh bậc lẻ luôn là một số chẵn. Chứng minh: Theo định lý trên thì tổng các bậc của các đỉnh luôn là một số chẵn do vậy số các đỉnh bậc lẻ luôn là một số chẵn. Hệ quả 2: Trong một graph vô hướng G có số lẻ đỉnh luôn có một số lẻ các đỉnh có bậc chẵn. Chứng minh: Theo hệ quả trên thì số đỉnh bậc lẻ trong graph G là một số chẵn. Do trong graph G có số lẻ đỉnh, nên số các đỉnh bậc chẵn phải là số lẻ. 3. Dãy cạnh kế tiếp Cho trước một graph G với tập đỉnh V và tập cạnh E. Hai cạnh của một graph cho trước được gọi là hai cạnh kề nhau nếu như chúng có một đỉnh chung. Một dãy m cạnh ei = ( Ai , Ai +1 ) với i = 1, 2,..., m được gọi là một dãy cạnh đối diện nối tiếp và thường được ký hiệu là: H = ( A1 , e1 , A2 , e2 ,..., ek , Ak +1 ) A H B C G D F E Trong trường hợp G là một graph đơn thì ta có thể biểu diễn một dãy cạnh kế tiếp qua các đỉnh của chúng, chẳng hạn dãy cạnh kế tiếp H của ta ở trên được ký hiệu đơn giản là: H = ( A1 , A2 ,..., Ak +1 ) Theo định nghĩa của ta thì một dãy các cạnh liên tiếp kề nhau (mỗi cạnh kề với cạnh tiếp theo) chưa hẳn đã là dãy cạnh kế tiếp. Mỗi dãy các cạnh liên tiếp kề nhau là một dãy cạnh kết tiếp chỉ khi đỉnh chung của một cạnh bất kì ( không phải là khuyên) với cạnh đúng trước nó và cạnh đúng sau nó khác nhau. Trong một dãy cạnh kế tiếp, Trường THPT Chuyên Thái Bình 190 HỘI CÁC TRƯỜNG THPT CHUYÊN KHU VỰC DUYÊN HẢI - ĐỒNG BẰNG BẮC BỘ HỘI THẢO KHOA HỌC LẦN THỨ VI một cạnh của graph có thể xuất hiện nhiều lần. Số m các cạnh được gọi là độ dài của dãy cạnh kế tiếp đã cho. Cho trước dãy cạnh kế tiếp H = ( A1 , e1 , A2 , e2 ,..., ek , Ak +1 ) , đỉnh A1 được gọi là đỉnh đầu và đỉnh Ak +1 được gọi là đỉnh cuối của H. H còn được gọi là dãy cạnh kế tiếp nối A1 và Ak +1 . Trong trường hợp A1 ≠ Ak , dãy cạnh kế tiếp H được gọi là dãy cạnh kế tiếp không khép kín. Còn khi A1 = Ak thì H được gọi là dãy cạnh kế tiếp khép kín. Một dãy cạnh kế tiếp e1 , e2 ,..., ek với ei ≠ e j cho mọi i ≠ j được gọi là một xích đơn. Một xích đơn với đỉnh đầu là A và đỉnh cuối là B được gọi là xích đơn nối A với B. Khái niệm dãy cạnh kế tiếp cũng như khái niệm xích đơn đóng một vai trò quan trọng trong việc mô tả sự liên thông (khả năng lưu thông dọc các cạnh) của một graph mà ta sẽ làm quen ở mục sau. Định lí sau đây cho ta thấy sự liên quan giữa hai khái niệm dãy cạnh kế tiếp và xích đơn trong một graph. Định lí 3. 1: Nếu có một dãy cạnh kế tiếp nối hai đỉnh A và B của graph thì cũng tồn tại một xích đơn nối A với B trong graph đã cho. 4.Chỉ số liên thông Trong mục này chúng ta làm quen với một khái niệm khá quan trọng của lí thuyết graph là khái niệm liên thông. Khi biểu diễn một graph trên mặt phẳng, chúng ta đã thấy rằng có nhiều khi hình biểu diễn của chúng là những cụm tách rời nhau không được nối với nhau. Tương ứng với mỗi hình rời nhau như vậy là một graph thành phần của graph đã cho mà ta sẽ gọi là một thành phần liên thông của graph cho trước. Để chính xác hóa khái niệm liên thông, trước hết chúng ta nói hai đỉnh của một graph cho trước là liên thông với nhau nếu có một dãy cạnh kế tiếp nối chúng với nhau trong graph đã cho. Chẳng hạn, graph được biểu diễn trong hình 25 của mục trước có đỉnh a và đỉnh g liên thông với nhau. Tất nhiên là một đỉnh cho trước luôn được coi là liên thông với chính nó (được nối với chính nó bởi một dãy cạnh kết tiếp có độ dài 0). Một graph được gọi là liên thông nếu hai đỉnh bất kì của nó liên thông với nhau. Quan hệ “liên thông” có những tính chất cơ bản sau: a) Mỗi đỉnh a của graph liên thông với chính nó. b) Nếu a liên thông với b thì b liên thông với a. c) Nếu a liên thông với b và b liên thông với c thì a liên thông với c. Trường THPT Chuyên Thái Bình 191 HỘI CÁC TRƯỜNG THPT CHUYÊN KHU VỰC DUYÊN HẢI - ĐỒNG BẰNG BẮC BỘ HỘI THẢO KHOA HỌC LẦN THỨ VI Thực chất đây là một qua hệ tương đương trong Tập hợp các đỉnh của graph. Quan hệ tương đương này chia tập đỉnh của graph thành các lớp có hai tính chất sau: 1) Các đỉnh thuộc cùng một lớp thì liên thông với nhau. 2) Các đỉnh không cùng thuộc một lớp không liên thông với nhau. Các lớp đỉnh này là đỉnh của các graph thành phần liên thông trong graph cho trước, được gọi là thành phần liên thông của graph đã cho. 5. Đường đi trong Graph Trong các mục trên ta đã làm quen với dãy cạnh kế tiếp và xích đơn. Trong thực tế ứng dụng của cuộc sống, ta thường gặp một khái niệm khác của dãy cạnh kế tiếp là những dãy cạnh kế tiếp được tuân thủ nguyên tắc tối ưu là chúng không đi qua đỉnh nào của graph quá một lần. Một dãy cạnh kế tiếp trong một graph cho trước được gọi là một đường đi nếu chúng không đi qua đỉnh nào của graph quá 1 lần. Cũng tương tự như với dãy cạnh kế tiếp, nếu a và b là hai đỉnh đầu tiên và đỉnh cuối cùng của đường W, thì ta nói rằng W nối a với b. Chúng cũng được gọi là đỉnh đầu và đỉnh cuối của con đường và được xem là phải khác nhau. Thông thường, đường đi được biểu diễn thông qua các đỉnh và các cạnh nối chúng, chẳng hạn W = ( p1 , e1 , p2 , e2 ,..., ek −1 , pk ) Với pi là các đỉnh và ei là các cạnh của W. Đặc biệt, khi graph cho trước là graph đơn, thì ta có thể biểu diễn một đường đi thông qua tập đỉnh của nó, chẳng hạn W = ( p1 , p2 ,..., pk ) Số cạnh của một đường đi được gọi là độ dài của nó. Tương tự như với xích đơn, ta có định lí sau đây về sự liên hệ giữa xích đơn và đường đi. Định lí 5.1. Nếu có một xích đơn nối hai đỉnh a và b của graph, thì cũng tồn tại một đường đi nối a với b trong graph đã cho. 6. Chu trình của GRAPH Khi định nghĩa đường đi nối hai đỉnh a và b của một graph, ta luôn giả thiết rằng các đỉnh a và b này phải khác nhau. Trong trường hợp a và b được nối với nhau bởi Trường THPT Chuyên Thái Bình 192 HỘI CÁC TRƯỜNG THPT CHUYÊN KHU VỰC DUYÊN HẢI - ĐỒNG BẰNG BẮC BỘ HỘI THẢO KHOA HỌC LẦN THỨ VI một cạnh, thì khi thêm cạnh (a, b) vào, ta thu được từ con đường đã cho mỗi đỉnh của graph được đi qua không quá một lần. Chu trình được kí hiệu bởi việc đưa ra các cạnh và cá đỉnh liên tiếp nhau trên chu trình. Chẳng hạn, nếu chu trình C đi qua các đỉnh p1 , p2 ,..., pk và các cạnh e1 , e2 ,..., ek thì ta viết C = ( p1 , e1 , p2 , e2 ,..., pk , ek , p1 ) Trong trường hợp graph là một graph đơn, thì thay vì viết rõ các cạnh và các đỉnh, chu trình được xác định duy nhất qua việc gọi tên các đỉnh của nó đi qua. Chẳng hạn, chu trình C đề cập ở trên có thể viết thành C = ( p1 , p2 ,..., pk , p1 ) Số cạnh của chu trình được gọi là độ dài của chu trình và thông thường hay được kí hiệu bởi l ( C ) . Một khuyên lập thành một chu trinfh có độ dài l. Một graph cho trước chỉ có chu trình có độ dài 2 nếu như nó có cạnh khép. Trong một graph đơn mỗi chu trình có độ dài ít nhất là 3. Một graph không đơn hiển nhiên luôn có ít nhất một chu trình (có độ dài 1 hoặc 2). Trong graph đơn không phải lúc nào ta cũng có thể tìm thấy một chu trình. Chẳng hạn các graph biểu diễn sơ đồ cấp diện, hoặc các sơ đồ cấp nước chẳng hạn. Định lí 6.1. Một graph có bậc của các đỉnh ≥ 2 luôn có một chu trình. Định lí 6.2. Một graph đơn với n ≥ 3 đỉnh và ≥ n cạnh luôn có ít nhất một chu trình. Định lí 6.3. Một graph đơn G với n ≥ 3 đỉnh và bậc nhỏ nhất trình C với độ dài l ( C ) ≥ + 1 . ≥ 2 có ít nhất một chu 7. Cầu trong GRAPH Trong đời sống, chúng ta không lạ gì khái niệm “cầu”. Khái niệm cầu có một mối quan hệ chặt chẽ với khái niệm liên thông. Một cạnh e của một graph liên thông G cho trước được gọi là cầu nếu như bỏ nó đi thì graph thu được có số thành phần liên thông nhiều hơn số thành phần liên thông của G. Một cạnh e của một graph liên thông G cho trước chỉ là cầu nếu như bỏ nó đi thì graph thu được G – e không còn liên thông nữa. Trường THPT Chuyên Thái Bình 193 HỘI CÁC TRƯỜNG THPT CHUYÊN KHU VỰC DUYÊN HẢI - ĐỒNG BẰNG BẮC BỘ HỘI THẢO KHOA HỌC LẦN THỨ VI Với kí hiệu ( G ) cho số thành phần liên thông của graph G ta có ( G ) < ( G − e ) cho mỗi cầu e trong G. Trong graph được biểu diễn trong hình dưới mỗi cạnh bất kì của nó là một cầu. Định lí sau đây cho ta hình dung được vị trí của cá đỉnh đầu và cuối của graph. Định lí 7.1. Cho e = ( x, y ) là một cầu trong graph G = ( X , E ) . Khi đó x và y thuộc các thành phần khác nhau của graph G – e. Định lí 7.2. Nếu e là một cầu cảu graph cho trước G, khi đó ta có (G − e) = (G ) + 1 Định lí 7.3. Một graph liên thông G cho trước có một cầu khi và chỉ khi chỉ số liên thông cạnh của nó K c ( G ) = 1 . Định lí 7.4. Một graph kiên thông G với cầu có chỉ số liên thông đỉnh là K c ( G ) = 1 . Định lí 7.5. Cầu của một graph G cho trước là tất cả những cạnh không nằm trên một chu trình nào của G. 8. Đỉnh khớp Chúng ta đã thấy rằng cầu đóng một vai trò quan trọng trong việc nghiên cứu những tính chất liên thông của một graph cho trước. Một cách trực quan, chúng ta có thể phát biểu như sau: Nếu một graph liên thông cho trước có cầu thì cầu này sẽ chia graph thành hai thành phần, được gọi là bờ, sao cho muốn đi từ bờ này tới bờ khác chúng ta bắt buộc phải đi qua cầu. Trong phần này, chúng ta nghiên cứu những đỉnh của graph mà vai trò của chúng trong graph cũng tương tự như vai trò của cầu. Chúng ta khảo sát graph G cho trước và một đỉnh x nằm trong graph G. Nếu như graph G − { x} có nhiều thành phần liên thông hơn số thành phần liên thông của graph G, thì ta gọi đỉnh x là đỉnh khớp của graph cho trước G. Đinh lí sau đây cho phép ta nhận biest lớp các graph liên thông với một đỉnh khớp. Định lí 8.1. Một graph liên thông G với n ≥ 3 đỉnh có một đỉnh khớp khi và chỉ khi chỉ số liên thông đỉnh là K c ( G ) = 1. Trường THPT Chuyên Thái Bình 194 HỘI CÁC TRƯỜNG THPT CHUYÊN KHU VỰC DUYÊN HẢI - ĐỒNG BẰNG BẮC BỘ HỘI THẢO KHOA HỌC LẦN THỨ VI Định lí 8.2. Giẳ sử rằng a là một đỉnh trong một graph cho trước G. Khi đó a là một đỉnh khớp của G khi và chỉ khi a có hai cạnh không cùng thuộc một chu trình của graph G. Định lí 8.3. Cho trước các cạnh e1 , e2 và e3 của một graph G. Nếu các cạnh e1 với e2 và với e3 liên thông chu trình, thì cạnh e1 cũng liên thông chu trình với e3 . 2. Một số bài toán áp dụng Bài 1: Trong một hội thảo khoa học tất cả các đại biểu tham dự biết tổng cộng 2n ngôn ngữ n ≥ 2 . Mỗi người biết đúng 2 ngôn ngữ và bất cứ hai người nào cũng biết chung nhiều nhất một ngôn ngữ. biết rằng với một số nguyên k thỏa mãn 1 ≤ k ≤ n − 1 đều có không quá k – 1 ngôn ngữ mà mỗi ngôn ngữ này có không quá k người biết. Chứng minh rằng ta có thể chọn ra một nhóm 2n đại biểu biết tổng cộng 2n ngôn ngữ và mỗi ngôn ngữ có đúng 2 đại biểu trong nhóm biết. Giải: Lập đồ thị G: đỉnh biểu diễn cho “ngôn ngữ”, cạnh nối hai đỉnh biểu diễn “người biết hai ngôn ngữ đó”. Vậy G là đồ thị 2n đỉnh. Điều kiện “hai người biết chung nhiều nhất một ngôn ngữ” nói rằng G là đồ thị đơn. Điều kiện còn lại cho biết: với mỗi k nguyên 1 ≤ k ≤ n − 1 có không quá k − 1 đỉnh, mỗi đỉnh có bậc nhỏ hơn hoặc bằng k (*). Theo đề bài, cần chứng minh: từ tất cả các cạnh của G có thể … 2n cạnh thuộc 2n đỉnh và mỗi đỉnh luôn thuộc đúng hai cạnh … 2n cạnh đó. Để chứng minh điều này ta sẽ chứng minh: Trong G tồn tại một đường đi khép kín có độ dài 2n và đi qua tất cả các đỉnh của G ( một đường đi như thế ta sẽ gọi là chu trình H. Ta chứng minh điều này bằng phản chứng. Giả sử trong G không có chung trình H. Khi đó tập các đỉnh không kề nhau của G là không rỗng và hữu hạn. Bằng cách thêm dần hai cạnh nối hai đỉnh không kề nhau ta sẽ xây dựng đồ thị 2n đỉnh G thỏa mãn 1) (*), 2) trong G không có chu trình H 3) Khi thêm cạnh nối hai đỉnh bất kì không kề nhau của G ta sẽ nhận được đồ thị có chu trình H. Xét G với v là đỉnh của G kí hiệu f ( v ) là bậc của v. Trường THPT Chuyên Thái Bình 195 HỘI CÁC TRƯỜNG THPT CHUYÊN KHU VỰC DUYÊN HẢI - ĐỒNG BẰNG BẮC BỘ HỘI THẢO KHOA HỌC LẦN THỨ VI a) Từ 2) và 3) suy ra giữa hai đỉnh bất kì không kề nhau của G đều tồn tại một đường đi nhận hai đỉnh ấy làm hai đầu mút, đi qua tất cả các đỉnh của G và có độ dài 2n − 1 b) Nếu hai đỉnh v và v’ của G có f ( v ) ≥ n, f ( v ') ≥ n thì v và v’ phải kề nhau. Thật vậy, giả sử v và v’ không kề nhau thì có đường đi v1 , v2 ,..., v2 n (v1 ≡ v, v2 n ≡ v ' đi qua tất cả các đỉnh của G và có độ dài 2n − 1 . Giả sử f ( v ) = s ≥ n . Kí hiệu vi1 , vi2 ,..., vis (2 = i1 < i2 < ... < is < 2n) là các đỉnh kề với v1 ≡ v . Khi đó với mỗi j = 1, 2,..., s các đỉnh v( i j ) −1 không kề với v2 n ≡ v ' vì nếu ngược lại thì chu trình H trong G là v1v2 ....v(i ) −1v2 n v2 n −1...vi mâu thuẫn với 2). Từ j j đó suy ra f ( v ') ≤ 2n − ( s − 1) ≤ n − 1 (do s ≥ n ), mâu thuẫn với f ( v ') ≥ n . Vậy v, v’ phải kều nhau. c) Từ b) suy ra tập v gồm các đỉnh v của G mà f ( v ) ≤ n − 1 là không rỗng, vậy có max v∈V f ( v ) = m ≤ n − 1 . Lấy v1 mà f ( v1 ) = m . Điều kiện (*) với k = n − 1 nói rằng có ít nhất 2n − ( n − 1) + 1 = n + 2 đỉnh có bậc ≥ n , do với k = n − 1 nói rằng có ít nhất một trong các đỉnh này, chẳng hạn v2n , không kề với v1 . Suy ra có đường đi v1 , v2 ,..., v2 n đi qua tất cả các đỉnh của G và có độ dài 2n − 1 . Kí hiệu vi , vi ,..., vi (2 = i1 < i2 < ... < im < 2n) là các đỉnh không kề với v1 thì lập luận như ở 1 2 m b) chứng tỏ với mọi j = 1 → n ta có v(i )−1 không kề với v2n (chú ý rằng điều kiện j (*) với k=1) chứng tỏ mọi đỉnh của G có bậc ≥ 2 . Áp dụng điều kiện (*) với { k=m ( 2 ≤ m ≤ n − 1) suy ra v(i )−1 , v(i )−1 ,..., v(i 1 2 m ) −1 } phải có ít nhất một đỉnh v q có f ( vq ) ≥ m + 1 . Từ định nghĩa của m suy ra f ( vq ) ≥ n như vậy vq , v2 n có f ( vq ) ≥ n , f ( v2n ) ≥ n mà không kề nhau, mâu thuẫn với b). Mâu thuẫn này cho ta điều phải chứng minh. Bài 2: Xét n điểm A1 , A2 ,..., An (n>2) trong không gian, trong đó không có 4 điểm nào đồng phẳng. Mỗi cặp điểm Ai , Aj ( i ≠ j ) được nối với nhau bởi một đoạn thẳng. Trường THPT Chuyên Thái Bình 196 HỘI CÁC TRƯỜNG THPT CHUYÊN KHU VỰC DUYÊN HẢI - ĐỒNG BẰNG BẮC BỘ HỘI THẢO KHOA HỌC LẦN THỨ VI Tìm giá trị lớn nhất của n sao cho có thể tô tất cả cá đoạn thẳng đó bằng hai màu xanh, đỏ thỏa mãn ba điều kiện sau: 1. Mỗi đoạn thẳng được tô bằng đúng một màu 2. Với mỗi i = 1, 2,..., n số đoạn thẳng có một đầu mút là Ai mà được tô màu xanh không vượt quá 4. 3. Với mỗi đoạn thẳng Ai , Aj được tô màu đỏ đều tìm thấy ít nhất một điểm Ak (k khác i,j) mà các đoạn thẳng Ak Ai và Ak Aj đều được tô màu xanh. Giải: Xét n điểm A1 , A2 ,..., An mà có thể tô màu tất cả các đoạn Ai Aj thỏa mãn đề bài. Xét Graph G có tập đỉnh V = { A1 , A2 ,..., An } và tập cạnh là các đoạn được tô màu xanh. Dễ thấy G đơn, vô hướng, n đỉnh và thỏa mãn: a) d ( Ai ) ≤ 4, ∀i = 1, n ( d ( Ai ) ký hiệu bậc của đỉnh Ai ). b) Với bất cứ hai đỉnh Ai , Aj nào cũng đều tồn tại một xích đơn nối chúng và có độ dài nhỏ hơn hoặc bằng 2. Vấn đề đặt ra ở bài đã ra tương đương với tìm số đỉnh n lớn nhất của Graph G đơn, vô hướng, n đỉnh và thỏa mãn a) và b). Xét một đỉnh Ai bất kì theo G. Khi đó mỗi đỉnh trong số n − 1 đỉnh còn lại phải kề với Ai hoặc kề với ít nhất một đỉnh kề với Ai (theo b)). Kết hợp với a) suy ra n ≤ 1 + 4 + 3 × 4 = 17 1) Xét n = 17 : Khi đó dễ thấy, phải có d ( Aj ) = 4 và do đó G có tất cả ∀i = 1,17 4 × 17 = 34 cạnh. 2 Trường THPT Chuyên Thái Bình 197 (*) HỘI CÁC TRƯỜNG THPT CHUYÊN KHU VỰC DUYÊN HẢI - ĐỒNG BẰNG BẮC BỘ HỘI THẢO KHOA HỌC LẦN THỨ VI A canh ria Ai Hinh 1 Xét đỉnh Ai bất kì của G. Từ (*) suy ra Ai kề với đúng 4 đỉnh khác, giả sử là Ai1 , Ai2 , Ai3 , Ai4 . Quy ước gọi tất cả các đỉnh còn lại của G là các đỉnh rìa, và gọi tất cả các cạnh có cả hai đầu mút là hai đỉnh rìa là các cạnh rìa. Từ b) và (*) suy ra mỗi đỉnh ( ) Aij , j = 1, 4 (xem H.1). Từ đó dễ thấy không có hai đỉnh nào của G cùng với Ai lập thành nhóm ba đỉnh đôi một kề nhau, nên trong G không có ba đỉnh nào đôi một kề nhau (vì Ai lấy ra xét là đỉnh bất kỳ). Vậy mỗi cạnh rìa đều có hai đầu mút là hai đỉnh rìa không cùng kề với một đỉnh Aij suy ra mỗi cạnh rìa cho ta một chu trình đơn đọ dài 5 và đi qua Ai . Mà số cạnh rìa có tất cả 34 − 16 = 18 , nên từ đó suy ra số chu trình đơn độ dài 5 trong G có tất cả là 18 × 17 ∉ , vô lý. Vậy n ≠ 17 . 5 2) Xét n=16. Khi đó dễ thấy, phải có d ( Aj ) = 4 ∀i = 1,16 (1) 16 × 4 = 32 cạnh. Xét một đỉnh Ai bất kì của G. Theo (1), Ai kề với 2 đúng 4 đỉnh khác, giả sử Ai1 , Ai2 , Ai3 , Ai4 . Tiếp tục bằng phương pháp lập luận như ở 1), Và do đó G có tất cả ta sẽ được: mỗi đỉnh Aij , ( j = 1, 4 ) đều kề với đúng ba đỉnh rìa và có đúng một đỉnh rìa, tạm gọi là Ak , kề với đúng hai đỉnh Ai (xem H.2). Từ đó, do Ai là bất kỳ nên suy ra: j Trong G không có ba đỉnh nào đôi một kề nhau, suy ra mỗi cạnh rìa không liên thuộc Ak cho ta đúng một chu trình đơn độ dài 5 và đi qua Ai , còn mỗi cạnh rìa liên thuộc Ak cho ta đúng hai chu trình đơn độ dài 5 và đi qua Ai . Mà số cạnh rìa có tất cả là Trường THPT Chuyên Thái Bình 198 HỘI CÁC TRƯỜNG THPT CHUYÊN KHU VỰC DUYÊN HẢI - ĐỒNG BẰNG BẮC BỘ HỘI THẢO KHOA HỌC LẦN THỨ VI 32 − 16 = 16 và trong số này có đúng hai cạnh liên thuộc Ak (do d ( Ak ) = 4 ), nên suy ra có tất cả 14 × 1 + 2 × 2 = 18 chu trình đơn độ dài 5 đi qua Ai . Vì Ai bất kỳ nên suy ra số chu trình đơn độ dài 5 trong G có tất cả là 18 × 16 ∈ , vô lý. Vậy n ≠ 16 . 5 Ai Ak Hình 2 3) Xét n=15. Ta có G được mô tả ở (H.3) thỏa mãn mọi yêu cầu đặt ra. B A C O N D M E L F K G J H I Hình 3 Vậy nmax = 15 . Trường THPT Chuyên Thái Bình 199 HỘI CÁC TRƯỜNG THPT CHUYÊN KHU VỰC DUYÊN HẢI - ĐỒNG BẰNG BẮC BỘ HỘI THẢO KHOA HỌC LẦN THỨ VI Bình luận: 1.Việc xây dựng G có 15 đỉnh xuất phát từ Graph quen thuộc(Graph Peterson) (H.4)và bởi vậy G còn có thể mô tả như sau(H.5): 2. Có thể xét trường hợp n = 16 bằng cách khác dễ lập luân hơn. Tuy nhiên việc xeta như đã trình bày ở trên đảm bảo cho lời giải nhất quán về phương pháp. Hình 4 B A G F K J L O M H E C N I D Trường THPT Chuyên Thái Bình 200 HỘI CÁC TRƯỜNG THPT CHUYÊN KHU VỰC DUYÊN HẢI - ĐỒNG BẰNG BẮC BỘ HỘI THẢO KHOA HỌC LẦN THỨ VI Hình 5 Bài 3: Trong không gian cho n điểm ( n ≥ 2 ) mà không có bốn điểm nào đồng phảng và cho 1 2 ( n − 3n + 4 ) đoạn thẳng mà tất cả các đầu mút của chúng nằm trong số n 2 điểm đã cho. Biết rằng có ít nhất một đoạn thẳng mà sau khi bỏ nó đi (giữ nguyên các đầu mút) thì sẽ tồn tại hai điểm phân biệt mà không phải là hai đầu mút của một đường gấp khúc nào. Hãy tìm số k lớn nhất sao cho có k đoạn thẳng tạo thành đường gấp khúc khép kín mà mỗi đỉnh của nó là mút của đúng hai đoạn thẳng thuộc đường gấp khúc đó. Giải: Xét graph G có tập đỉnh là tập gồm n điểm đã cho và tập cạnh là tập gồm 1 2 ( n − 3n + 4 ) đoạn thẳng đã cho. Từ giả thiết của bài toán ta thấy trong G tồn tại một 2 cạnh mà sau khi bỏ nó đi thì được G’ không liên thông. Giả sử a và b là hai đỉnh không liên thông với nhau trong G’/ Gọi Va và Vb lần lượt là tập gồm tất cả các đỉnh của G’ mà liên thông với a và b. Giả sử Va = n1 và Vb = n2 . 1 2 ( n − 3n + 4 ) cạnh; n1 ≥ 1, n2 ≥ 1, n1 + n2 ≤ n và 2 1 2 1 1 1 n − 3n + 4 ) ≤ n1 ( n1 − 1) + n2 ( n2 − 1) + ( n − n1 − n2 )( n − n1 − n2 − 1) hay ( 2 2 2 2 ( n1 − 1)(1 − n2 ) + ( n − n1 − n2 )(1 − n1 − n2 ) ≥ 0 . Do đó Dễ thấy, G’ có ( n1 − 1)(1 − n2 ) = 0  ( n − n1 − n2 )(1 − n1 − n2 ) = 0 Vậy n1 = n − 1, n2 = 1 hoặc n2 = n − 1 và n1 = 1 . Từ đó suy ra G’ có một đỉnh cô lập và ( n − 1) đỉnh mà bậc của mỗi đỉnh bằng n − 2 . Do đó G có một đỉnh bậc 1, ( n − 2 ) đỉnh mà bậc của mỗi đỉnh bằng n − 2 và một đỉnh có bậc bằng n − 1 . Bởi thế chu trình đơn có độ dài lớn nhất trong G là chu trình đơn độ dài n − 1 nếu n ≥ 4 , 0 nếu n = 2 hoặc n = 3 Trường THPT Chuyên Thái Bình 201 HỘI CÁC TRƯỜNG THPT CHUYÊN KHU VỰC DUYÊN HẢI - ĐỒNG BẰNG BẮC BỘ HỘI THẢO KHOA HỌC LẦN THỨ VI Vậy n − 1 kmax =  0 (n ≥ 4) ( n = 2, n = 3) Bài 4: Trong mặt phẳng cho 3n điểm (n>1) mà không có ba điểm nào thẳng hàng và khoảng cách giữa hai điểm bất kỳ không vượt quá 1. Chứng minh rằng có thể dựng được n tam giác đôi một rời nhau và thỏa mãn đồng thời các điều kiện sau 1. Mỗi điểm trong 3n điểm đã cho là đỉnh của đúng một tam giác 2. Tổng diện tích của n tam giác nhỏ hơn 1 . 2 Hai tam giác được gọi là rời nhau nếu chúng không có điểm nào chung nằm bên trong cũng như bên trên cạnh tam giác. Giải: Gọi 3n điểm đã cho là A1 , A2 ,... A3n . Hiển nhiên trong mặt phẳng chứa 3n điểm đó, ta có thể dựng được đường thẳng ∆ sao cho Ai ∉ ∆, i = 1,3n, A1 , A2 ,... A3n nằm về cùng một phía của ∆ ; và ∆ không song song với Ai Aj ( ∀i ≠ j ∈ {1, 2,...,3n} ) . Kys hieeju d A là khoảng cách từ điểm Ai đến ∆ . Khi đó i d Ai ≠ d Aj ( ∀i ≠ j ∈ {1, 2,...,3n} ) . Không mất tính tổng quát , giả sử: d A1 < d A 2 < ... < d A3 n (1) Qua mỗi điểm A3i +1 , i = 0,..., n − 1 , kẻ đường thẳng ∆i ∆ dễ dàng suy ra n tam giác A3 j +1 A3 j + 2 A3 j +3 , i = 0,..., n − 1 đôi một rời nhau và mỗi điểm Ai , i = 1,3n là đỉnh có đúng một tam giác trong số n tam giác đó. 1 2 Bây giờ ta sẽ chứng minh tổng S diện tích của n tam giác nói trên thỏa mãn S < . Thật vậy, xét ∆A3 j +1 A3 j + 2 A3 j +3 , i ∈ {0,1,..., n − 1} và gọi Si là diện tích nó. Dễ thấy có thể dựng được hai đường thẳng a,b cùng vuông góc với ∆ và sao cho Trường THPT Chuyên Thái Bình 202 HỘI CÁC TRƯỜNG THPT CHUYÊN KHU VỰC DUYÊN HẢI - ĐỒNG BẰNG BẮC BỘ HỘI THẢO KHOA HỌC LẦN THỨ VI 1) A đi qua đúng một trong ba điểm A3 j +1 A3 j + 2 A3 j +3 còn b đi qua ít nhất một trong hai điểm còn lại. 2) Cả ba điểm A3 j +1 A3 j + 2 A3 j +3 cùng nằm trong dải phảng ( kể cả hai biên) bị giới hạn bởi a và . Thế thì nếu gọi { A} = a ∩ ∆i , {B} = a ∩ ∆i +1 , {C} = b ∩ ∆i + 2 , {D} = b ∩ ∆i ta sẽ có hình 1 2 chữ nhật ABCD chứa toàn bộ A3 j +1 A3 j + 2 A3 j +3 . Từ đó Si < S ABCD = 1 1 AD.CD < d i 2 2 với di là khoảng cách giữa hai đường thẳng ∆ i và ∆i+1 . Từ đó suy ra n −1 s = ∑ Si < i =0 1 n −1 1 1 di < A1 A3n ≤ ∑ 2 i =0 2 2 (vì A1 A3n ≤ 1 ). Bài toán được chứng minh. Bài 5: Người ta muốn mời một số em học sinh tới dự một buổi gặp mặt, mà trong số đó mỗi em chưa quen với ít nhất là 56 em khác, và với mỗi cặp hai em chưa quen nhau thì đều có ít nhất một em quen với cả hai em đó. Hỏi số học sinh được mời dự buổi gặp mặt nói trên có thể là 65 em được hay không? Giải: Giả sử số học sinh được mời là 65 em. Ta đặt tương ứng mỗi em với một điểm trên mặt phẳng và hai em được đặt tương ứng với hai điểm khác nhau. Với mỗi cặp, hai em chưa quen nhau ta nối hai điểm tương ứng với hai em đó bởi một đoạn thẳng. Khi đó ta được một Graph đơn, vô hướng, có 65 đỉnh, bậc mỗi đỉnh không nhỏ hơn 56 và với hai điẻnh kề nhau bất kỳ luôn tồn tại ít nhất một điểm không kề với cả hai đỉnh ấy, có 65 đỉnh và thỏa mãn Trường THPT Chuyên Thái Bình 203 HỘI CÁC TRƯỜNG THPT CHUYÊN KHU VỰC DUYÊN HẢI - ĐỒNG BẰNG BẮC BỘ HỘI THẢO KHOA HỌC LẦN THỨ VI A A1 A2 A11 A17 A3 A81 A27 A21 A87 1) Bậc của mỗi đỉnh không lớn hơn 8 2) Với hai đỉnh không kề nhau, tồn tại ít nhất một đỉnh kề với cả hai đỉnh ấy. Xét G : xét đỉnh A bất kỳ của G và gọi A1 , A2 ,..., Ak ( k ≤ 8 ) là tất cả các đỉnh kề với A. Nếu k ≤ 7 thì sẽ có tối đa 7 2 = 49 đỉnh mà mỗi đỉnh kề với ít nhất một trong các đỉnh A1 , A2 ,..., Ak và không kề với A. Suy ra số đỉnh của G không vượt quá 49 + 7 + 9 = 57 < 65 trái với giả thiết. Vậy phải có k = 8 suy ra mỗi đỉnh cảu G có bậc bằng 8. Từ đây, kết hợp với 2) ta được i) Ai , A j không kề nhau ∀i ≠ j ∈ {1, 2,...,8} ii) Mỗi đỉnh Ai , i = 1,8 , ngoài A ra sẽ kề với đúng bảy đỉnh khác và nếu kí ( ) hiệu Ai , ( t = 1, 7 ) là bảy đỉnh ấy thì t { A ,..., A } ∩ { A ,..., A } = ∅ i1 i7 j1 ( ∀i ≠ j ∈ {1, 2,...,8}) j7 Từ đó suy ra trong G không có chu trình đơn độ dài 3 cũng như chu trình đơn ( ) độ dài 4. Do vậy Ai và Ai không kề nhau ∀i = 1,8 , ∀t ≠ s ∈ {1, 2,..., 7} , và do đó t s nếu Ai kề Aj (i ≠ j ) thì Ai không kề Ai m ≠ s . Từ đó suy ra có tất cả 14 ( t 3 8 s t m ) = 14.7.8 chu trình đơn độ dài 6 đi qua A. Vì A là đỉnh bất kỳ của G nên số chu trình đơn độ dài 6 trong G ;à 14.7.8.65 49.8.65 = ∉ . Điều vô lý. Vậy không tồn tại G và do đó không tồn tại 6 3 G thỏa mãn đề bài. Trường THPT Chuyên Thái Bình 204 HỘI CÁC TRƯỜNG THPT CHUYÊN KHU VỰC DUYÊN HẢI - ĐỒNG BẰNG BẮC BỘ HỘI THẢO KHOA HỌC LẦN THỨ VI Bài 6: Ở một nước có 25 thành phố. Hãy xác định số k bé nhất sao cho có thể thiết lập các đường bay (dùng cho cả đi lẫn về) giữa các thành phố để hai điều kiện sau được đồng thời thỏa mãn 1. Từ mỗi thành phố có đường bay trực tiếp đến đúng k thành phố khác 2. Nếu giữa hai thành phố không có đường bay trực tiếp thì tồn tại ít nhất một thành phố có đường bay trực tiếp đến hai thành phố đó. Giải: 1) Giả sử k là số sao cho có thể thiết lập được hệ thống đường bay thỏa mãn các điều kiện của đề bài. Khi đó, tổng số đường bay trực tiếp giữa hai thành phố sẽ là 25 × k . 2 Suy ra k ≡ 0 ( mod 2 ) Xét một thành phố A bất kỳ. Theo giả thiết, từ A có đường bay trực tiếp đến k thành phố khác, gọi là A1 , A 2 ,..., Ak . Mỗi thành phố Ai , i = 1, k , lại có đường bay trực tiếp đến k – 1 thành phố khác, (không kể A). Hơn nữa, ta lại có: Nếu từ B đến A không có đường bay trực tiếp thì B phải có đường bay trực tiếp đến ít nhất một thành phố Ai . Từ những lập luận trên suy ra, số thành phố chỉ có thể tối đa là 1 + k + k ( k –1) = k 2 + 1 . Như vậy 25 ≤ k 2 + 1 . Kết hợp với k ≡ 0 ( mod 2 ) , suy ra k ≥ 6 . 2) Với k = 6 ta sẽ chỉ ra cách thiếp lập hệ thống đường bay thỏa mãn các điều kiện cỉa đề bài. Chia 25 thành phố thành năm nhóm, mỗi nhóm gồm năm thành phố. Các thành phố của nhóm thứ i, i = 1,5 , ta kí hiệu bởi A1(i ) , A2(i ) , A3(i ) , A4(i ) , A5(i ) . Với các thành phố trong cùng nhóm i , ta thiết lập các đường bay Trường THPT Chuyên Thái Bình 205 HỘI CÁC TRƯỜNG THPT CHUYÊN KHU VỰC DUYÊN HẢI - ĐỒNG BẰNG BẮC BỘ HỘI THẢO KHOA HỌC LẦN THỨ VI i i i i i i i i i i A1( ) A2( ) , A2( ) A3( ) , A3( ) A4( ) , A4( ) A5( ) , A5( ) A1( ) . Giữa các thành phố thuộc hai nhóm i, j bất kỳ, i ≠ j ∈ {1, 2,3, 4,5} , xây dựng các đường bay sau A1(i ) A1( j ) , A2(i ) A4( j ) , A3(i ) A2( j ) , A4(i ) A5( j ) , A5(i ) A3( j ) . Bằng cách xây dựng các đường bay như trên, ta có: Từ thành phố A bất kỳ sẽ có đường bay trực tiếp đến đúng 2 thành phố, trong cùng nhóm với A và có đường bay trực tiếp đến đúng 4 thành phố khác nhóm với A. Do vậy từ mỗi thành phố sẽ có đường bay trực tiếp đến đúng 6 thành phố khác. Hơn nữa, với A, B là hai thành phố bất kỳ mà giữa chúng không có đường bay trực tiếp ta thấy: - Nếu A, B cùng thuộc nhóm thì dễ thấy luôn tồn tại 1 thành phố trong nhóm đó mà từ C có đường bay trực tiếp đến cả A và B. - Nếu A, B không cùng nhóm thì qua hình vẽ trên dễ dàng kiểm tra được sự tồn tại của thành phố C mà từ C có đường bay trực tiếp đến cả A và B. 3) Vậy kmin = 6 . Bài 7: Cho các số nguyên dương n,k,p với k ≥ 2 và k ( p + 1) ≤ n . Cho n điểm phân biệt cùng nằm trên một đường tròn. Tô tất cả n điểm đó bởi hai màu xanh, đỏ (mỗi điểm tô bởi một màu) sao cho có đúng k điểm được tô bởi màu xanh và trên mỗi cung tròn mà hai đầu mút là hai điểm màu xanh liên tiếp (tính theo chiều quay của kim đồng hồ) đều có ít nhất p điểm được tô bởi màu đỏ. Hỏi có tất cả bao nhiêu cách tô màu khác nhau? (Hai cách tô màu được gọi là khác nhau nếu có ít nhất một điểm được tô bởi hai màu khác nhau trong hai cách đó). Giải: Trước hết, ta chứng minh khẳng định sau: Khẳng định K. Cho n điểm phân biệt cùng nằm trên một đường thẳng. Tô n điểm đó bởi hai màu xanh liên tiếp ( tính từ trái qua phải) có ít nhất p điểm được tô bởi màu đỏ (tính từ trái qua phải) có ít nhất p điểm được tô bởi màu đỏ và ở bên phải Trường THPT Chuyên Thái Bình 206 HỘI CÁC TRƯỜNG THPT CHUYÊN KHU VỰC DUYÊN HẢI - ĐỒNG BẰNG BẮC BỘ HỘI THẢO KHOA HỌC LẦN THỨ VI điểm màu xanh cuối cùng có ít nhất p điểm được tô bởi màu đó. Khi đó số cách tô màu khác nhau là ( kn − kp ) Chứng minh. Lần lượt từ trái qua phải, gọi các điểm là 1, 2,.., n . Đặt tương ứng mỗi cách tô màu với bộ k số nguyên dương ( i1 < i2 < ... < ik ) , trong đó i1 , i2 ,..., ik là các điểm được tô màu xanh. Dễ thấy, tương ứng nói trên xác lập một song anh từ tập gồm tất cả các cách tô màu tới tập T = {( i1 < i2 < ... < ik ) | i j ∈ {1, 2,..., n − p}} ∀i = 1, k i j +1 − i j > p∀i = 1, k − 1 Xét ánh xạ f :T → T ' = {( j < j 1 2 < ... < jk ) | {1, 2,..., n − kp} ∀t = 1, k ( i1 < i2 < ... < ik ) ∈ T → ( i1 , i2 ,..., ik − ( k − 1) p ∈ T ') } Dễ chứng minh được f là song ánh từ T đến T’. Từ đó, ta có điều phải chứng minh. 2) Trở lại bài toán. Lần lượt, theo chiều kim đồng hồ, gọi các điểm là A1 , A2 ,..., An . Gọi X là tập gồm tất cả các cách tô màu khác nhau. Xét phân hoạch X = X '∪ X '' Trong đó X={ x ∈ X | trong x có điểm màu xanh thuộc { Ai ,..., Ap } }, X’’=X\X’. Hiển nhiên, với x ∈ X '' thì trong x không có điểm màu xanh nào thuộc tập { A1 , A2 ,..., Ap } . Do đó, theo khẳng định K ta có cardX’’= ( kn − kp ) Xét X’. Kí hiệu X i ' = {x ∈ X ' | trong x có điểm Ai được tô màu xanh, i = 1, p . Thế thì X i '∩ X j ' = ∅ ∀i ≠ j ∈ {1, 2,..., p} và X = i =p1 X i ' . Với mỗi i = 1, p , theo khẳng định K, ta có cardX i ' = cardX ' = p ( kn −−1kp −1 ) . Vậy cardX = ( kn − kp ) + p ( nk −−1kp −1 ) . Bài 8: Trường THPT Chuyên Thái Bình 207 ( k −1 n −1− p − ( k −1) p )=( k −1 n − kp −1 ) . Do đó HỘI CÁC TRƯỜNG THPT CHUYÊN KHU VỰC DUYÊN HẢI - ĐỒNG BẰNG BẮC BỘ HỘI THẢO KHOA HỌC LẦN THỨ VI Trong một cuộc hội thảo có n, n ≥ 10 người tham dự. Biết rằng n + 2 người tham dự. 1. Mỗi người quen với ít nhất   3  2. Hai người bất kỳ A và B nếu không quen nhau thì quen nhau gián tiếp nghĩa là có k ( k ≥ 1) người A1 , A2 ,..., Ak sao cho A quen A1 , Ai quen Ai +1 , ( i = 1, 2,..., k − 1) và Ak quen B. 3. Không thể xếp n người thành một hàng ngang sao cho hai người cạnh nhau bất kỳ đều quen nhau. Chứng minh rằng có thể chia n người thành hai nhóm: nhóm thứ nhất xếp được quanh một bàn tròn sao cho hai người cạnh nhau bất kỳ đều quen nhau, còn nhóm thứ hai gồm người đôi một không quen nhau. Giải: Chuyển bài toán sang ngôn ngữ Graph, trong đó mỗi người coi là một điểm trên mặt phẳng, còn quan hệ quen nhau coi là một cạnh (1 đoạn thẳng với giả thiết rằng các đoạn thẳng này không cắt nhau trừ hai điểm đầu mút), ta có graph G đơn, vô hướng n + 2  3  với tập đỉnh gồm n điểm p = { A1 , A2 ,..., An } và bậc của đỉnh A bất kỳ là d ( A ) ≥  Điều kiện “Hai người bất kỳ quen nhau hoặc quen nhau gián tiếp chứng tỏ Graph G là liên thông. Trong G (hữu hạn) xét đường gấp khúc nhiều cạnh nhất Po, giả sử Po có k đỉnh là P0 = { A1 , A2 ,..., Ak } với Ai Ai +1 (i = 1, 2,..., k − 1) là các cạnh ( Ai kề với Ai +1 ) Do điều kiện (3) thì k ≤ n − 1 Gọi N(A) là tập các đỉnh kề với đỉnh A. Ta có N ( A1 ) ⊂ { A2 ,..., Ak } và N ( Ak ) ⊂ { A2 ,..., Ak −1} Vì trái lại thì tồn tại đường gấp khúc khác có nhiều cạnh hơn Po. Giả sử N ( Ai ) = { Ai , Ai ,..., Ai } , i ∈ {1, 2,..., n} ký hiệu 1 2 s Trường THPT Chuyên Thái Bình 208 HỘI CÁC TRƯỜNG THPT CHUYÊN KHU VỰC DUYÊN HẢI - ĐỒNG BẰNG BẮC BỘ HỘI THẢO KHOA HỌC LẦN THỨ VI { N ( A ) = {A } } N ( Ai ) = Ai1 +1 , Ai2 +1 ,..., Ais +1 + − i1 −1 i , Ai2 −1 ,..., Ais −1 B A j+1 Aj-1 A1 Ak Do k ≤ n − 1 nên tồn tại đỉnh B ∉ P0 . Ta có N ( B ) ∩ N ( Ak ) = ∅ + Thật vậy nếu ∃A j ∈ N ( B ) ∩ N ( Ak ) thì tồn tại đường gấp khúc + ( A ,..., A 1 j −1 , Ak , Ak −1 ,..., Aj +1 , Aj , B ) có k+1 cạnh, trái giả thiết đối với P0 . Lập luận tương tự có N ( B ) ∩ N ( A1 ) = ∅ . Ta cũng có N ( A1 ) ∩ N ( Ak ) = ∅ vì nếu trái lại thì − − + − + − + n + 2 n+2  N ( B ) ∪ N ( A1 ) ∪ N ( Ak ) = N ( B ) + B ( A1 ) + N ( Ak ) ≥ 3  > 3 − 1 = n − 1   3   3  Suy ra số đỉnh của tập hợp này lớn hơn hoặc bằng n mà tập hợp đó không chứa đỉnh B. Mâu thuẫn. Vậy ∃Ai ∈ N ( A1 ) ∩ N ( Ak ) − + Khi đó tồn tại đường gấp khúc khép kín có k – 1 đỉnh thuộc tập Pc \ { Ai } là ( A1 , A2 ,..., Ai −1 , Ak , Ak −1 ,..., Ai +1 ) Trường THPT Chuyên Thái Bình 209 HỘI CÁC TRƯỜNG THPT CHUYÊN KHU VỰC DUYÊN HẢI - ĐỒNG BẰNG BẮC BỘ HỘI THẢO KHOA HỌC LẦN THỨ VI Ai Ai-1 Ai+1 Ak+1 A2 A1 Ak Tập còn lại chứa các đỉnh đôi một không kề nhau (không có đoạn thẳng nối chung) vì nếu trái lại, chẳng hạn có B1 , B2 ≠ P0 \ { Ai } mà B1 kề với B2 do tính liên thông tồn tại đường gấp khúc chứa B1 , B2 và P0 \ { Ai } có nhiều cạnh hơn P0 mâu thuẫn. Trường THPT Chuyên Thái Bình 210 HỘI CÁC TRƯỜNG THPT CHUYÊN KHU VỰC DUYÊN HẢI - ĐỒNG BẰNG BẮC BỘ HỘI THẢO KHOA HỌC LẦN THỨ VI SỬ DỤNG PHƯƠNG PHÁP THIẾT LẬP HỆ THỨC TRUY HỒI ĐỂ GIẢI BÀI TOÁN ĐẾM TỔ HỢP Tổ Toán –Tin học - THPT Chuyên Lào Cai Chuyên để tổ hợp và toán rời rạc là một trong những nội dung khó trong chương trình thi chọn học sinh giỏi các cấp, đã có rất nhiều các tài liệu viết về nội dung này với nhiều cách tiếp cận khác nhau. Bài toán đếm tổ hợp là một dạng bài toán thường xuyên xuất hiện trong các kỳ thi, cùng với các bài toán tồn tại tổ hợp, bài toán tối ưu tổ hợp,… Có nhiều cách giải quyết bài toán đếm như phương pháp song ánh, phương pháp hàm sinh,…; bài viết này xin trình bày phương pháp thiết lập hệ thức truy hồi để giải một số bài toán đếm, ý tưởng chung của phương pháp này là thiết lập hệ thức truy hồi giữa phép đếm cần tính Sn với Sn −1 , Sn − 2 ,... từ đó suy ra Sn . Trước hết ta xét ví dụ mở đầu sau. Bài 1. ( HSG-VT-2009-2010) Trường THPT Chuyên Thái Bình 211 HỘI CÁC TRƯỜNG THPT CHUYÊN KHU VỰC DUYÊN HẢI - ĐỒNG BẰNG BẮC BỘ HỘI THẢO KHOA HỌC LẦN THỨ VI Cho số nguyên dương n . Gọi M n là tập các số tự nhiên (viết trong hệ thập phân) có n chữ số, các chữ số lớn hơn 1 và không có hai chữ số cùng nhỏ hơn 7 đứng liền nhau. Tính số phần tử của tập M n . Lời Giải: Kí hiệu un = M n , Gọi X n , Yn lần lượt là tập các số tự nhiên theo thứ tự : Có chữ số tận cùng nhỏ hơn 7 và các số có tận cùng lớn hơn 6. Ta có: M n = X n ∪ Yn , X n ∩ Yn = ∅ . Lấy một phần tử của M n+1 , bỏ đi phần tử cuối cùng ta được một phần tử của M n . Ngược lại, xét một phần tử x của M n . - Nếu x có tận cùng nhỏ hơn 7 thì có một cách thêm chữ số 0 vào vị trí đầu ta được một phần tử của X n +1 và có đúng 3 cách thêm vào chữ số cuối để tạo ra một phần tử của Yn +1 . - Nếu x có tận cùng lớn hơn 6 thì thì có 5 cách thêm vào chữ số cuối để để tạo ra một phần tử của X n +1 và có 3 cách thêm vào chữ số cuối để tạo ra một phần tử của Yn +1 .  X n +1 = X n + 5 Yn Vậy :   Yn +1 = 3 X n + 3 Yn ⇒ M n +1 = 4 ( X n + Yn ) + 4 Yn = 4 ( X n + Yn ) + 12 ( X n −1 + Yn −1 ) ⇒ un +1 = 4un + 12un −1 , n ≥ 2 Từ đó tìm được un = ( ) ( ) 1 1 n −1 n −1 . 15.6n −1 + ( −2 ) ⇒ M n = 15.6 n −1 + ( −2 ) 2 2 Nhận xét: • Số có n chữ số viết trong hệ thập phân có thể có chữ số đầu là những số 0 • Cần xây dựng được dãy các mối quan hệ giữa X n với X n +1 & Yn +1 ; Yn với X n +1 & Yn +1 • Cần phải có khả năng trong việc gọi X n , Yn để xây dựng hệ thức truy hồi Trường THPT Chuyên Thái Bình 212 HỘI CÁC TRƯỜNG THPT CHUYÊN KHU VỰC DUYÊN HẢI - ĐỒNG BẰNG BẮC BỘ HỘI THẢO KHOA HỌC LẦN THỨ VI • Bài toán đếm số phần tử mà có biến n chưa cụ thể ta thường sử dụng phương pháp này • ? Phải chăng chỉ có duy nhất cách gọi X n , Yn như trên. Một số bài toán vận dụng Bài 2. (Romania 2003) Cho số nguyên dương n . Có bao nhiêu số tự nhiên có n chữ số được lập từ các chữ số {2,3, 7,9} và chia hết cho 3. Lời Giải: Gọi M n là tập hợp gồm tất cả các số có n chữ số được lập từ các chữ số {2,3, 7,9} Gọi An , Bn , Cn lần lượt là tập hợp gồm tất cả các số có n chữ số mà chia cho 3 được dư là 0,1,2. Khi đó ta có M n = An ∪ Bn ∪ Cn , An ∩ Bn = ∅, Bn ∩ Cn = ∅, An ∩ Cn = ∅ ⇒ M n = An + Bn + Cn Lấy một phần tử thuộc vào M n +1 , bỏ đi phần tử cuối cùng ta được phần tử thuộc Mn Với x ∈ M n ta có • Nếu x ≡ 0 ( mod 3) hay x ∈ An thì 2 cách thêm vào chữ số cuối để được phần tử thuộc An +1 , có 1 cách thêm vào chữ số cuối để được phần tử thuộc Bn +1 , có 1 cách thêm vào chữ số cuối để được phần tử thuộc Cn +1 . Trường THPT Chuyên Thái Bình 213 HỘI CÁC TRƯỜNG THPT CHUYÊN KHU VỰC DUYÊN HẢI - ĐỒNG BẰNG BẮC BỘ HỘI THẢO KHOA HỌC LẦN THỨ VI • Nếu x ≡ 1( mod 3) hay x ∈ An thì 1 cách thêm vào chữ số cuối để được phần tử thuộc An +1 , có 2 cách thêm vào chữ số cuối để được phần tử thuộc Bn +1 , có 1 cách thêm vào chữ số cuối để được phần tử thuộc Cn+1 . • Nếu x ≡ 2 ( mod 3) hay x ∈ An thì 1 cách thêm vào chữ số cuối để được phần tử thuộc An +1 , có 1 cách thêm vào chữ số cuối để được phần tử thuộc Bn +1 , có 2 cách thêm vào chữ số cuối để được phần tử thuộc Cn +1 .  An +1 = 2 An + Bn + Cn  Vậy ta có hệ  Bn +1 = An + 2 Bn + Cn   Cn +1 = An + Bn + 2 Cn ⇒ M n +1 = 4 M n = ... = 4n. M 1 Ta có M1 = 4 ⇒ M n +1 = 4n +1 ⇒ M n = 4n ⇒ An +1 = 4n + An = 4n + 4n −1 + ... + 41 + A1 Có A1 = 2 ⇒ An = 2 + ( 4 + 42 + ... + 4n −1 ) = 2 + 4. 4n −1 − 1 4n + 2 = 3 3 Nhận xét: • Lời giải trên cũng giúp ta tìm được các số…chia 3 dư 1; dư 2. Bài 3. ( THTT 5/2010) Từ các số 1,2,3,4,5 có thể lập được bao nhiêu số tự nhiên có n chữ số sao cho trong mỗi số đó đều chứa một số lẻ các chữ số 1 và một số chẵn các chữ số 2 ( n là một số nguyên dương cho trước)? Lời Giải: Với mỗi số nguyên dương n , kí hiệu M n là tập tất cả các số tự nhiên có n chữ số được lập từ các số 1,2,3,4,5,và An ; Bn ; Cn ; Dn là tập các số tự nhiên có n chữ số được lập từ các số 1,2,3,4,5 theo tứ tự chứa một số lẻ các chữ số 1 và chẵn các chữ số 2, chứa một số lẻ các chữ số 1 và lẻ các chữ số 2, chứa một số chẵn các Trường THPT Chuyên Thái Bình 214 HỘI CÁC TRƯỜNG THPT CHUYÊN KHU VỰC DUYÊN HẢI - ĐỒNG BẰNG BẮC BỘ HỘI THẢO KHOA HỌC LẦN THỨ VI chữ số 1 và chẵn các chữ số 2, chứa một số chẵn các chữ số 1 và lẻ các chữ số 2. Dễ thấy An , Bn , Cn , Dn An = Bn = Cn = Dn = đôi một rời nhau và M n = An ∪ Bn ∪ Cn ∪ Dn , n 1 5 Mn = . 4 4 Lấy một phần tử của M n +1 , bỏ đi phần tử cuối ta được một phần tử của M n , ngược lại lấy một phần tử x của M n -Nếu x ∈ An thì có 3 cách thêm vào chữ số cuối để tạo ra một phần tử của . -Nếu x ∈ Bn thì có 1 cách thêm vào chữ số cuối để tạo ra một phần tử của An+1 . - Nếu x ∈ Cn thì có 1 cách thêm vào chữ số cuối để tạo ra một phần tử của An +1 . - Nếu x ∈ Dn thì không có cách thêm nào vào chữ số cuối để tạo ra một phần tử của An +1 . Vậy An +1 = 3 An + Bn + Cn = An + ( An + Bn + Cn + Dn ) = An + 5n Từ A1 = 1, An +1 = An + 5n ⇒ An +1 = A1 + 5 + 52 + ... + 5n = 5n +1 − 1 5n − 1 ⇒ An = 4 4 Bài 4. Từ các số 3,4,5,6 có thể lập được bao nhiếu số tự nhiên có n chữ số mà mỗi chữ số đó đều chia hết cho 3( n là số nguyên dương cho trước)? Lời Giải: Gọi M n là tập tất cả các số tự nhiên có n chữ số được tạo thành từ các số 3,4,5,6 và An , Bn , Cn lần lượt là các tập con của M n mà chia cho 3 có số dư là 0,1,2. Ta có M n = An ∪ Bn ∪ Cn , và An ∩ Bn = Bn ∩ Cn = Cn ∩ An = ∅ . Lấy một phần tử của M n +1 , bỏ đi phần tử cuối ta được một phần tử của M n , ngược lại lấy một phần tử x của M n . Trường THPT Chuyên Thái Bình 215 HỘI CÁC TRƯỜNG THPT CHUYÊN KHU VỰC DUYÊN HẢI - ĐỒNG BẰNG BẮC BỘ HỘI THẢO KHOA HỌC LẦN THỨ VI - Nếu x ∈ An thì có 2 cách thêm vào chữ số cuối để được một phần tử của An +1 và một cách thêm vào chữ số cuối để được một phần tử của Bn +1 , Cn +1 . - Nếu x ∈ Bn thì có 2 cách thêm vào chữ số cuối để được một phần tử của Bn +1 và có và một cách thêm vào chữ số cuối để được một phần tử của An +1 , Cn +1 . - Nếu x ∈ Cn thì có 2 cách thêm vào chữ số cuối để được một phần tử của Cn +1 và có và một cách thêm vào chữ số cuối để được một phần tử của An +1 , Bn +1 .  An +1  - Vậy ta có hệ  Bn +1   Cn +1 Từ đó ta có M n +1 = An +1 + = 2 An + Bn + Cn = An + 2 Bn + Cn = An + Bn + 2 Cn Bn +1 + Cn +1 = 4 ( An + Bn + Cn ) = 4 M n Mà M1 = 2 ⇒ M n+1 = 4n+1 ⇒ M n = 4n ⇒ An+1 = An + 4n Từ đó tính được An = 4n + 2 . 3 Bài 5. Có người ngồi thành một hàng ngang vào chiếc ghế. Hỏi có bao nhiêu cách lập hàng mới cho người đó mà trong mỗi cách lập hàng mới: mỗi người hoặc giữ nguyên vị trí của mình, hoặc đổi chỗ cho người liền bên trái, hoặc đổi chỗ cho người liền bên phải. Lời Giải: Đánh số thứ tự vị trí các ghế từ trái qua phải là 1,2,3,…,n Gọi Sn là số cách lập hàng mới cho n người thỏa mãn đề bài. Dễ thấy S1 = 1, S2 = 2. Với n ≥ 3 : Xét một cách lập hàng mới thỏa mãn điều kiện. Có hai loại hàng được lập: Trường THPT Chuyên Thái Bình 216 HỘI CÁC TRƯỜNG THPT CHUYÊN KHU VỰC DUYÊN HẢI - ĐỒNG BẰNG BẮC BỘ HỘI THẢO KHOA HỌC LẦN THỨ VI Loại 1: Người ở vị trí số 1 giữ nguyên vị trí. Rõ ràng số hàng được lập loại này là Sn-1 cách. Loại 2: Người ở vị trí số 1 đổi chỗ, khi đó người ở vị trí số 1 chỉ có thể xếp vào vị trí số 2 và người ở vị trí 2 phải chuyển sang vị trí 1. Số hàng loại này là Sn-2. Từ đó ta có Sn = Sn −1 + Sn − 2 , n ≥ 3 . Vậy: S1 = 1,S2 = 2,Sn + 2 = Sn +1 + Sn , n ∈ * Bài 6 (Trung quốc 1989) Có thể nào chia được 1989 điểm thành 30 nhóm có cỡ của các nhóm đó không bằng nhau để cho số các tập hợp gồm 3 điểm mà mỗi điểm được chọn từ 3 nhóm khác nhau là lớn nhất. Lời Giải: Giả sử các cỡ của 30 nhóm là. a1 < a2 < a3 < ... < a30 Ta sẽ gọi nhóm có cỡ ak vắn tắt là nhóm k. Giả sử ak ≤ ak +1 − 3 Khi đó xét việc thay ak bởi ak + 1 và ak +1 bởi ak +1 − 1 ta vẫn còn được các nhóm có cỡ không bằng nhau. Số tất cả các bộ ba mà không có phần tử nào thuộc nhóm k hoặc k+1 thì không bị ảnh hưởng, chỉ có các bộ ba có đúng một phần tử thuộc nhóm k hoặc k+1 bị. Nhưng số các bộ ba có đúng một phần tử thuộc nhóm k và một phần tử thuộc nhóm k+1 tăng lên, bởi vì ak ak +1 < ( ak + 1)( ak +1 − 1) Như thế khoảng trống lớn nhất là 2. Giả sử có hai khoảng trống độ dài là 2. Ta giả sử a j + 1 < a j +1 < ak < ak +1 − 1 Bây giờ ta có thể thay a j và ak +1 bởi a j + 1 và ak +1 − 1 . Trường THPT Chuyên Thái Bình 217 HỘI CÁC TRƯỜNG THPT CHUYÊN KHU VỰC DUYÊN HẢI - ĐỒNG BẰNG BẮC BỘ HỘI THẢO KHOA HỌC LẦN THỨ VI Lập luận giống như trước số các bộ ba cũng tăng lên. Vậy có nhiều lắm là hai khoảng trống độ dài 2. Điều này chỉ đủ cho ta xác định các cỡ. Giả sử các cỡ tạo thành một dãy đơn giản có tất cả các khoảng trống bằng 1. Nếu thành phần đầu tiên là n thì thành phần sau cùng là n+ 29. và tổng là 30 ( 2n + 29 ) 2 . Nhưng tổng này không thể bằng 1989 vì 1989 không là bội của 5. Do đó ta giả sử các cỡ tạo thành một dãy dơn giản có tất cả các khoảng trống bằng 1. ngoại trừ một thành phần bị bỏ qua, để có một khoảng trống bằng 2.Nếu thành phần đầu tiên là v và thành phần bị bỏ qua là m thì ta có: 30 ( 2n + 29 ) 2 − m = 1989 Nếu n ≤ 50 thì m ≤ 2015 − 1989 = 26 ,số này quá nhỏ vì ta phải có m giữa n và n +30 Nếu n ≥ 52 thì m ≥ 2077 − 1989 = 88 , số này quá lớn. Vậy n=51 và m =57 Suy ra các cỡ là: 51,52, 56, 58, 59,60,...81. Bài 7( Dự tuyển IMO lần thứ 38) : Trong thành phố a có n cô gái và n chàng trai và các cô gái đều quen biết các chàng trai. Trong thành phố B có n cô gái g1 ; g 2 ;...; g n và 2n-1 chàng trai b1 ; b2 ;...; b2 n −1 . Các cô gái gi chỉ quen các chành trai b1 ; b2 ;...; b2i −1 và không quen biết các chàng trai khác. Kí hiệu A(r), B(r) lần lượt là số các cách thức khác nhau để r cô gái từ thành phố A và thành phố B có thể khiêu vũ với r chàng trai từ chính thành phố của họ tạo thành r cặp, mỗi cô gái với một chàng trai mà cô ấy quen biết. Chứng minh rằng A(r) =B(r). Trường THPT Chuyên Thái Bình 218 HỘI CÁC TRƯỜNG THPT CHUYÊN KHU VỰC DUYÊN HẢI - ĐỒNG BẰNG BẮC BỘ HỘI THẢO KHOA HỌC LẦN THỨ VI Lời Giải: Ta kí hiệu A(r) và B(r) bởi A(n, r) và B(n, r) Ta có thể chọn r cô gái trong n cô gái của thành phố A bằng Cnr cách Ta có thể chọn r chàng trai trong n chàng trai của thành phố A bằng Cnr cách Vì mỗi cô gái trong A đều quen với các chàng trai nên bất kì nhóm nào gồm r cô gái được chọn ra đều có thể xếp cặp với r chàng trai nên ta có A ( n, r ) = Cnr .Cnr .r ! = Cnr . n! ( n − r )! Cho n ≥ 3 và 2 < r bn thì xn ≥ bn + 1 = an + 1 ≥ Tn + 1 Vậy ta có bn +1 ≥ xn + bn Tn + Tn + 1 1 ≥ = Tn + 2 2 2 Do Tn và b n+1 đều là số nguyên thì bn +1 ≥ Tn + 1 . Từ đó và (*) suy ra hoặc Tn +1 > Tn , nếu ở thời điểm thứ n chỉ có duy nhất một số bn = Tn hoặc Tn +1 = Tn , nếu có ít nhất một học sinh khác B có số kẹo là Tn . Vậy dãy (Tn ) là dãy tự nhiên không giảm, hơn nữa khi bn = Tn < an thì đến thời điểm thứ n + 1 có bn+1 ≥ Tn + 1 nên số Tn sẽ mất đi một lần, và cứ tiếp tục chuyển kẹo sau hữu hạn lần thì số Tn mất hết hết nghĩa là dãy (Tn ) tăng thực sự . Do dãy ( M n ) là dãy tự nhiên không tăng còn dãy (Tn ) là dãy tự nhiên không giảm và có lúc tăng thực sự nên đến một thời điểm nào đó phải có M k = Tk , lúc đó số keojcuar mọi học sinh đều như nhau. Bài 10 (VMO-2002) Cho tập S gồm tất cả số nguyên trong đoạn: [1; 2002]. Gọi T là tập hợp gồm tất cả các tập con không rỗng của S. Với mỗi tập X thuộc T kí hiệu m(X) là trung bình cộng của tất cả các số thuộc X. Đặt m = các tập hợp X thuộc T. Tìm m. Trường THPT Chuyên Thái Bình 222 ∑ m( X ) ở đây tổng lấy theo tất cả T HỘI CÁC TRƯỜNG THPT CHUYÊN KHU VỰC DUYÊN HẢI - ĐỒNG BẰNG BẮC BỘ HỘI THẢO KHOA HỌC LẦN THỨ VI Lời Giải: Với mỗi k thuộc {1, 2, ...., 2002} đặt mk = ∑ m ( X ) ở đây tổng lấy theo tất cả các tập hợp X thuộc T mà X = k k Xét số a bất kì thuộc tập S. Dễ thấy a có mặt trong C2001 tập X thuộc T mà X = k k −1 k −1 = 1001.2003.C2001 Suy ra: k .mk = (1 + 2 + 3 + ... + 2002 ) C2001 ( ) k −1 2003. 22002 − 1 C2001 2003 2002 k nên ∑ m ( X ) = ∑ mk = 1001.2003.∑ = . ∑ C2002 = k 2 k =1 2 k =1 k =1 2002 2002  ∑ m ( X )  2003  = 2 T   Vì T = 22002 − 1 nên m =  Bài 11: Xếp n học sinh ngồi quanh một bàn tròn. Ngăn hàng đề có tất cả m loại đề thi. Hỏi có bao nhiêu cách phát đề cho học sinh sao cho không có 2 học sinh nào ngồi cạnh nhau có cùng đề thi? Lời Giải: Gọi Sn là số cách phát đề cho học sinh sao cho không có 2 học sinh nào ngồi cạnh nhau có cùng đề thi Cố định một học sinh làm vị trí đầu tiên và các học sinh bên tay phải của học sinh đó là vị trí thứ 2, thứ 3,…, thứ n.( học sinh ở vị trí thứ n ngồi cạnh học sinh ở vị trí thứ nhất) Ta thấy: +) Nếu học sinh ở vị trí thứ nhất và học sinh ở vị trí thứ n-1 có đề thi khác nhau thì sẽ có m-2 cách phát đề cho học sinh ở vị trí thứ n. +) Nếu học sinh ở vị trí thứ nhất và học sinh ở vị trí thứ n-1 có đề thi giống nhau thì có m-1 cách phát đề cho học sinh ở vị trí thứ n. Trường THPT Chuyên Thái Bình 223 HỘI CÁC TRƯỜNG THPT CHUYÊN KHU VỰC DUYÊN HẢI - ĐỒNG BẰNG BẮC BỘ HỘI THẢO KHOA HỌC LẦN THỨ VI Do đó ta có hệ thức:Sn = (m-2)Sn-1 + (m-1)Sn-2 , (n ≥ 4) Sử dụng phương pháp sai phân để tính Sn. Xét phương trình đặc trưng: x2 - (m-2)x - (m-1) = 0 ⇔ x = -1, x = m-1 Sn = a(-1)n + b(m-1)n Do S2 = m(m-1), S3 = m(m-1)(m-2), suy ra: a+ b(m-1)2 = m(m-1) và -a +b(m-1)3= m(m-1)(m-2) Do đó: a = m-1 và b = 1 Vậy Sn = (m-1)(-1)n + (m-1)n (n≥2) Bài 12 ( IMO-2011): Giả sử n > 0 là một số nguyên. Cho một cái cân đĩa và quả cân có trọng lượng 20 , 21,..., 2n −1 . Ta muốn đặt lên cái cân mỗi một trong n quả cân, lần lượt từng quả một, theo cách để đảm bảo đĩa cân bên phải không bao giờ nặng hơn đĩa cân bên trái . Ở mỗi bước ta chọn một trong các quả cân chưa được đặt lên rồi đặt nó lên đĩa bên phải, hoặc đĩa bên trái, cho đến khi tất cả các quả cân đều được đặt lên đĩa. Hỏi có bao nhiêu cách để thực hiện việc đặt cân theo đúng mục đích đề ra? Lời Giải: Gọi Sn là số cách thực hiện việc đặt n quả cân lên đĩa thỏa mãn yêu cầu đề ra. Xét cách đặt n + 1 quả cân có trọng lượng 20 , 21,..., 2n . Do quả cân có trọng lượng nên trong mọi cách đặt cân thỏa mãn thì luôn được đặt ở đĩa cân bên trái. Nếu quả cân 2n được chọn để đặt cuối cùng ( chỉ có một cách đặt, vì quả 2n chỉ đặt lên đĩa bên trái ) và số cách đặt n quả cân còn lại là Sn. Nếu quả cân 2n được đặt ở bước thứ i ( i = 1,2,…,n). Do có n cách chọn , và trong trường hợp này quả cân có trọng lượng 2n −1 có 2 cách đặt ( đặt lên đĩa bên phải hay đĩa bên trái đều thỏa mãn) , do đó số cách đặt n+1 quả cân trong trường hợp này là 2nSn. Vậy ta có hệ thức truy hồi Sn +1 = 2nSn + Sn = (2n + 1)Sn Ta có S1 = 1 nên Sn = (2n − 1)(2n − 3)...3.1 Trường THPT Chuyên Thái Bình 224 HỘI CÁC TRƯỜNG THPT CHUYÊN KHU VỰC DUYÊN HẢI - ĐỒNG BẰNG BẮC BỘ HỘI THẢO KHOA HỌC LẦN THỨ VI Bài 13: Cho A = {1, 2, …, 2n}. Một tập con của A được gọi là tốt nếu nó có đúng hai phần tử x, y và |x – y| ∈ {1, n}. Tìm số các tập hợp {A1, A2, …, An} thoả mãn điều kiện Ai là tập con tốt với mọi i = 1, 2, …, n và A1 ∪ A2 ∪ … ∪ An = A. Lời giải: Từ giả thiết, ta sẽ viết lại bài toán như sau (các bạn tự kiểm tra tính tương đương của bài toán này so với bài ban đầu): “Cho 1 hình chữ nhật kích thước được chia thành các ô vuông đơn vị. Đánh số các ô từ trái qua phải là 1,2,...,n (hàng 1) và n + 1,n + 2,..,2n (hàng 2) Lát chúng bằng các quân domino 1 * 2 sao cho chúng phủ kín hình chữ nhật và không có 2 quân nào đè lên nhau. Ngoài ra, với n lẻ, ta được bổ sung thêm 1 quân domino "đặc biệt" có thể phủ kín 2 ô n và n + 1. Đếm số cách lát thỏa mãn đề bài” Với bài toán này, xét Sn là số cách lát thỏa mãn đề bài với hình chữ nhật kích thước 2 * n. Ta sẽ tìm cách xây dựng công thức truy hồi cho Sn Giả sử ta đã lát được hình chữ nhật 2 * (n + 1) bằng các quân domino. Xét quân domino phủ lên ô vuông n. Có 3 khả năng xảy ra: 1/ Quân domino đó phủ lên 2 ô: kích thước . Rõ ràng phần còn lại là 1 hình chữ nhật 2 * n, và số cách lát trong tình huống này là Sn 2/ Quân domino đó phủ lên 2 ô (n, n + 1) . Như vậy, buộc phải có 1 quân domino phủ lên 2 ô(2n-1,2n) và khi đó, phần còn lại là 1 hình chữ nhật kich thước 2 * (n - 1). Tức số cách lát trong tình huống này là Sn-1 3/ Quân domino đó phủ lên 2 ô (n, n+1) (với n lẻ). Khi đó, phần còn lại chỉ có thể lát được bằng các quân domino nằm ngang (nếu có 1 quân domino nào nằm dọc thì nó sẽ chia hình chữ nhật thành 2 phần, mỗi phần có 1 số lẻ ô chưa được lát (do quân domino "đặc biệt" gây ra)) Trường THPT Chuyên Thái Bình 225 HỘI CÁC TRƯỜNG THPT CHUYÊN KHU VỰC DUYÊN HẢI - ĐỒNG BẰNG BẮC BỘ HỘI THẢO KHOA HỌC LẦN THỨ VI Tức trong trường hợp này chỉ có 1 cách lát duy nhất Như vậy ta xây dựng được công thức truy hồi như sau: S2k = S2k −1 + S2k − 2 − 1 (lưu ý rằng khi n chẵn thì không có quân domino "đặc biệt" nên phải bớt đi 1 cách của S2k-1) (lập luận tương tự với quân domino “đặc biệt”) Và bằng quy nạp ta sẽ thu được , trong đó Fk là số Fibonacci thứ k của dãy Fibonacci được xác định bởi công thức Cuối cùng ta được công thức tổng quát : n n n 1  1 + 5   1 − 5   1 − ( −1) Sn =   −  + 2 5  2   2     Bài 14: Cho tập hợp S = {1, 2, 3, …, n}. Tìm số cách chia tập S thành 3 tập con khác rỗng sao cho mỗi tập con không chứa hai số nguyên liên tiếp. Lời Giải: Kí hiệu S(n) là số cách chia tập S thành 3 tập con không chứa khác rỗng mà bất kì tập con nào cũng không chứa 2 phần tử liên tiếp nhau Ta sẽ tìm cách tính S(n+1) theo S(n) Giả sử ta đã chia được 3 tập con và tổng số phần tử của chúng là n. Bổ sung thêm phần tử n+1. Sẽ có 2 khả năng xảy ra: - Khả năng 1: n+1 không tạo thành 1 tập con mới (tức tập chứa n + 1 có ít nhất 1 phần tử khác) Khi đó, rõ ràng ta có 2 cách bổ sung n+1 (vào 1 trong 2 tập không chứa n). Vậy số cách xây dựng tập con trong trường hợp này là 2S(n) - Khả năng 2: n+1 tạo thành 1 tập con mới. Khi đó, n số từ 1 đến n phải nằm trong 2 tập hợp còn lại. Có thể thấy ngay chỉ có 1 cách chia thỏa mãn (1 tập chứa các số chẵn và tập còn lại chứa các số lẻ). Do đó, số cách trong trường hợp Trường THPT Chuyên Thái Bình 226 HỘI CÁC TRƯỜNG THPT CHUYÊN KHU VỰC DUYÊN HẢI - ĐỒNG BẰNG BẮC BỘ HỘI THẢO KHOA HỌC LẦN THỨ VI này là 1 cách Vậy Mặt ta thu được khác, kiểm công thức truy hồi S(n+1)=2S(n)+1 tra trực tiếp ta có S(3)=1, nên : Như vậy, chia sô cách tập hợp thỏa mãn đề bài là S(n) = 2n − 2 − 1, S(1) = S(2) = 0 Bài 15: Cho A và E là 2 đỉnh đối tâm của 1 hình bát giác đều. Một con ếch bắt đầu nhảy từ A. Tại bất cứ đỉnh nào trừ E, con ếch có thể tới một trong 2 đỉnh kề. Nếu ếch nhảy tới E thì nó dừng lại. Tính số cách để ếch nhảy từ A tới E mất đúng n bước. (n>4) Lời giải: Gọi an là số cách để ếch nhảy từ A đến E mất đúng n bước.(n>4) Dễ thấy a2n-1=0, n≥1 (vì để nhảy từ A tới E cần một số chẵn bước nhảy). Sau 2 bước nhảy, ếch chỉ có thể đên B hoặc C hoặc trở về A. Do đó: a2n= 2(a2n-2+ b2n-2), trong đó: bn là số cách nhảy để ếch nhảy từ B( hoặc C) đến E mất đúng n bước. Từ B (hoặc C), sau 2 bước nhảy ếch chỉ có thể trở về B hoặc đến A (do n>4). Suy ra:b2n = 2b2n-2 + an-2 Từ 2 hệ thức truy hồi trên ta suy ra: a2n = 4a2n-2 – 2a2n-4 Giải hệ thức trên suy ra a 2n = Trường THPT Chuyên Thái Bình ( 1  2+ 2 2  227 ) n −1 ( − 2− 2 ) n −1   . HỘI CÁC TRƯỜNG THPT CHUYÊN KHU VỰC DUYÊN HẢI - ĐỒNG BẰNG BẮC BỘ HỘI THẢO KHOA HỌC LẦN THỨ VI C. MỘT SỐ BÀI TẬP TỰ LUYỆN 1. ( VMO – 1997): n đường tròn chia mặt phẳng làm bao nhiêu phần nếu bất cứ cặp đường tròn nào cũng có hai điểm chung và không có 3 đường tròn nào có điểm chung. 2. ( Estonia 2007): Xét lưới ô vuông 10 x 10. Với mỗi nước đi ta tô màu hình vuông đơn vị nằm ở giao của 2 hàng và 2 cột. Một nước đi là hợp lệ nếu ít nhất 1 trong 4 hình vuông này trước đó không được tô. Hỏi số nước đi lớn nhất có thể để tô toàn bộ lưới ô vuông là bao nhiêu? 3. Cho các số k, n ∈ * và n > 3. Cho đa giác lồi A1A2...An. Hỏi có tất cả bao nhiêu cách tô màu n đỉnh của đa giác lồi đó bằng k mầu sao cho trong mỗi cách tô không có hai đỉnh kề nhau nào cùng được tô một màu. 4. Có bao nhiêu cách chia n cái kẹo cho k em bé ( k < n)c sao cho mỗi em bé có ít nhất một cái kẹo. 5. Các số 1,2,3,...,n ( n > 4) được viết liên tiếp trên một vòng tròn. Hai số không kề nhau được gọi là liên thông nếu một trong hai cung tạo bởi chúng chứa toàn số bé hơn chúng. Tìm số cặp liên thông. Trường THPT Chuyên Thái Bình 228 HỘI CÁC TRƯỜNG THPT CHUYÊN KHU VỰC DUYÊN HẢI - ĐỒNG BẰNG BẮC BỘ HỘI THẢO KHOA HỌC LẦN THỨ VI ĐƯỜNG ĐỐI TRUNG VÀ ĐIỂM LEMOINE TRONG TAM GIÁC Mai Xuân Huy THPT Chuyên Hạ Long, Quảng Ninh Đường đối trung trong tam giác vốn là khái niệm quen thuộc trong hình học. Giao của các đường đối trung là điểm Lemoine. Bài viết dưới đây sẽ tổng hợp lại các kiến thức về đường đối trung và điểm Lemoine để bạn đọc có được cái nhìn cơ bản nhất và hoàn thiện hơn về các khái niệm này. I. Đường đối trung trong tam giác. 1. Đường đẳng giác x 1.1 Định nghĩa: Cho góc xAy , hai tia As và At được gọi z là đẳng giác nếu chúng đối xứng nhau qua tia phân giác của góc xAy . 1.2. Định lý: Cho tam giác ABC với hai đường đẳng giác AA1 và AA2. A AB 2 BA1.BA2 Chứng minh rằng = AC 2 CA1.CA2 A B A1 A2 Trường THPT Chuyên Thái Bình C 229 t y HỘI CÁC TRƯỜNG THPT CHUYÊN KHU VỰC DUYÊN HẢI - ĐỒNG BẰNG BẮC BỘ HỘI THẢO KHOA HỌC LẦN THỨ VI Chứng minh: S ABA1 S ACA2 S ABA2 S ACA1 = AB. AA1 sin BAA1 BA1 AB BA1 AA2 = ⇒ = . (1) AC. AA2 sin CAA2 CA2 AC CA2 AA1 = AB. AA2 sin BAA2 BA2 AB BA2 AA1 = ⇒ = . (2) AC. AA1 sin CAA1 CA1 AC CA1 AA2 Nhân các vế của (1) và (2) ta được điều phải chứng minh. 2. Đường đối trung 2.1 Định nghĩa: Cho tam giác ABC, đường đối trung kẻ từ A của tam giác là đường đối xứng với trung tuyến đỉnh A qua phân giác trong góc A. 2.2 Định lý: Cho tam giác ABC, AE là đường đối trung khi và chỉ khi BE AB 2 . = CE AC 2 Giải: Gọi AM là đường đẳng giác với AE. Khi đó theo A AB 2 BE.BM định lý 1.2 ta có . = AC 2 CE.CM Do đó AE là đường đối trung ⇔ AM là trung tuyến BE AB 2 . = ⇔ BM=CM ⇔ CE AC 2 B Nhận xét: Đường đối trung chia cạnh đối diện của tam giác theo tỉ lệ bằng bình phương tỉ lệ hai cạnh bên. 2.3 Các đường đối trung đồng quy tại một điểm gọi là điểm Lemoine của tam giác. Dễ chứng minh tính chất trên bằng định lý Ceva. Ví dụ 1: Trường THPT Chuyên Thái Bình 230 E M C HỘI CÁC TRƯỜNG THPT CHUYÊN KHU VỰC DUYÊN HẢI - ĐỒNG BẰNG BẮC BỘ HỘI THẢO KHOA HỌC LẦN THỨ VI Chứng minh rằng tiếp tuyến của đường tròn ngoại tiếp tam giác ABC tại hai đỉnh cắt nhau tại điểm nằm trên đường đối trung của tam giác đi qua đỉnh thứ ba. B Xét tam giác ABC như hình vẽ. Ta có M IB S ABI S MBI S MAB AB MB sin MBA = = = = . . IC S ACI S MCI S MAC AC MC sin MCA D I A C = AB sin BCA AB 2 . = AC sin CBA AC 2 Do đó I là chân đường đối trung kẻ từ A của tam giác ABC. Ví dụ 2: Cho tam giác ABC, một đường tròn qua BC cắt các cạnh AB, AC tại M, N. Tìm quỹ A tích trung điểm của MN. Giải: N' M Gọi M’; N’ là điểm đối xứng với M, N qua phân giác góc A. N M' Dễ chứng minh được M’N’ song song với BC. Trung điểm của M’N’ thuộc trung tuyến kẻ từ A nên suy B C ra trung điểm của MN thuộc đường đối trung kẻ tử A của tam giác ABC. 3. Một số tính chất của điểm Lemoine 3.1 Định lý: Cho X là một điểm nằm trên đường đối trung kẻ từ A của tam giác ABC. CMR khoảng cách từ C đến AB và AC tỉ lệ với độ dài của AB và AC. A Giải X Trường THPT Chuyên Thái Bình 231 B E C HỘI CÁC TRƯỜNG THPT CHUYÊN KHU VỰC DUYÊN HẢI - ĐỒNG BẰNG BẮC BỘ HỘI THẢO KHOA HỌC LẦN THỨ VI S AEB d ( X ; AB ) d ( E ; AB ) = = AB d ( X ; AC ) d ( E ; AC ) S AEC AC = EB AC AB 2 AC AB . = . = EC AB AC 2 AB AC Từ định lý trên suy ra tính chất sau: 3.2 Tính chất: Cho L là điểm Lemoine của tam giác ABC, khi đó: d ( L; BC ) d ( L; CA) d ( L; AB ) . = = BC CA AB 3.3 Tính chất: Cho điểm X nằm trong tam giác ABC. Khi đó d 2 ( X ; BC ) + d 2 ( X ; CA) + d 2 ( X ; AB) nhỏ nhất khi và chỉ khi X là điểm Lemoine của tam giác ABC. Chứng minh: A Gọi D, E, F lần lượt là chân đường cao kẻ từ X xuống E F L BC, CA, AB. Ta có a. XD + b. XE + c. XF = 2 S ABC = const nên B D 4 S 2 ≤ ( XD 2 + XE 2 + XF 2 )(a 2 + b 2 + c 2 ) XD 2 + XE 2 + XF 2 ≥ 4S 2 . a 2 + b2 + c2 Dấu “=” xảy ra khi và chỉ khi XD XE XF hay X là điểm Lemoine của tam giác = = a b c ABC. 3.4 Tính chất: Cho tam giác ABC với L là điểm Lemoine. D, E, F là chân đường cao kẻ từ L xuống BC, CA, AB. Khi đó L là trọng tâm tam giác DEF. Giải: Vì L là điểm Lemoine của tam giác ABC nên Trường THPT Chuyên Thái Bình 232 C HỘI CÁC TRƯỜNG THPT CHUYÊN KHU VỰC DUYÊN HẢI - ĐỒNG BẰNG BẮC BỘ HỘI THẢO KHOA HỌC LẦN THỨ VI LD LE LF . = = a b c A Theo định lý con nhím E F L a b c LD + LE + LF = 0 LD LE LF C D B hay LD + LE + LF = 0 nên L là trọng tâm tam giác DEF. Tam giác DEF gọi là tam giác pedal của điểm L. Nhận xét: Bằng định lý con nhím ta còn chỉ ra được a 2 LD + b 2 LE + c 2 LF = 0 3.5 Tính chất: Cho X, Y, Z là các điểm thuộc các cạnh BC, CA, AB của tam giác ABC. Khi đó XY 2 + YZ 2 + ZX 2 nhỏ nhất khi X, Y, Z là các đỉnh của tam giác pedal của điểm Lemoin L. Chứng minh A Gọi G là trọng tâm tâm tam giác XYZ, khi đó F XY + YZ + ZX = 3(GX + GY + GZ ) 2 2 2 2 2 2 Z Kẻ GD, GE, GF vuông góc với BC, CA, AB, ta có GD + GE + GF ≤ GX + GY + GZ 2 2 2 2 2 B E Y G D X 2 Mà theo tính chất 3.3 GD 2 + GE 2 + GF 2 nhỏ nhất khi G là điểm Lemoine của tam giác ABC. Từ đó suy ra đpcm. 3.6 Tính chât: Cho tam giác ABC, khoảng cách từ điểm Lemoine L đến các cạnh của tam giác bằng a; b; c trong đó Trường THPT Chuyên Thái Bình = 2 S ABC . a + b2 + c2 2 233 C HỘI CÁC TRƯỜNG THPT CHUYÊN KHU VỰC DUYÊN HẢI - ĐỒNG BẰNG BẮC BỘ HỘI THẢO KHOA HỌC LẦN THỨ VI Gọi các khoảng cách này lần lượt là La ; Lb ; Lc , theo tính chất 3.2 ta có La Lb Lc = = ⇒ ( La ; Lb ; Lc ) = (a; b; c) a b c Mà 2 S = aLa + bLb + cLc ⇒ 2 S = (a 2 + b 2 + c 2 ) ⇒ = 2 S ABC từ đó suy ra đpcm. a + b2 + c2 2 3.7 Tính chất: Độ dài các cạnh của tam giác pedal của điểm Lemoine L là 2 ma ; 2 mb ; 2 mc , trong đó = 2 S ABC . a + b2 + c2 2 Gọi Lb ; Lc là chân các đường cao kẻ từ L xuống CA; AB, Áp dụng định lý Cosin cho tam giác LLb Lc , ta có: Lb Lc 2 = LLc 2 + LLb 2 − 2 LLc .LLb cos(1800 − A) = 2 (2(b 2 + c 2 ) − a 2 ) = 4 2 ma2 Hệ quả: 1. Cho tam giác ABC với 3 điểm X, Y, Z thuộc BC, CA, AB thì XY 2 + YZ 2 + ZX 2 ≥ 12S 2 . a 2 + b2 + c2 2. Cho X, Y, Z trùng với trung điểm của BC, CA, AB thì ta có 1 2 12 S 2 (a + b 2 + c 2 ) ≥ 2 ⇔ a 2 + b 2 + c 2 ≥ 4 3S 2 2 4 a +b +c 3. Cho X, Y, Z trùng với chân các đường cao kẻ từ A, B, C thì (a 2 + b 2 + c 2 )(a 2 cos 2 A + b 2 cos 2 B + c 2 cos 2 C ) ≥ 12 S 2 3.8 Tính chất: Diện tích tam giác pedal của điểm L được tính bởi S L = 3 12 S ABC . (a 2 + b 2 + c 2 ) Hướng dẫn: Ta tính diện tích tam giác theo độ dài ba đường trung tuyến của nó. Ví dụ 3 Đường thẳng đi qua trung điểm cạnh đáy của tam giác và trung điểm của đường cao tương ứng đi qua điểm Lemoine L. Chứng minh: Trường THPT Chuyên Thái Bình 234 HỘI CÁC TRƯỜNG THPT CHUYÊN KHU VỰC DUYÊN HẢI - ĐỒNG BẰNG BẮC BỘ HỘI THẢO KHOA HỌC LẦN THỨ VI Ta chứng minh bài toán bằng phương pháp vector, trước hết ta dễ chứng minh nhận xét sau: Cho tam giác ABC, H là chân đường cao kẻ từ A. Khi đó HB.cot C = − HC.cot B . Chú ý rằng kết quả trên đúng với mọi tam giác bất kì. A Gọi A2 là chân đường vuông góc kẻ từ A. I, K là trung điểm BC, AA2. L là điểm Lemoine của tam giác ABC. K Ta chứng minh L thuộc KA1. L Giả sử góc C không vuông, khi đó B A2A1 C A2 B a +c −b cot B =− =− 2 a + b2 − c2 cot C A2C 2 2 A2 chia BC theo tỉ lệ − LA2 = a 2 + c2 − b2 nên a 2 + b2 − c2 a 2 + c2 − b2 LC (a 2 + b 2 − c 2 ) LB + (a 2 + c 2 − b 2 ) LC a 2 + b2 − c2 = a 2 + c2 − b2 2a 2 1+ 2 a + b2 − c2 LB + 2 LK = LA + LA2 = = 2 2a 2 LA + (a 2 + b 2 − c 2 ) LB + (a 2 + c 2 − b 2 ) LC 2a 2 2a 2 LA + 2b 2 LB + 2c 2 LC + (a 2 − b 2 − c 2 )( LB + LC ) a 2 − b 2 − c 2 = ( LB + LC ) 2a 2 2a 2 (do a 2 LA + b 2 LB + c 2 LC = 0 ). Đẳng thức cuối suy ra L thuộc KA 1. A 4. Một số tính chất liên quan tới đường tròn M' M N' 4.1 Khái niệm đường đối song: tam giác ABC và các điểm M, N nằm trên AB, AC sao cho MN//BC. M’, N’ là điểm đối xứng với M, N qua phân giác Trường THPT Chuyên Thái Bình 235 B N C HỘI CÁC TRƯỜNG THPT CHUYÊN KHU VỰC DUYÊN HẢI - ĐỒNG BẰNG BẮC BỘ HỘI THẢO KHOA HỌC LẦN THỨ VI trong góc A. Khi đó M’N’ gọi là đường đối song của MN. Nhận xét: 4 điểm M, N, M’, N’ nằm trên một đường tròn. và 4 điểm B, C, M’, N’ nằm trên một đường tròn. 4.2 Tính chất: Đường đối song qua điểm Lemoine của tam giác cắt các cạnh bên của tam giác tạo thành 6 điểm cùng thuộc một đường tròn. A Chứng minh: Gọi 6 điểm như hình vẽ. A2 A3 Xét đường đối song B1C1. Gọi B '; C '; L ' lần lượt là C1 điểm đối xứng với B1;C1;L qua phân giác góc A thì L B1C1//BC và L’ thuộc trung tuyến kẻ từ A, do đó L’ là trung điểm của B1C1 và do đó L là trung điểm của B1 B B2 C3 B1C1. Tương tự L là trung điểm B2A2; A3C3. Do các tứ giác BCC1B1; CAA3C3 nội tiếp nên dễ chứng minh được tam giác LA3B3 cân tại L. Từ đó chỉ ra được L cách đều 6 điểm trên. 4.3 Tính chất: Đường thẳng đi qua điểm Lemoine của tam giác và song song với cạnh đáy cắt các cạnh bên tạo thành 6 điểm cùng thuộc một đường tròn. Trường THPT Chuyên Thái Bình 236 C HỘI CÁC TRƯỜNG THPT CHUYÊN KHU VỰC DUYÊN HẢI - ĐỒNG BẰNG BẮC BỘ HỘI THẢO KHOA HỌC LẦN THỨ VI Chứng minh: Gọi 6 điểm như hình vẽ. Do AHLF là hình bình hành nên HF là đường đối song của tam giác ABC. Do đó theo 4.1 thì 4 điểm H,F,E,D cùng thuộc một đường tròn. A F Tương tự các điểm D,G,K,H và K,E,F,G cùng thuộc H một đường tròn. D Do đó ta chỉ cần chứng minh HDFG là hình thang cân E L B K G C nội tiếp. Thật vậy, ta có AHF = DEF = BCA = BDG Ví dụ 4 Chứng minh rằng tâm đường tròn ở tính chất 4.3 là trung điểm của đoạn thẳng nối điểm Lemoine với tâm đường tròn ngoại tiếp tam giác. A Chứng minh: Gọi J là trung điểm HF. Gọi O là tâm đường tròn ngoại tiếp tam giác ABC F J H và I là trung điểm LO. D Ta chứng minh OA vuông góc với HF B Trường THPT Chuyên Thái Bình 237 L G I E O K C HỘI CÁC TRƯỜNG THPT CHUYÊN KHU VỰC DUYÊN HẢI - ĐỒNG BẰNG BẮC BỘ HỘI THẢO KHOA HỌC LẦN THỨ VI Ta có AOC = 2 ABC nên OAC = 1800 − 2 ABC = 900 − ABC 2 mà HFA = ABC (HFCB nội tiếp) nên OAC + HFA = 900 hay OA ⊥ HF Do đó JI ⊥ HF hay JI là trung trực của HF nên trung trực HF đi qua I. Tương tự trung trực DG cũng đi qua I. Từ đó dễ chỉ ra được I là tâm đường tròn qua 6 điểm D,E,F,G,H,K. Ví dụ 5 Cho tam giác ABC với điểm Lemoine L. Các đường thẳng LA, LB, LC cắt đường tròn ngoại tiếp tam giác tại điểm thứ hai D, E, F. Chứng minh rằng L cũng là điểm Lemoine của tam giác DEF. Trước hết ta chứng minh một số bổ đề sau: Bổ đề 1: Cho tam giác ABC, trọng tâm G của tam giác định ra các góc a;a’;b;b’;c;c’ như hình vẽ mà ta kí hiệu là [(a;a’); (b;b’); (c;c’)] thì điểm Lemoine L định ra các góc [(a’;a);(b’;b);(c’;c)]. A a' a A a a' L G b' B b Trường THPT Chuyên Thái Bình c' c C 238 B b b' c' c C HỘI CÁC TRƯỜNG THPT CHUYÊN KHU VỰC DUYÊN HẢI - ĐỒNG BẰNG BẮC BỘ HỘI THẢO KHOA HỌC LẦN THỨ VI Bổ đề này có được hiển nhiên do tính chất đối xứng của trung tuyến và đường đối trung qua phân giác. Bổ đề 2: Cho tam giác ABC và trọng tâm G định ra các góc [(a;a’); (b;b’); (c;c’)] thì nếu một tam giác T có các góc a’+c; b’+a; c’+b và điểm G’ định ra các góc [(a’;c);(b’;a);(c’;b)] thì G’ là trọng tâm tam giác T. A a a' c D Xét tam giác ABC với các đường trung F C' a b' tuyến AA’; BB’; CC’. c' b E H B' G Từ A kẻ đường thẳng song song với CG cắt b' B BG tại H. Dễ chứng minh được tam giác AGH đồng dạng với tam giác T và trọng tâm E định ra các góc [(a’;c);(b’;a);(c’;b)]. Do đó suy ra G’ là trọng tâm tam giác T. Bổ đề 3: Tam giác T có các góc a’+c; b’+a; c’+b và điểm L định ra các góc [(c;a’);(a;b’);(b;c’)] thì L là điểm Lemoine của tam giác T. Đến đây ta suy ra ngay cách chứng minh bài toán trên Trường THPT Chuyên Thái Bình 239 c b A' c' C HỘI CÁC TRƯỜNG THPT CHUYÊN KHU VỰC DUYÊN HẢI - ĐỒNG BẰNG BẮC BỘ HỘI THẢO KHOA HỌC LẦN THỨ VI A E a' a F a c a' b' L b b' c' c B C c' b D Cuối cùng xin đưa ra một số bài toán để bạn đọc phát hiện ra thêm một số tính chất của đường đối trung và điểm Lemoine Trường THPT Chuyên Thái Bình 240 HỘI CÁC TRƯỜNG THPT CHUYÊN KHU VỰC DUYÊN HẢI - ĐỒNG BẰNG BẮC BỘ HỘI THẢO KHOA HỌC LẦN THỨ VI 1. Cho tam giác ABC với đường đối trung AK. Đường tròn ngoại tiếp tam giác AKC cắt AB tại P và đường tròn ngoại tiếp tam giác AKB cắt AC tại Q. CMR KP=KQ. 2. Cho tam giác ABC, lấy các điểm D, E, F trên BC, CA, AB sao cho DE//AB; DF//AC. Đường tròn ngoại tiếp tam giác DEF cắt BC, CA, AB tại D 1; E1; F1. M là giao điểm của DE và D1F1, N là giao điểm của DF và D1E1. CMR A, M, N thẳng hàng. 3. Cho tam giác ABC với đường đối trung AD. E, F thuộc đường thẳng CA; AB sao cho DE//AB; DF//AC. a. CMR đường tròn ngoại tiếp tam giác DEF tiếp xúc với BC. (đường tròn Tucker) b. CMR nếu AB; AC cắt đường tròn Tucker tại E’ và F’ thì E’F’//BC. 4. Cho tam giác ABC với điểm Lemoine L. A1;B1;C1 đối xứng với L qua BC, CA, AB. Chứng minh rằng L là trọng tâm tam giác A1B1C1. 5. Cho tam giác ABC, đường đối trung AD, điểm Lemoine L, CMR LA b 2 + c 2 . = LD a2 6. Cho tam giác ABC, các đường cao kẻ từ B và C cắt đường phân giác góc A tại P và Q. Đường thẳng qua P song song AB cắt đường thẳng qua Q song song với AC tại R. CMR AR là đường đối trung kẻ từ A của tam giác ABC. 7. Cho tam giác ABC trọng tâm G. Gọi P, Q, R lần lượt là tâm đường tròn ngoại tiếp các tam giác PBC; QCA; RAB. Chứng minh rằng tâm đường tròn ngoại tiếp O và trọng tâm G của tam giác ABC lần lượt là trọng tâm và điểm Lemoine của tam giác PQR. Trường THPT Chuyên Thái Bình 241 HỘI CÁC TRƯỜNG THPT CHUYÊN KHU VỰC DUYÊN HẢI - ĐỒNG BẰNG BẮC BỘ HỘI THẢO KHOA HỌC LẦN THỨ VI 8. Cho tam giác ABC đường đối trung AD. Từ D kẻ DM, DN song song với AC; AB (M thuộc AB, N thuộc AC). CMR tứ giác BMNC nội tiếp, gọi A 1 là tâm đường tròn ngoại tiếp tứ giác trên. Tương tự định nghĩa B1;C1. Chứng minh rằng AA1;BB1;CC1 đồng quy. 9. Cho 2 đường tròn (C) và (C’) cắt nhau tại 2 điểm A,B. PT là tiếp tuyến chung của hai đường tròn. Tiếp tuyến tại P, T của đường tròn ngoại tiếp tam giác APT cắt nhau tại S. H đối xứng với B qua PT. CMR A, H, S thẳng hàng. 10. Cho tam giác ABC nội tiếp đường tròn (O;R), L là điểm Lemoine của tam giác. K là điểm xác định bởi OK .OL = R 2 . KA, KB, KC cắt (O) tại điểm thứ hai A’;B’;C’. CMR L là điểm Lemoine của tam giác A’B’C’. Tài liệu tham khảo 1. Một số chuyên đề hình học phẳng bồi dưỡng học sinh giỏi – Đỗ Thanh Sơn. 2. Các tài liệu sưu tầm trên mạng Internet. Trường THPT Chuyên Thái Bình 242 HỘI CÁC TRƯỜNG THPT CHUYÊN KHU VỰC DUYÊN HẢI - ĐỒNG BẰNG BẮC BỘ HỘI THẢO KHOA HỌC LẦN THỨ VI BÀI TOÁN CON BƯỚM VÀ MỘT SỐ ỨNG DỤNG Đào Thị Lê Dung . THPT Chuyên Thái Bình Bài toán con bướm là một định lý hay và có nhiều ứng dụng trong toán học. Bài viết dưới đây nhằm mục đích giới thiệu nội dung, các cách chứng minh và một vài ứng dụng . I. NỘI DUNG BÀI TOÁN: Cho đường tròn (O) với dây cung AB. I là trung điểm của AB. Qua I dựng hai dây cung MN, PQ khác AB sao cho MP và NQ cắt AB lần lượt tại E và F. Chứng minh I là trung điểm của EF. M N P P I B I E A F B A Q O O Q M N Bài toán trên có nhiều cách chứng minh, mỗi cách đều có cái hay và thú vị riêng. Sau đây ta cùng điểm qua một số cách chứng minh sơ cấp. Cách 1 N P E A F I B D C O Q M Trường THPT Chuyên Thái Bình 243 HỘI CÁC TRƯỜNG THPT CHUYÊN KHU VỰC DUYÊN HẢI - ĐỒNG BẰNG BẮC BỘ HỘI THẢO KHOA HỌC LẦN THỨ VI Gọi C, D lần lượt là trung điểm của MP, NQ, ta có: OC ⊥ MP , OD ⊥ NQ , OI ⊥ AB nên tứ giác IOCE và IODF là các tứ giác nội tiếp. Do vậy: g.IOE = g. ICE, gIOF = g.IDF (g: góc) (1). Dễ thấy hai tam giác IMP và IQN đồng dạng và IC, ID là hai trung tuyến IC IP PM CP = = = nên ∆ICP : ∆IDN ⇒ g.ICE = g.IDF (2). ID IN NQ DN tương ứng nên: Từ (1) và (2) ta được g.IOE = g.IOF nên tam giác OEF cân tại O. Vì vậy I là trung điểm của AB (đpcm). Cách 2(của Coxeter và Greitzer) N P K B C I F E A H D O M Q Gọi C, D thứ tự là hình chiếu vuông góc của E trên IP, IM. Gọi K, H thứ tự là hình chiếu vuông góc của F trên IM, IQ. IE ED IE (1); ∆IEC : ∆IFH ⇒ = = IF FK IF PE EC (3); ∆MED : ∆QFH ⇒ = Ta có: ∆PEC : ∆NFK ⇒ NF FK ∆IED : ∆IFK ⇒ Trường THPT Chuyên Thái Bình 244 EC (2) FH ME ED (4) = QF FH HỘI CÁC TRƯỜNG THPT CHUYÊN KHU VỰC DUYÊN HẢI - ĐỒNG BẰNG BẮC BỘ HỘI THẢO KHOA HỌC LẦN THỨ VI ( IE 2 ED EC ME PE AE BE = = ) = . . . FK FH NF QF AF BF IF Từ (1), (2), (3), (4) ta có: mà AE BE ( AI − EI )( BI + IE ) AI 2 − EI 2 . = = AF BF AI 2 − IF2 ( AI + IF)( IB − IF) IE 2 AI 2 − EI 2 AI 2 ) = = = 1 ⇒ IE = IF IF AI 2 − IF2 AI 2 Vây ( Ta được đpcm. Cách 3 D N P I A B F E O Q M Trường hợp MP và NQ song song là trường hợp đơn giản. Do vậy ta chỉ cần xét MP và NQ giao nhau. Ta gọi D là giao của chúng. Xét tam giác EFD. Theo định lý Menenauyt ta có: IF ME ND IF PE QD IF ME ND PE QD . . . . . . . = 1; = 1 ⇒ ( )2 . =1 IE MD NF IE PD QF IE MD NF PD QF Vì DN.DQ = DP.DM nên ta có: IF2 ME PE IF2 NF QF = ⇒ = . . 1; . IE 2 NF QF IE 2 ME PE Trường THPT Chuyên Thái Bình 245 HỘI CÁC TRƯỜNG THPT CHUYÊN KHU VỰC DUYÊN HẢI - ĐỒNG BẰNG BẮC BỘ HỘI THẢO KHOA HỌC LẦN THỨ VI Mặt khác : NF.QF = AF.BF và ME.PE = EA.EB nên ta có: IF2 AF BF ( AI + IF) ( AI − IF) AI 2 − IF 2 = = = =1 . IE 2 EA EB ( AI − IE) ( AI + IE) AI 2 − IE 2 (đpcm). Cách 4: L N P I B A F E K O Q M Từ F kẻ đường thẳng d song song với MP, cắt MN ở L và cắt PQ ở K. Ta có: g.FLN = g.IME = g.FQK mà LNF và QKF là hai tam giác đồng dạng nên ta có: LF FQ = . Vì vây :LF .FK = FN .FQ = ( AI + IF)( BI − IF) = AI 2 − IF2 FN FK Tương tự : EP.EM = AI2 – IE2 Mặt khác: Trường THPT Chuyên Thái Bình 246 HỘI CÁC TRƯỜNG THPT CHUYÊN KHU VỰC DUYÊN HẢI - ĐỒNG BẰNG BẮC BỘ HỘI THẢO KHOA HỌC LẦN THỨ VI FK EP = . FI EI FL EM FK .FL EP.EM ∆IFN : ∆IEM ⇒ = ⇒ = FI EI FI 2 EI 2 mà LF.FK = AI2 − IF2 , EP.EM = AI2 − IE 2 ∆IEP : ∆IFK ⇒ nên FK .FL EP.EM AI2 − IF2 AI2 − IE 2 AI2 AI2 = ⇔ = ⇔ = ⇔ IE = IF FI 2 EI 2 IF2 IE 2 IF2 IE 2 II. MỘT VÀI BIẾN TẤU CỦA BÀI TOÁN CON BƯỚM Trước tiên ta thay đổi cách phát biểu của bài toán con bướm như sau: Bài toán 1: Cho tứ giác ADBC nội tiếp đường tròn (O). Gọi I là giao hai đường chéo AB và CD. Một đường thẳng d vuông góc với OI cắt AD, BC theo thứ tự tại M và N. Chứng minh I là trung điểm của MN. N/x: Nếu ta đổi vai trò của hai đường chéo và hai cạnh đối diện cho nhau ta thu được bài toán sau. Bài toán 2: Cho tứ giác ABCD nội tiếp đường tròn (O). Gọi I là giao hai đường chứa hai cạnh bên AD và BC. Một đường thẳng d vuông góc với OI cắt AC, BD theo thứ tự tại M và N. Chứng minh I là trung điểm của MN. I N d D C B O A N/x: Ở bài toán trên nếu ta thay hai đường chéo bằng hai cạnh đối diện còn lại thì ta có bài toán 3. Trường THPT Chuyên Thái Bình 247 HỘI CÁC TRƯỜNG THPT CHUYÊN KHU VỰC DUYÊN HẢI - ĐỒNG BẰNG BẮC BỘ HỘI THẢO KHOA HỌC LẦN THỨ VI Bài toán 3 : Cho tứ giác ABCD nội tiếp đường tròn (O). Gọi I là giao hai đường AD và BC. Một đường thẳng d vuông góc với OI cắt AB,C D theo thứ tự tại M và N. Chứng minh I là trung điểm của MN. I M N D C B O A Ở bài toán 2 và 3 phép chứng minh được lặp lại tương tự cách 1 đã trình bày bên trên. N/x: Từ ba bài toán trên ta có thể tổng quát thành bài toán sau: Bài toán 4 : Cho tròn (O), d là một đường thẳng tùy ý không tiếp xúc với (O). Gọi I là hình chiếu vuông góc của O trên d. Qua I kẻ hai cát tuyến IAD và IBC bất kỳ tới (O). Gọi M, N thứ tự là giao của d với AB và CD. Gọi E,F thứ tự là giao của d với AC và BD. Gọi P, Q là giao của d với đường tròn (O) (nếu có). Chứng minh ba đoạn MN, EF, PQ có cùng trung điểm. Rõ ràng nếu d cắt (O) ta được bài toán 1, còn d nằm ngoài (O) ta được bài toán 2 và 3. Đặc biệt nếu ta cho cát tuyến TAD suy biến thành tiếp tuyến tại A khi đó A trùng D, thì ta được bài toán sau: Bài toán 5 : Cho tròn (O) và d là một đường thẳng nằm ngoài (O). Gọi I là hình chiếu vuông góc của O trên d. Qua I kẻ tiếp tuyến IA và cát tuyến IBC tới (O). Gọi M, N thứ tự là giao của d với AB và AC. Chứng minh IM = IN. Trường THPT Chuyên Thái Bình 248 HỘI CÁC TRƯỜNG THPT CHUYÊN KHU VỰC DUYÊN HẢI - ĐỒNG BẰNG BẮC BỘ HỘI THẢO KHOA HỌC LẦN THỨ VI III. ÁP DỤNG BÀI TOÁN CON BƯỚM VÀO GIẢI TOÁN. Khi áp dụng vào bài tập điều quan trọng là ta phải phát hiện và xây dựng được mô hình của định lý con bướm. VD1: (Mongolian TST 2008) Cho tam giác nhon ABC nội tiếp đường tròn (O). Gọi CD là đường cao, H là trực tâm của tam giác ABC. Một đường thẳng đi qua d vuông góc với OD cắt BC tại E. CHứng minh g. DHE = g. ABC. Nhận xét: Coi đường thẳng d là đường thẳng DE thì ta thấy mô hình con bướm xuất hiện , như vậy một cách tự nhiên ta nghĩ đến việc đủ tứ giác nội tiếp ACBF lấy giao của d với AF . Lời giải Gọi F là giao khác C của CD với (O). K là giao của d với AF. Theo một tính chất quen thuộc về trực tâm thi ta có D là trung điểm của FH. Áp dụng bài toán con bướm số 1 ta được D là trung điểm của EK. Suy ra FKHE là hình bình hành hay ta có AF và EH song song. Vậy ta được đpcm vì hai góc đó cùng bằng góc AFD. VD2 : (MOP 1998) Cho hai đường tròn (C) và (C’ ) có cùng bán kính và cắt nhau tại hai điểm A, B. Gọi O là trung điểm của AB. CD là một dây cung của (C) đi qua O. Gọi P là một giao điểm của đoạn thẳng CD với (C’). EF là một dây cung của (C’) qua O và Q là một giao điểm đoạn EF với (C) . Chứng minh AB, CQ, EP đồng quy. Nhận xét: với giả thiết O là trung điểm của AB ta không khó để dựng hình phụ để xuất hiện bài toán con bướm. Lời giải Trường THPT Chuyên Thái Bình 249 HỘI CÁC TRƯỜNG THPT CHUYÊN KHU VỰC DUYÊN HẢI - ĐỒNG BẰNG BẮC BỘ HỘI THẢO KHOA HỌC LẦN THỨ VI K H A M D F I J O Q P B E C Gọi H, K lần lượt là giao điểm thứ hai của CD và (C’), của EF và (C). Gọi S, S’ lần lượt là giao của CQ, EP với AB. M là giao của KD và AB. Áp dụng định lý con bướm ta có O là trung điểm của MS. Mặt khác do hai đường tròn (C ) và (C’) cung bán kính nên O là trung điểm của AB thì O cũng là trung điểm của PD, EK. Do đó tứ giác PDEK là hình bình hành. Suy ra : ∆KOM = ∆EOS'( g .c.g ) ⇒ OM = OS' hay O là trung điểm của MS’. Vậy S, S’ trùng nhau. Ta được đpcm. VD3 : (Singapore 2011) Cho tam giác ABC nhọn, không cân với AB > AC. O, H lần lượt là tâm đường tròn ngoại tiếp và trực tâm của tam giác ABC. Gọi D là chân đường cao hạ từ A của tam giác ABC. Lấy Q thuộc cạnh AC, kéo dài HQ cắt BC tại P sao cho D là trung điểm của PB. Chứng minh ODQ là một góc vuông. Nhận xét: Để chứng minh QR và OD vuông góc ta nghĩ đến việc kéo dài QD để dựng hoàn chỉnh dây cung EF. Khi đó ta cần chứng minh O là trung điểm của EF. Lời giải Trường THPT Chuyên Thái Bình 250 HỘI CÁC TRƯỜNG THPT CHUYÊN KHU VỰC DUYÊN HẢI - ĐỒNG BẰNG BẮC BỘ HỘI THẢO KHOA HỌC LẦN THỨ VI A E P Q O H C D R G B F Gọi G là giao của AD với (O), ta có D là trung điểm của GH. Gọi R là giao của QD và BG. Theo gt suy ra D đồng thời là trung điểm của BP và HG nên BGPH là hình bình hành. Do vậy D là trung điểm của QR. Theo định lý con bướm áp dụng với tứ giác nội tiếp ACGB ta được D cũng là trung điểm của EF. Vậy OD và EF vuông góc hay ta được đpcm. VD4 : (Moldova TST 2010) Cho tam giác nhọn ABC, H là trực tâm và M là trung điểm của BC. Kẻ đường thẳng qua H vuông góc với HM cắt AB, AC lần lượt tại P và Q. Chứng minh MP = MQ. Lời giải Trường THPT Chuyên Thái Bình 251 HỘI CÁC TRƯỜNG THPT CHUYÊN KHU VỰC DUYÊN HẢI - ĐỒNG BẰNG BẮC BỘ HỘI THẢO KHOA HỌC LẦN THỨ VI A D K F Q H E P B M C Coi d là đường thẳng PQ . Vì d vuông góc với HM nên ta nghĩ đến việc dựng một đường tròn tâm M. Gọi D,K lần lượt là chân đường cao ứng với hai đỉnh B, C ta có DKHC là một tứ giác nội tiếp. Dung định lý con bướm ta có ngay H là trung điểm của PQ ta đó ta suy ra đpcm. VD5 : Cho tam giác ABC nội tiếp đường tròn (O), I là tâm đường tròn nội tiếp. Đường thẳng BI, CI cắt đường tròn tại E và F. Gọi K, D lần lượt là giao điểm của AI với EF và BC. Biết AB + AC = 2BC. Chứng minh I là trung điểm của KD. Lời giải Trường THPT Chuyên Thái Bình 252 HỘI CÁC TRƯỜNG THPT CHUYÊN KHU VỰC DUYÊN HẢI - ĐỒNG BẰNG BẮC BỘ HỘI THẢO KHOA HỌC LẦN THỨ VI A F K E I B D C M Gọi M là giao khác A của AI và (O). Ở đây xuất hiện bài toán con bướm với tứ giác nội tiếp là BCEF. Do đó ta cần chứng minh I là trung điểm của AM. Thật vậy: Dễ thấy ∆MAC : ∆BAD( g.g ) ⇒ MC BD ID CD BD + CD BC 1 = = = = = = MA BA IA CA BA + CA 2 BC 2 . Cũng dễ dàng chứng minh tam giác MIC cân tại M nên MC = MI. Do đó IM = IA suy ra đpcm. VD6 : Cho hai tam giác nhọn A1BC và A2 BC cùng nội tiếp đường tròn (O) và trực tâm lần lượt là H1, H2. Gọi M, N thứ tự là giao của H1H2 với A2B và A2C. Biết góc A1H2H1 = 900. Chứng minh A1M = A1N. Lời giải Trường THPT Chuyên Thái Bình 253 HỘI CÁC TRƯỜNG THPT CHUYÊN KHU VỰC DUYÊN HẢI - ĐỒNG BẰNG BẮC BỘ HỘI THẢO KHOA HỌC LẦN THỨ VI A2 A1 B' C' N O H1 H2 M B I C Gọi I là trung điểm BC. Dựng các đường cao BB’ và CC’ của tam giác A2 BC. 0 Trước tiên từ gt góc A1H2H1 = 90 chứng minh A1, H2, I thẳng hàng. Sau đó áp dụng định lý con bướm đối với đường tròn đường kính BC ta được H2M = H2N nên suy ra đpcm. IV. MỞ RỘNG BÀI TOÁN CON BƯỚM Dạng mở rộng: Cho đường tròn (O) với dây cung AB. I là trung điểm của AB. MN và PQ là hai dây cung của (O) cắt AB thứ tự tại P và S . Khi đó PS và EF có cùng trung điểm là I. Bài toán trên còn được gọi là bài toán con bướm kép. Nhưng ta còn thấy rằng định lý con bướm trên phát biểu không chỉ đúng cho đường tròn mà còn đúng với elip, parabol và hypabol nữa gọi chung là ba đường conic. Trường THPT Chuyên Thái Bình 254 HỘI CÁC TRƯỜNG THPT CHUYÊN KHU VỰC DUYÊN HẢI - ĐỒNG BẰNG BẮC BỘ HỘI THẢO KHOA HỌC LẦN THỨ VI Định lý 1: Cho 1 đường conic (C) với AB là một dây cung. Gọi PQ, MN là hai dây cung khác của (C) cùng đi qua trung điểm I của AB. Giả sử MP, NQ thứ tự cắt AB tại E và F. Khi đó I cũng là trung điểm của EF. Nếu thay các đường conic bằng các cặp đường thẳng ta cũng được bài toán tương tự , cụ thể Định lý 2: Cho B, C là hai điểm tùy ý thứ tự thuộc hai đường thẳng d1, d2 phân biệt. Gọi I là trung điểm của BC. Gọi MN, PQ là hai đoạn thẳng cùng đi qua I khác BC, trong đó M, Q thuộc d1; N, P thuộc d2. Giả sử E,F thứ tự là giao của PM ,QN với đường thẳng BC. Khi đó I là trung điểm của EF. Chứng minh: TH1: d1, d2 song song thì đây là trường hợp tầm thường. TH2: d1, d2 cắt nhau tại A. A P M E I B C F N Q Không mất tính tổng quát ta xét hình vẽ bên. Áp dụng định lý Menenauyt cho tam giác ABC với các cát tuyến EMP và FNQ EB PC MA . . = 1 n ên EC PA MB ta có: FC QB NA . . = 1 n ên FB QA NC EB PA MB . (1) = EC PC MA FC QA NC . (2) = FB QB NA Trường THPT Chuyên Thái Bình 255 HỘI CÁC TRƯỜNG THPT CHUYÊN KHU VỰC DUYÊN HẢI - ĐỒNG BẰNG BẮC BỘ HỘI THẢO KHOA HỌC LẦN THỨ VI Áp dụng định lý Menenauyt cho tam giác ABC với các cát tuyến MIN và QIP MB NA IC MB NC . . (3) = 1 vì IB = IC nên = MA NC IB MA NA ta có: PA IC QB PA QA . . (4) = 1 n ên = PC IB QA PC QB EB FC = Từ (1), (2), (3), (4) suy ra EC FB nên BE = CF hay I là trung điểm của EF. V. BÀI TẬP TỰ LUYỆN Bài 1: Cho đường tròn (O) với dây cung PQ và M là trung điểm của PQ.Gọi AB, CD là hai dây cung của (O) cùng đi qua M. Gọi H,K lần lượt là giao điểm của PQ với AC và BD. Chứng minh: HA.HC KB.KD = HM 2 KM 2 Bài 2: Cho tam giác nhọn ABC với AD là đường cao, O, H lần lượt là tâm đường tròn ngoại tiếp và trực tâm tam giác ABC. Kẻ đường thẳng qua D và vuông góc với OD cắt AB ở K. Chứng minh g.DHK + g.AHK = 1800. Bài 3: Cho tam giác ABC với đường tròn nội tiếp (I) và đường tròn ngoại tiếp (O). Đường tròn (I) tiếp xúc với BC tại D. Hai điểm M, S lần lượt là giao điểm của (O) với AI và AO. Trên đt DM ta lấy điểm X, trên đt AO lấy điểm Y sao cho I, X, Y thẳng hàng. Chứng minh: IX = IY ⇔ OI ⊥ XY Bài 4: Cho tam giác ABC với M là trung điểm của BC. Giả sử AM cắt đường tròn nội tiếp(I) tam giác ABC tại K, L. Các đường thẳng song song với BC qua K, L cắt đường tròn (I) tại X, Y. Giả sử AX cắt BC tại P, AY cắt (ABC) tại D, DM cắt (ABC) tại E, AM cắt (ABC) tại F. Chứng minh E,P, F thẳng hàng. Bài 5: Cho tứ giác ABCD nội tiếp đường tròn (O). Các đường chéo AC, BD cắt nhau tại I khác O. Qua I kẻ đường thẳng vuông góc với OI cắt AB, CD lần lượt tại M, N. Chứng minh AB = CD khi và chỉ khi BM = CN. Bài 6: Cho tứ giác ABCD có hai đường chéo cắt nhau tại O. Đường thẳng d qua O cắt các đường tròn ngoại tiếp các tam giác OAB, ABC, OCD, ODA lần lượt Trường THPT Chuyên Thái Bình 256 HỘI CÁC TRƯỜNG THPT CHUYÊN KHU VỰC DUYÊN HẢI - ĐỒNG BẰNG BẮC BỘ HỘI THẢO KHOA HỌC LẦN THỨ VI tại M, N, P, Q khác O. Chứng minh O là trung điểm của đoạn MP khi và chỉ khi nó là trung điểm của đoạn NQ. Bài 7: Cho tứ giác ABCD có hai đường chéo cắt nhau tại I và một đường thẳng d cắt AB, BC, CD, DA lần lượt tại M, N, P, Q. . Chứng minh I là trung điểm của đoạn MP khi và chỉ khi nó là trung điểm của đoạn NQ. Bài 8: Cho tứ giác ABCD có hai đường chéo cắt nhau tại O. Biết tồn tại bốn điểm A’, B’, C’, D’ lần lượt thuộc các cạnh AB, BC, CD, DA sao cho A’B’C’ D’ là hình bình hành tâm O. Chứng minh ABCD cũng là hình bình hành. Bài 9: Cho tứ giác ABCD có góc BAD = góc BCD = 900. Gọi E là giao của hai đường chéo AC và BD. Chứng minh trung điểm nối tâm các đường tròn ngoại tiếp tam giác ABE và CDE thuộc đường thẳng BD. Bài 10: Cho tam giác ABC nội tiếp đường tròn (O). d là đường thẳng tùy ý không đi qua O và P là hình chiếu của O trên d. Đường thẳng d cắt cácđương thẳng chứa các cạnh BC, CA, AB tại X, Y, Z. Gọi X’, Y’, Z’ lần lượt là các điểm đối xứng của X, Y, Z qua P. Chứng minh AX’, BY’, CZ’ đồng quy tại một điểm trên đường tròn (O). VI. LỜI KẾT. Qua việc giới thiệu nội dung và các biến tấu phong phú của mô hình bài toán con bướm, ta thấy đây là một vấn đề hay có nhiều ứng rộng đặc sắc và các hướng phát triển sâu hơn. Tuy nhiên do trình độ còn hạn chế nên bài viết trên chỉ nhằm mục đích tổng hợp và phân tích được một số vấn đề. Rất mong nhận được nhiều ý kiến đóng góp quý giá của đồng nghiệp về các hướng mở rộng và phát triển sâu hơn ứng dụng của nó. Xin chân thành cảm ơn. VII. TƯ LIỆU THAM KHẢO 1. Tài liệu chuyên toán hình học 10. 2. Tạp chí Toán học và Tuổi trẻ. 3. Tài liệu của Hoàng Minh Quân (mạng Mathscope.com). 4.Các chuyên đề hình học bồi dưỡng HSG trung học cơ sở (Trần Văn Tấn) Trường THPT Chuyên Thái Bình 257 HỘI CÁC TRƯỜNG THPT CHUYÊN KHU VỰC DUYÊN HẢI - ĐỒNG BẰNG BẮC BỘ HỘI THẢO KHOA HỌC LẦN THỨ VI Sö dông ph­¬ng ph¸p song ¸nh gi¶i mét sè bµi to¸n TỔ hîp NguyÔn ThÞ Ngäc Ánh Tr- êng THPT Chuyªn Th¸i Nguyªn Email: anhtoan416@gmail.com Thay thÕ cho viÖc ®Õm sè phÇn tö cña mét tËp hîp A nhÊt ®Þnh, ta ®i ®Õm sè phÇn tö cña mét tËp hîp B cã cïng sè phÇn tö víi tËp hîp A. Sè phÇn tö cña tËp hîp B lµ dÔ ®Õm. §Ó cã ®- îc kÕt qu¶ nµy ta cÇn chøng minh cã mét song ¸nh gi÷a hai tËp hîp A vµ B. Mét sè s¸ch tham kh¶o ®· ®Ò cËp ®Õn ph- ¬ng ph¸p nµy, tuy nhiªn nh÷ng bµi to¸n ®- îc nªu ra vÉn ë møc khã so víi ®¹i ®a sè häc sinh. Chóng t«i xin tr×nh bµy ph- ¬ng ph¸p nµy th«ng qua c¸c vÝ dô tõ dÔ ®Õn khã ®Ó gióp c¸c em häc sinh dÔ ®äc, dÔ hiÓu h¬n. Tõ ®ã cã thÓ ¸p dông ph- ¬ng ph¸p song ¸nh vµo gi¶i c¸c bµi to¸n tæ hîp kh¸c. 1. Kh¸i niÖm ®¬n ¸nh, toµn ¸nh, song ¸nh a. ¸nh x¹ f tõ tËp hîp X vµo tËp hîp Y (ký hiÖu f: X → Y) lµ mét quy t¾c cho t- ¬ng øngmçi phÇn tö x ∈ X víi mét phÇn tö x¸c ®Þnh y ∈ Y, phÇn tö y gäi lµ ¶nh cña phÇn tö x, ký hiÖu y = f(x). Víi mçi tËp A ⊂ X: f(A) = { f ( x) x ∈ A Trường THPT Chuyên Thái Bình 258 } gäi lµ ¶nh cña tËp A HỘI CÁC TRƯỜNG THPT CHUYÊN KHU VỰC DUYÊN HẢI - ĐỒNG BẰNG BẮC BỘ HỘI THẢO KHOA HỌC LẦN THỨ VI b. Toµn ¸nh lµ ¸nh x¹ tõ X vµo Y trong ®ã f(X) = Y c. §¬n ¸nh lµ ¸nh x¹ tõ X vµo Y tháa m·n: ∀x1 , x2 ∈ X : x1 ≠ x2 ⇒ f ( x1 ) ≠ f ( x2 ) d. Song ¸nh lµ ¸nh x¹ võa lµ ®¬n ¸nh, võa lµ toµn ¸nh. 2. ¸p dông ph- ¬ng ph¸p song ¸nh vµo gi¶i mét sè bµi to¸n tæ hîp Ký hiÖu A lµ sè phÇn tö cña tËp hîp A. Cho f lµ mét ¸nh x¹ tõ X ®Õn Y: +) f lµ ®¬n ¸nh th× X ≤ Y +) f lµ toµn ¸nh th× X ≥ Y +) f lµ song ¸nh th× X = Y Muèn chøng minh hai tËp hîp X vµ Y cã sè l- îng phÇn tö b»ng nhau ta chøng minh cã mét song ¸nh f tõ X tíi Y. Tøc lµ, chøng minh víi mçi x ∈ X cã duy nhÊt mét phÇn tö y ∈ Y vµ ng- îc l¹i víi mçi y∈ Y cã duy nhÊt mét phÇn tö x ∈ X sao cho y = f(x). VÝ dô 1: Cho mét l- íi gåm c¸c « vu«ng.C¸c nót ®- îc ®¸nh sè tõ 0 ®Õn m theochiÒu tõ tr¸i sang ph¶i vµ tõ 0 ®Õn n theo chiÒu tõ d- íi lªn trªn. Hái cã bao nhiªu ®- êng ®i kh¸c nhau tõ nót (0 ; 0) ®Õn nót (m, n) nÕu chØ cho phÐp ®i trªn c¹nh c¸c « vu«ng theo chiÒu tõ tr¸i sang ph¶i hoÆc tõ d- íi lªn trªn. Trường THPT Chuyên Thái Bình 259 HỘI CÁC TRƯỜNG THPT CHUYÊN KHU VỰC DUYÊN HẢI - ĐỒNG BẰNG BẮC BỘ HỘI THẢO KHOA HỌC LẦN THỨ VI n 0 m,n (m,n) 1 0 1 0 1 0 0 1 m (0,0) Gi¶i: Mét ®- êng ®i nh- thÕ ®- îc xem gåm (m + n) ®o¹n (mçi ®o¹n lµ mét c¹nh « vu«ng). T¹i mçi ®o¹n chØ ®- îc chän mét trong hai gi¸ trÞ ®i lªn (ta m· hãa lµ 1) hay sang ph¶i (ta m· hãa lµ 0). Sè ®o¹n ®i lªn ®óng b»ng n vµ sè ®o¹n sang ph¶i ®óng b»ng m. Nh- vËy, cã mét song ¸nh gi÷ tËp hîp A c¸c ®- êng ®i tháa m·n yªu cÇu bµi to¸n víi tËp hîp B c¸c d·y nhÞ ph©n cã cïng ®é dµi (m + n). Trong mçi d·y nhÞ ph©n ®ã cã ®óng n thµnh phÇn b»ng 1, m thµnh thµnh b»ng 0. DÔ thÊy B = C mn + n ⇒ A = C mn + n VËy sè ®- êng ®icÇn t×m lµ Cmn + n NhËn xÐt: Ta cho mét h¹t chuyÓn ®éng trªn mét ®- êng ®i tháa m·n yªu cÇu bµi to¸n trªn (tøc lµ xuÊt ph¸t tõ ®iÓm (0,0) kÕt thóc t¹i ®iÓm (m, n) vµ chØ ®- îc phÐp ®i lªn hoÆc sang ph¶i). Gäi xi +1 lµ sè ®o¹n mµ h¹t ®ã ®i lªn theo ®- êng th¾ng ®øng cã chØ sè i ( i = 0, 1, 2,…, m). Khi ®ã mçi ®­êng ®i tháa m·n t­¬ng øng víi mét nghiÖm nguyªn kh«ng ©m cña ph- ¬ng tr×nh: x1 + x2 + x3 + ... + xm +1 = n (1) Trường THPT Chuyên Thái Bình 260 HỘI CÁC TRƯỜNG THPT CHUYÊN KHU VỰC DUYÊN HẢI - ĐỒNG BẰNG BẮC BỘ HỘI THẢO KHOA HỌC LẦN THỨ VI Do ®ã, gäi C lµ tËp hîp c¸c nghiÖm nguyªn kh«ng ©m cña ph- ¬ngtr×nh (1) th×: C = A = C mn + n VÝ dô 2: ( Bµi to¸n chia kÑo cña Euler) Cã n c¸i kÑo chia cho (m+1) em bÐ. Hái cã tÊt c¶ bao nhiªu c¸ch chia kÑo. ( BiÕt r»ng trong c¸ch chia cã thÓ cã em kh«ng ®- îcchiÕc kÑo nµo). Gi¶i: XÐt mét c¸ch chia nµo ®ã tháa m·n yªu cÇu cña bµi to¸n. Gäi xi lµ sè kÑo em thø i ®- îcnhËn trong c¸ch chia ®ã. Ta cã: x1 + x2 + ... + xm+1 = n Gäi D lµ tËp hîp c¸c c¸ch chia kÑo tháa m·n yªu cÇu bµi to¸n. Ta cã mèi quan hÖ: D = C = C mn + n ¸p dông kÕt qu¶ bµi to¸n nµy chóng ta cã thÓ gi¶i ®- îc nhiÒu bµi to¸n ®Õm hay trong tæ hîp. VÝ dô 3: Mçi ®Ønh cña mét cöu gi¸c ®Òu ®- îc t« bëi mét mµu xanh hoÆc ®á. Tam gi¸c ®á lµ tam gi¸c cã ba ®Ønh ®- îc t« bëi mµu ®á. T- ¬ng tù cã c¸c tam gi¸c xanh. Chøng minh r»ng trong c¸c tam gi¸c t¹o ra tõ 9 ®Ønh cña cöu gi¸c, cã hai tam gi¸c cïng mµu ®ång d¹ng. Trường THPT Chuyên Thái Bình 261 HỘI CÁC TRƯỜNG THPT CHUYÊN KHU VỰC DUYÊN HẢI - ĐỒNG BẰNG BẮC BỘ HỘI THẢO KHOA HỌC LẦN THỨ VI Gi¶i: A3 A2 A4 A5 A1 A9 A6 A7 A8 Ký hiÖu 9 ®Ønh cña cöu gi¸c theo thø tù lµ: A1, A2, A3,… A9 . ChÝn ®Ønh t« bëi 2 mµu nªn cã Ýt nhÊt 5 ®Ønh cïng 1 mµu. Kh«ng mÊt tæng qu¸t, gi¶ sö 5 ®Ønh ®ã cïng mµu ®á. Do ®ã cã C 53 = 10 tam gi¸c ®á. Gäi (C) lµ ®- êng trßn ngo¹i tiÕp cöu gi¸c. C¸c ®Ønh cña cöu gi¸c chia (C) thµnh 9 cung cã ®é dµi b»ng nhau lµ c¸c cung: A1A2, A2A3, A3A4,…, A9A1. Mçi cung ®ã ta gäi lµ mét m¶nh. LÊy tam gi¸c AiAjAk lµ tam gi¸c tháa m·n: A i A j ≤ A jA k ≤ A k A j . Ký hiÖu aij lµ sè m¶nh trong cung AiAj kh«ng chøa Ak. Khi ®ã, mçi tam gi¸c AiAjAk t- ¬ng øng víi mét bé sè ®Æc tr- ng (aij, ajk, aki). VÝ dô, tam gi¸c A9A2A5 t- ¬ng øng bé sè (2, 3, 4) Trường THPT Chuyên Thái Bình 262 HỘI CÁC TRƯỜNG THPT CHUYÊN KHU VỰC DUYÊN HẢI - ĐỒNG BẰNG BẮC BỘ HỘI THẢO KHOA HỌC LẦN THỨ VI Hai tam gi¸c ®ång d¹ng lµ hai tam gi¸c cã cïng bé sè ®Æc tr- ng bao gåm 3 sè tù nhiªn kh«ng gi¶m (a, b, c) tháa m·n: a + b +c = 9 Ta dÔ dµng liÖt kª ®- îc cã 7 bé tháa m·n ®iÒu kiÖn trªn lµ: (1, 1, 7); (1, 2, 6); (1, 3, 5); (1, 4, 4); (2, 2, 5); (2, 3, 4) ; (3, 3, 3) Chóng ta cã 10 tam gi¸c ®á vµ 7 bé sè nªn sÏ cã 2 tam gi¸c cã cïng bé sè ®Æc tr- ng=> ®iÒu ph¶i chøng minh. VÝ dô 4: [ Balkan 1997] LÊy m vµ n lµ sè tù nhiªn lín h¬n 1. Gäi S tËp hîp cã n phÇn tö. LÊy A1, A2, A3,…,Am lµ nh÷ng tËp con cña S. Gi¶ sö r»ng, cø 2 phÇn tö bÊt kú x, y thuéc S ®Òu cã 1 tËp hîp Ai ( i = 1, m ) tháa m·n ®iÒu kiÖn: nÕu x ∈ Ai th× y ∉ Ai cßn nÕu x ∉ Ai th× y ∈ Ai. Chøng minh r»ng: n ≤ 2m . Gi¶i: Mçi phÇn tö x cña S ta cho t- ¬ng øng víi mét d·y nhÞ ph©n f ( x ) = ( x1 , x2 ,..., xm ) , víi x i = 1 nÕu xi ∈ Ai vµ x i = 0 nÕu xi ∉ Ai . Chóng ta cã ¸nh x¹: f: S → T = {( x , x ,..., x 1 2 m ) xi ∈{0;1}} Trường THPT Chuyên Thái Bình 263 HỘI CÁC TRƯỜNG THPT CHUYÊN KHU VỰC DUYÊN HẢI - ĐỒNG BẰNG BẮC BỘ HỘI THẢO KHOA HỌC LẦN THỨ VI Tõ gi¶ thiÕt ta cã, nÕu x ≠ y ⇒ f ( x ) ≠ f ( y ) . VËy f lµ mét ®¬n ¸nh . ⇒ S ≤T Mçi phÇn tö cña T lµ mét d·y nhÞ ph©n cã ®é dµi m nªn T = 2m . VËy n ≤ 2 . m VÝ dô 5: Cho tam gi¸c ABC ®Òu cã ®é dµi c¹nh b»ng a. Chia nhá tam gi¸c ABC thµnh mét l- íi a2 tam gi¸c ®Òu c¹nh b»ng 1 bëi c¸c ®o¹n th¼ng song song víi c¸c c¹nh AB, BC, CA (xem h×nh vÏ). §Õm sè h×nh b×nh hµnh giíi h¹n bëi nh÷ng ®o¹n th¼ng cña l- íi nãi trªn. Gi¶i: Chóng ta nhËn thÊy mçi h×nh b×nh hµnh nãi trªn cã ®óng 2 c¹nh song song víi 2 c¹nh nµo ®ã cña ∆ ABC. A c b b' Trường THPT Chuyên Thái Bình c' 264 HỘI CÁC TRƯỜNG THPT CHUYÊN KHU VỰC DUYÊN HẢI - ĐỒNG BẰNG BẮC BỘ HỘI THẢO KHOA HỌC LẦN THỨ VI Ta ký hiÖu SBC lµ tËp c¸c h×nh b×nh hµnh giíi h¹n bëi nh÷ng ®o¹n th¼ng cña l- ií tháa m·n cã hai c¹nh song song víi AB, AC. T- ¬ng tù, ta cã 2 tËp hîp n÷a lµ SAB vµ SAC. Do tÝnh ®èi xøng ta cã: S AB = S AC = S BC Trªn c¹nh AB kÐo dµi vÒ phÝa B, lÊy B’ sao cho B n¾m gi÷a A, B’ vµ BB’ = 1. T­¬ng tù, ta dùng C’ (xem h×nh vÏ). LÊy P lµ mét h×nh b×nh hµnh trong tËp hîp S BC . KÐo dµi c¸c c¹nh cña P c¾t ®o¹n th¾ng B’C’ t¹i 4 ®iÓm ph©n biÖt. Trªn ®o¹n B’C’ cã (a+2) ®iÓm (tÝnh c¶ hai ®Çu nót B’, C’) chia ®o¹n B’C’ thµnh c¸c ®o¹n th¼ng liªn tiÕp cã ®é dµi b»ng 1. DÔ thÊy, cã mét song ¸nh tõ SBC ®Õn tËp hîp (a +2) ®iÓm nµy. ⇒ S BC = C a4+ 2 VËy sè h×nh b×nh hµnh tháa m·n b»ng: S AB + S AC + S BC = 3 S BC = 3Ca4+ 2 VÝ dô 6: Cho n lµ mét sè tù nhiªn, n ≥ 2 . XÐt d·y S = (1, 2, 3,…, n). Mét d·y con cña S ®- îc gäi lµ d·y céng tÝnh nÕu nã lµ mét cÊp sè céng cã Ýt nhÊt hai phÇn tö. Mét d·y céng tÝnh ®- îc gäi lµ dµi nhÊt nÕu d·y nµy kh«ng thÓ kÐo dµi thªm b»ng c¸ch bæ xung vµo d·y mét phÇn tö nµo ®ã cña S. §Õm sè d·y con céng tÝnh dµi nhÊt cña S ? Trường THPT Chuyên Thái Bình 265 HỘI CÁC TRƯỜNG THPT CHUYÊN KHU VỰC DUYÊN HẢI - ĐỒNG BẰNG BẮC BỘ HỘI THẢO KHOA HỌC LẦN THỨ VI Gi¶i : Tr- íc tiªn ta xÐt tr- êng hî p n ch½n. Gi¶ sö, n = 2m ( m nguyªn d- ¬ng). LÊy a1 < a2 < ... < ak lµ mét d·y con céng tÝnh dµi nhÊt cña S, ( k nguyªn k ≥ 2 ). Ta cã nhËn xÐt : a1 ≤ m . ThËt vËy: Gi¶ sö ng- îc l¹i a1 ≥ m + 1, suy ra a2 − a1 ≤ 2m − (m + 1) = m − 1. V× 2a1 − a2 = a1 − (a2 − a1 ) ≥ m + 1 − (m − 1) = 2 nªn ta cã thÓ thªm vµo d·y con nãi trªn sè ( 2a1 − a2 ) ®Ó lµm t¨ng ®é dµi cña nã. §iÒu nµy m©u thuÉn víi tÝnh chÊt dµi nhÊt cña d·y con. VËy a1 ≤ m .T- ¬ng tù, ta còng chøng minh ®- îc a2 ≥ m + 1 . Do ®ã, mét d·y con céng tÝnh dµi nhÊt ph¶i cã hai sè h¹ng liªn tiÕp ai vµ ai +1 víi 1 ≤ ai ≤ m < m + 1 ≤ ai +1 ≤ n . Chóng ta cho t- ¬ng øng mçi d·y con víi mét cÆp sè gåm hai sè h¹ng liªn tiÕp nãi trªn (ai , ai +1 ) . Mçi cÆp ®ã x¸c ®Þnh duy nhÊt mét d·y con céng tÝnh . Hai cÆp kh¸c nhau th× c«ng sai cña hai d·y kh¸c nhau. Ta t¹o ra c¸c d·y con céng tÝnh dµi nhÊt tõ c¸c d·y con céng tÝnh nµy b»ng c¸ch më réng hÕt vÒ hai phÝa cña (ai , ai +1 ) sao cho trong ®o¹n [1, n] d·y nµy kh«ng thÓ më réng thªm n÷a. VËy t- ¬ng øng mçi cÆp (ai , ai +1 ) víi mét d·y con céng tÝnh dµi nhÊt lµ mét song ¸nh. Cã m c¸ch chän ai vµ m c¸ch chän ai +1 nªn cã m2 d·y con céng tÝnh dµi nhÊt trong tr- êng hîp nµy. T- ¬ng tù nÕu n lµ sè lÎ, n = 2m + 1 ( m nguyªn d- ¬ng). Sè d·y con céng tÝnh dµi nhÊt trong tr- êng hîp nµy lµ m.(m + 1) d·y. VÝ dô 7 : Trường THPT Chuyên Thái Bình 266 HỘI CÁC TRƯỜNG THPT CHUYÊN KHU VỰC DUYÊN HẢI - ĐỒNG BẰNG BẮC BỘ HỘI THẢO KHOA HỌC LẦN THỨ VI Mét tËp hîp cã ni vËt gièng nhau cã cïng kÝ hiÖu i; i = 1, 2,..., m. Cã bao nhiªu c¸ch chän ra Ýt nhÊt mét vËt tõ tËp hîp trªn ? Gi¶i : Chóng ta nhËn xÐt : Cho sè tù nhiªn N cã d¹ng ph©n tÝch tiªu chuÈn lµ: N = p1n1 . p2n2 ..... pmnm trong ®ã p1 , p2 ,..., pm lµ c¸c sè nguyªn tè. Mét - ícbÊt kú cña a a a N cã d¹ng p1 1 . p2 2 ..... pm m ; 0 ≤ ai ≤ ni , i = 1, m . Sè ai cã (ni + 1) c¸ch chän nªn theo quy t¾c nh©n, N cã tÊt c¶ (n1 + 1).(n2 + 1)....(nm + 1) − 1 - íc kh¸c 1. Ta coi mçi vËt cã kÝ hiÖu i lµ pi . Mçi c¸ch chän tháa m·n bµi to¸n nãi trªn t- ¬ng øng víi mét - íc sè kh¸c 1 cña N. (VÝ dô,c¸ch chän mét vËt p1 t- ¬ng øng víi - ícp1 , c¸ch chän hai vËt trong ®ã cã mét vËt p1 vµ mét vËt p2 t- ¬ng øng víi - ícp1. p2 cña N…). VËy sè c¸ch chän tháa m·n yªu cÇu bµi to¸n lµ: (n1 + 1).(n2 + 1)....(nm + 1) −1 VÝ dô 8 : §Ó xem mét buæi biÓu diÔn xiÕc, mçi ng- êi ph¶i mua mét vÐ vµo gi¸ 1 USD. Mçi kh¸n gi¶ chØ ®- îc phÐp mua mét vÐ. Mäi ng- êi ®Õn mua vÐ ®øng xÕp thµnh mét hµng däc tr- íc cöa b¸n vÐ. Mçi ng- êi chØ mang ®óng mét tê 1 USD hoÆc ®óng 1 tê 2 USD. Ng- êi b¸n vÐ quªn kh«ng mang theotiÒn. Gi¶ sö cã n ng- êi mang tê 1 USD vµ m ng- êi mang tê 2 USD (m ≤ n ). T×m sè c¸ch xÕp hµng sao cho ng- êi cã tê 1 USD th× ®- îc nhËn ngay vÐ, ng- êi cã tê 2 USD th× khi ®Õn l- ît cña m×nh ®- îc nhËn ngay vÐ vµ mét tê 1 USD tr¶ l¹i ? Trường THPT Chuyên Thái Bình 267 HỘI CÁC TRƯỜNG THPT CHUYÊN KHU VỰC DUYÊN HẢI - ĐỒNG BẰNG BẮC BỘ HỘI THẢO KHOA HỌC LẦN THỨ VI Gi¶i : M· hãa ng- êi cã tê 1 USD bëi sè 1, ng- êi cã tê 2 USD bëi sè 2. Mçi c¸ch xÕp hµng bÊt kú t- ¬ng øng víi mét vÐc t¬ cã (m+n) thµnh phÇn trong ®ã n thµnh phÇn b»ng 1, m thµnh phÇn b»ng 2. Thµnh phÇn thø i t- ¬ng øng víi ng- êi xÕp hµng ë vÞ m trÝ thø i. Sè vÐc t¬ nh- thÕ lµ Cn+m . Mét vÐc t¬ gäi lµ tèt nÕu t- ¬ng øng víi c¸ch xÕp hµng tháa m·n yªu cÇu bµi to¸n. C¸c vÐc t¬ cßn l¹i gäi lµ c¸c vÐc t¬ xÊu. Chóng ta ®i ®Õm xem cã bao nhiªu vÐc t¬ xÊu b»ng c¸ch x©y dùng mét song ¸nh tõ tËp A c¸c vÐc t¬ xÊu ®Õn tËp B c¸c vÐc t¬ cã (m + n + 1) thµnh phÇn . Mçi vÐc t¬ cña B cã hai tÝnh chÊt : i, Cã m thµnh phÇn 2, (n+1) thµnh phÇn 1 ii, Thµnh phÇn 2 ®øng vÞ trÝ ®Çu tiªn. m−1 Ta cã B = Cm+n . C¸ch x©y dùng song ¸nh nh- sau: - Gi¶ sö v lµ mét vÐc t¬ xÊu, tøc lµ tõ thµnh phÇn ®Çu tiªn ®Õn hÕt thµnh phÇn thø ( i-1) th× t- ¬ng øng víi viÖc mua vÐ diÔn ra su«n sÎ. §Õn thµnh phÇn thø i t- ¬ng øng víi ng- êi thø i mua vÐ nh- ng ng- êi b¸n vÐ kh«ng cã tiÒn tr¶ l¹i. VÞ trÝ i lóc nµy ta gäi lµ vÞ trÝ xÊu. Nh- vËy, tõ thµnh phÇn 1 tíi hÕt (i-1) cã sè l- îng thµnh phÇn 1 b»ng sè l- îng thµnh phÇn 2. X©y dùng mét vÐc t¬ v ' b»ng c¸ch thùc hiÖn hai b- íc: - B- íc 1: Thªm thµnh phÇn 1 vµo tr- íc thµnh phÇn ®Çu tiªn cñav . Khi ®ã, vÞ trÝ xÊu lµ ( i +1). Trường THPT Chuyên Thái Bình 268 HỘI CÁC TRƯỜNG THPT CHUYÊN KHU VỰC DUYÊN HẢI - ĐỒNG BẰNG BẮC BỘ HỘI THẢO KHOA HỌC LẦN THỨ VI - B- íc 2: Tõ vÞ trÝ ®Çu tiªn cña vÐc t¬ ë b- íc 1 tíi hÕt vÞ trÝ (i+1), thay c¸c gi¸ trÞ 1 bëi 2 vµ gi¸ trÞ 2 bëi 1. C¸c thµnh phÇn tõ vÞ trÝ (i+2) trë ®i gi÷ nguyªn gi¸ trÞ cò. Sau hai b- íc trªn ta thu ®- îc vÐc t¬v ' thuéc tËp B. - XÐt vÐc t¬ bÊt kú u ' bÊt kú thuéc B gäi j lµ sè tù nhiªn bÐ nhÊt tháa m·n tõ vÞ trÝ 1 ®Õn hÕt vÞ trÝ j tháa m·n sè thµnh phÇn 1 b»ng sè thµnh phÇn 2. Thao t¸c ng- îc l¹i ë trªn, tõ vÞ trÝ 1 tíi hÕt vÞ trÝ j ta thay 2 bëi 1 vµ 1 bëi 2. C¸c vÞ trÝ cßn l¹i gi÷ nguyªn nh- cò. Bá ®i sè 1 ë hµnh phÇn ®Çu tiªn ta ®- îc mét vÐc t¬ xÊu thuéc A. VËy cã mét song ¸nh tõ A ®Õn B nªn sè vÐc t¬ tèt b»ng: Cnm+m - Cmm+−n1 §©y còng lµ kÕt qu¶ cÇn t×m cña bµi to¸n. Phương pháp song ánh có nội dung đơn giản nhưng lại có ứng dụng rất sâu sắc và có hiệu quả trong nhiều bài toán đếm. Qua các ví dụ nêu trên, chúng tôi hy vọng, các em học sinh mới làm quen với phương pháp này có thể tích lũy cho mình một số kinh nghiệm nhất định trong việc giải Toán rời rạc. Các đồng nghiệp có thể tìm thấy vài ví dụ lạ và thú vị minh họa cho bài giảng của mình trên lớp. Với kinh nghiệm giảng dạy còn ít ỏi, bài viết chắc chắn còn có sai sót. Kính mong được sự đóng góp của các bạn đồng nghiệp. Chúng tôi xin chân thành cảm ơn! Sau đây là một số bài tập dành cho các bạn tự luyện. 3. Mét sè bµi tËp ®Ò nghÞ: Trường THPT Chuyên Thái Bình 269 HỘI CÁC TRƯỜNG THPT CHUYÊN KHU VỰC DUYÊN HẢI - ĐỒNG BẰNG BẮC BỘ HỘI THẢO KHOA HỌC LẦN THỨ VI Bµi 1: Thªm gi¶ thiÕt ë vÝ dô 8 r»ng cã ®· cã q ng- êi xÕp hµng s½n . Mçi ng- êi trong hä ®Òu cã ®óng mét tê 1 USD. Hái cã bao nhiªu c¸ch xÕp hµng tháa m·n? Bµi 2: [AIME] Cã 8 chiÕc nhÉn ph©n biÖt, h·y t×m sè c¸ch ®eo 5 chiÕc nhÉn trong sè 8 chiÕc ®· cho vµo 4 ngãn tay cña cïng mét bµn tay ( kh«ng ®eo vµo ngãn tay c¸i). BiÕt r»ng ta kh«ng quan t©m ®Õn thø tù cña nh÷ng chiÕc nhÉn trªn mét ngãn tay vµ trong c¸ch ®eo cã thÓ cã ngãn tay kh«ng cã nhÉn nµo. Bµi 3: Tung ®ång thêi 5 con sóc s¾c c©n ®èi ®ång chÊt. TÝnh x¸c suÊt ®Ó tæng c¸c chÊm xuÊt hiÖn trªn 5 mÆt b»ng 14? Bài 4: Cho tập hợp A = {1; 2; 3; …;18}. Có bao nhiêu cách chọn ra năm số trong tập A sao cho hiệu của hai số bất kì trong năm số đó không nhỏ hơn hai. Bài 5: Một cửa hàng có m loại kem khác nhau. Một khách hàng cần mua n cốc kem ở đó. Hỏi: a, Người khách hàng đó có tất cả bao nhiêu sự lựa chọn? b, Người khách hàng đó có tất cả bao nhiêu sự lựa chọn sao cho cả m loại kem đều có mặt trong mỗi sự lựa chọn? Bài 6: Có bao nhiêu số nguyên dương nhỏ hơn 1.000.000 mà có tổng bằng 15? Bài 7: (HSGQG - 2012) Cho một nhóm gồm 5 cô gái, ký hiệu và G1, G2, G3, G4, G5 và 12 chàng trai. Có 17 chiếc ghế được sắp thành một hàng ngang. Người ta xếp nhóm người đã cho ngồi vào các chiếc ghế đó sao cho các điều kiện sau được đồng thời thoả mãn: 1, Mỗi ghế có đúng một người ngồi; 2, Thứ tự ngồi của các cô gái, xét từ trái qua phải, là G1, G2, G3, G4, G5; 3, Giữa G1 và G2 có ít nhất 3 chàng trai; Trường THPT Chuyên Thái Bình 270 HỘI CÁC TRƯỜNG THPT CHUYÊN KHU VỰC DUYÊN HẢI - ĐỒNG BẰNG BẮC BỘ HỘI THẢO KHOA HỌC LẦN THỨ VI 4, Giữa G4 và G5 có ít nhất 1 chàng trai và nhiều nhất 4 chàng trai; Hỏi có tất cả bao nhiêu các xếp như vậy? (Hai cách xếp được coi là khác nhau nếu tồn tại một chiếc ghế mà người ngồi ở chiếc ghế đó trong hai cách xếp là khác nhau) Bµi 8: Mét c¬ quan an ninh ®Æc biÖt cã 15 nh©n viªn. Mçi nh©n viªn cã mét thÎ tõ ®Ó ra vµo cöa. Cã m m· ph©n biÖt ®­îc l­u trong mçi thÎ tõ. Cã 15 thÎ kh¸c nhau. §Ò më cöa, mçi ng­êi ph¶i ®­a thÎ tõ vµo mét m¸y kiÓm tra ë cöa . Trong m¸y cã tËp hîp A gåm n m· ph©n biÖt. TËp hîp m m· trong mçi thÎ lµ tËp con A. Hîp cña tÊt c¶ c¸c m· cã trong 6 thÎ bÊt kú chÝnh b»ng A. V× lý do an ninh cöa chØ më khi cã Ýt nhÊt 6 thÎ cïng ®­a vµo m¸y kiÓm tra. T×m gi¸ trÞ cña n vµ m tháa m·n n nhá nhÊt vµ mäi yªu cÇu ë trªn ®­îc tháa m·n? ( BiÕt r»ng thø tù cña c¸c phÇn tö trong c¸c tËp hîp nãi trªn lµ kh«ng quan träng). TÀI LIỆU THAM KHẢO: [1] .Phan Huy Khải (2002), Các phương pháp giải toán sơ cấp 12, Nhà xuất bản Hà Nội. [2]. Đề thi học sinh giỏi quốc gia môn toán lớp 12 năm 2012. [3] .Một số thông tin trên mạng Internet. [4]. V.K. Balakrishnan, Ph.D (1995), Theory and problems of combinatorics, McGraw-Hill, INC, Singapore. [5]. Titu Andreescu Zuming Feng (2004), A Path to Combinatoricts for Undergraduates ( Counting Strategies), Birkhauser Boston, United states of America. Trường THPT Chuyên Thái Bình 271 [...]... Để giải bài toán trong trường hợp tổng quát, ta cần chỉ ra một cách chia tổng quát hoặc quy được trường hợp n lớn về với các trường hợp n nhỏ hơn Qua việc nghiên cứu các trường hợp nhỏ lẻ, thấy khá khó có thể tìm quy luật tổng quát cho việc phân chia, bởi vậy, ta thử tìm cách đưa các trường hợp sau về trường hợp trước đó, các trường hợp n=5,6,7,8,9 cũng gần như rất khó đưa được về trường hợp trước,... Mathlinks.ro Trường THPT Chuyên Thái Bình 24 HỘI CÁC TRƯỜNG THPT CHUYÊN KHU VỰC DUYÊN HẢI - ĐỒNG BẰNG BẮC BỘ HỘI THẢO KHOA HỌC LẦN THỨ VI Trường THPT Chuyên Thái Bình 25 HỘI CÁC TRƯỜNG THPT CHUYÊN KHU VỰC DUYÊN HẢI - ĐỒNG BẰNG BẮC BỘ HỘI THẢO KHOA HỌC LẦN THỨ VI Trường THPT Chuyên Thái Bình 26 HỘI CÁC TRƯỜNG THPT CHUYÊN KHU VỰC DUYÊN HẢI - ĐỒNG BẰNG BẮC BỘ HỘI THẢO KHOA HỌC LẦN THỨ VI Trường THPT Chuyên. .. HỘI CÁC TRƯỜNG THPT CHUYÊN KHU VỰC DUYÊN HẢI - ĐỒNG BẰNG BẮC BỘ HỘI THẢO KHOA HỌC LẦN THỨ VI Trường THPT Chuyên Thái Bình 28 HỘI CÁC TRƯỜNG THPT CHUYÊN KHU VỰC DUYÊN HẢI - ĐỒNG BẰNG BẮC BỘ HỘI THẢO KHOA HỌC LẦN THỨ VI Trường THPT Chuyên Thái Bình 29 HỘI CÁC TRƯỜNG THPT CHUYÊN KHU VỰC DUYÊN HẢI - ĐỒNG BẰNG BẮC BỘ HỘI THẢO KHOA HỌC LẦN THỨ VI Trường THPT Chuyên Thái Bình 30 HỘI CÁC TRƯỜNG THPT CHUYÊN... KHOA HỌC LẦN THỨ VI Trường THPT Chuyên Thái Bình 31 HỘI CÁC TRƯỜNG THPT CHUYÊN KHU VỰC DUYÊN HẢI - ĐỒNG BẰNG BẮC BỘ HỘI THẢO KHOA HỌC LẦN THỨ VI Trường THPT Chuyên Thái Bình 32 HỘI CÁC TRƯỜNG THPT CHUYÊN KHU VỰC DUYÊN HẢI - ĐỒNG BẰNG BẮC BỘ HỘI THẢO KHOA HỌC LẦN THỨ VI Trường THPT Chuyên Thái Bình 33 HỘI CÁC TRƯỜNG THPT CHUYÊN KHU VỰC DUYÊN HẢI - ĐỒNG BẰNG BẮC BỘ HỘI THẢO KHOA HỌC LẦN THỨ VI Trường. .. KHU VỰC DUYÊN HẢI - ĐỒNG BẰNG BẮC BỘ HỘI THẢO KHOA HỌC LẦN THỨ VI Trường THPT Chuyên Thái Bình 34 HỘI CÁC TRƯỜNG THPT CHUYÊN KHU VỰC DUYÊN HẢI - ĐỒNG BẰNG BẮC BỘ HỘI THẢO KHOA HỌC LẦN THỨ VI Trường THPT Chuyên Thái Bình 35 HỘI CÁC TRƯỜNG THPT CHUYÊN KHU VỰC DUYÊN HẢI - ĐỒNG BẰNG BẮC BỘ HỘI THẢO KHOA HỌC LẦN THỨ VI Trường THPT Chuyên Thái Bình 36 ... tử 5 ≤ x ≤ 10 , nên có thể chi 1 5 = 5 + 1 0 = 6 + 9 = 7 + 8 Trường hợp n = 5 (5-5-5) sang trường hợp ( 22-22-22), chênh lên 17 phần tử mỗi tập, số phần tử mỗi tập thuộc {6,7,8,9,10,11} mà 17=6+11=7+10=8+9 Trường THPT Chuyên Thái Bình 18 HỘI CÁC TRƯỜNG THPT CHUYÊN KHU VỰC DUYÊN HẢI - ĐỒNG BẰNG BẮC BỘ HỘI THẢO KHOA HỌC LẦN THỨ VI ******* Từ đây, ta có lời giải tổng quát bằng quy nạp với việc xây dựng... để phân hoạch tập { 1 , 2 , … , n } thành các tập 2 phần tử thỏa mãn đề bài trong từng trường hợp và chỉ ra với số k lớn hơn các số đã định thì không thể phân chia được Ngoài ra, với 2 trường hợp n=5m+1, n=5m+2 có thể chọn chung các tập, 3 trường hợp n=5m+3, 5m+4, 5m+5 cũng có thể chọn chung các tập Vậy chủ yếu làm việc với 2 trường hợp n=5m+1 và n=5m+3 Ta có lời giải sau: ******* Với n=5m+1,5m+2 ta... đến cách chia lớp theo kiểu đồng dư modulo Trường THPT Chuyên Thái Bình n + 1 thì mỗi lớp có đúng 11 1 C 2nn n +1 phần tử HỘI CÁC TRƯỜNG THPT CHUYÊN KHU VỰC DUYÊN HẢI - ĐỒNG BẰNG BẮC BỘ HỘI THẢO KHOA HỌC LẦN THỨ VI Quan trọng là xem xét cái gì thay đổi khi chuyển vị trí để dựa vào đó làm căn cứ chia lớp theo modulo n + 1 Ta hãy bắt đầu với các trường hợp nhỏ lẻ Trước hết, nghiên cứu sự thay đổi khi... vì số lượng chênh mỗi màu là như nhau, từ 3-3-4 lên 18-18-19, tức là chênh đều lên 15, và ta cần có 3 phần, mỗi phần 15 phần tử cùng màu, được chia thành các tập có nhiều hơn hoặc bằng 5 phần tử ( 3-3-4 lên 9-9-10 chênh mỗi tập lên 6 phần tử, nhưng mỗi tập đều nhiều hơn 5 phần tử thì không thể làm vậy) Vậy, nguyên nhân có thể đưa được từ trường hợp n=10 về trường hợp n=4 là vì: Mỗi tập chênh 15 phần... phép biến đổi Trường THPT Chuyên Thái Bình 22 |A+E-2C| A-E E A A-C C A-E A-C A C-E E C-E HỘI CÁC TRƯỜNG THPT CHUYÊN KHU VỰC DUYÊN HẢI - ĐỒNG BẰNG BẮC BỘ HỘI THẢO KHOA HỌC LẦN THỨ VI A B F C E A-C A A-E C D E C-E A-E A-C C A-C C-E C-E làm giảm tính lẻ của tổng A+C+E nếu không có số nào bằng 0, là một phép biến đổi đáng lưu ý Từ đây đặt ra câu hỏi: Khi có 1 số bằng 0 thì có đưa về trường hợp 2 số bằng ...HỘI CÁC TRƯỜNG THPT CHUYÊN KHU VỰC DUYÊN HẢI - ĐỒNG BẰNG BẮC BỘ HỘI THẢO KHOA HỌC LẦN THỨ VI Chuyên đề xếp loại xuất sắc BẮT ĐẦU TỪ NHỮNG TRƯỜNG HỢP NHỎ Nguyễn Thế Sinh Giáo viên THPT chuyên Nguyễn... 15-15-15 18-18-19 22-22-22 Để giải toán trường hợp tổng quát, ta cần cách chia tổng quát quy trường hợp n lớn với trường hợp n nhỏ Qua việc nghiên cứu trường hợp nhỏ lẻ, thấy khó tìm quy luật tổng... chia, vậy, ta thử tìm cách đưa trường hợp sau trường hợp trước đó, trường hợp n=5,6,7,8,9 gần khó đưa trường hợp trước, với n=10, ta đưa n=4, số lượng chênh màu nhau, từ 3-3-4 lên 18-18-19, tức chênh

Ngày đăng: 10/10/2015, 22:46

Từ khóa liên quan

Tài liệu cùng người dùng

Tài liệu liên quan